Int

  • April 2020
  • PDF

This document was uploaded by user and they confirmed that they have the permission to share it. If you are author or own the copyright of this book, please report to us by using this DMCA report form. Report DMCA


Overview

Download & View Int as PDF for free.

More details

  • Words: 67,955
  • Pages: 290
TEORIA CALCOLO INTEGRALE

Indice I Teoria dell’integrazione di una funzione reale di variabile reale 5 1 Integrale definito 1.1 Funzioni primitive . . . . . . . . . . . . . . 1.2 Misura degli insiemi piani . . . . . . . . . 1.3 Area del rettangoloide . . . . . . . . . . . 1.4 Definizione di integrale definito . . . . . . 1.5 Proprietà dell’integrale definito . . . . . . 1.5.1 Proprietà additiva . . . . . . . . . 1.5.2 Proprietà distributiva . . . . . . . . 1.6 Teorema della media . . . . . . . . . . . . 1.7 Teorema fondamentale del calcolo integrale

. . . . . . . . .

6 6 8 14 19 22 23 23 23 25

2 Integrale indefinito 2.1 Definizione di integrale indefinito . . . . . . . . . . . . . . . . . . 2.2 Integrali indefiniti fondamentali . . . . . . . . . . . . . . . . . . .

28 28 29

. . . . . . . . .

. . . . . . . . .

. . . . . . . . .

. . . . . . . . .

. . . . . . . . .

. . . . . . . . .

. . . . . . . . .

. . . . . . . . .

. . . . . . . . .

. . . . . . . . .

. . . . . . . . .

. . . . . . . . .

II Esercizi sull’integrazione di una funzione reale di variabile reale 31 3 Integrali indefiniti 3.1 Integrali indefiniti fondamentali . . . . . . . . . . . . . 3.2 Soluzioni . . . . . . . . . . . . . . . . . . . . . . . . . . 3.2.1 Soluzioni . . . . . . . . . . . . . . . . . . . . . . 3.3 Integrali di somme di funzioni . . . . . . . . . . . . . . 3.3.1 Soluzioni . . . . . . . . . . . . . . . . . . . . . . 3.4 Integrali di una f (ξ (x)), con ξ (x) funzione lineare . . 3.4.1 Soluzioni . . . . . . . . . . . . . . . . . . . . . . 3.5 Integrazione per introduzione sotto il segno di integrale 3.5.1 Calcolare gli integrali: . . . . . . . . . . . . . . 3.5.2 Soluzioni . . . . . . . . . . . . . . . . . . . . . . 3.5.3 Soluzioni . . . . . . . . . . . . . . . . . . . . . .

. . . . . . . . . . .

. . . . . . . . . . .

. . . . . . . . . . .

. . . . . . . . . . .

. . . . . . . . . . .

. . . . . . . . . . .

32 32 32 34 34 34 36 36 39 40 40 49

INDICE

3.6

3.7 3.8

3.9

3.10 3.11

3.12

2

3.5.4 Calcolare gli integrali: . . . . . . . . . . . . . . . . . . . . 57 Integrazione per sostituzione . . . . . . . . . . . . . . . . . . . . . 61 3.6.1 Calcolare gli integrali: . . . . . . . . . . . . . . . . . . . . 64 3.6.2 Soluzioni . . . . . . . . . . . . . . . . . . . . . . . . . . . . 64 3.6.3 Calcolare i seguenti integrali utilizzando le sostituzioni trigonometriche . . . . . . . . . . . . . . . . . . . . . . . . . . . . 69 Z Z 2 sin xdx, cos2 xdx . . . . . . . . . . . . . . . . . . . . . . . . 72

Integrazione per parti . . . . . . . . . . . . . . . . . . . . . . . . . 3.8.1 Calcolare gli integrali: . . . . . . . . . . . . . . . . . . . . 3.8.2 Soluzioni . . . . . . . . . . . . . . . . . . . . . . . . . . . . 3.8.3 Calcolare gli integrali: . . . . . . . . . . . . . . . . . . . . 3.8.4 Soluzioni . . . . . . . . . . . . . . . . . . . . . . . . . . . . 3.8.5 Formule ricorrenti . . . . . . . . . . . . . . . . . . . . . . . 3.8.6 Calcolare i seguenti integrali applicando il metodo più opportuno . . . . . . . . . . . . . . . . . . . . . . . . . . . . 3.8.7 Soluzioni . . . . . . . . . . . . . . . . . . . . . . . . . . . . Integrali contenenti un trinomio di secondo grado . . . . . . . . . 3.9.1 Calcolare gli integrali: . . . . . . . . . . . . . . . . . . . . 3.9.2 Soluzioni . . . . . . . . . . . . . . . . . . . . . . . . . . . . 3.9.3 Calcolare gli integrali . . . . . . . . . . . . . . . . . . . . . 3.9.4 Soluzioni . . . . . . . . . . . . . . . . . . . . . . . . . . . . 3.9.5 Calcolare gli integrali . . . . . . . . . . . . . . . . . . . . . 3.9.6 Soluzioni . . . . . . . . . . . . . . . . . . . . . . . . . . . . Coppia di integrali notevoli . . . . . . . . . . . . . . . . . . . . . Esercizi riepilogativi sugli integrali contenenti un trinomio di secondo grado . . . . . . . . . . . . . . . . . . . . . . . . . . . . . . 3.11.1 Calcolare i seguenti integrali . . . . . . . . . . . . . . . . . 3.11.2 Calcolare i seguenti integrali . . . . . . . . . . . . . . . . . 3.11.3 Soluzioni . . . . . . . . . . . . . . . . . . . . . . . . . . . . Integrazione delle funzioni razionali . . . . . . . . . . . . . . . . . 3.12.1 Funzioni razionali proprie . . . . . . . . . . . . . . . . . . Z dx 3.12.2 . . . . . . . . . . . . . . . . . . . . . . . . . . . . (x2 +1)2 3.12.3 Calcolare i seguenti integrali 3.12.4 Soluzioni . . . . . . . . . . . 3.12.5 Calcolare i seguenti integrali 3.12.6 Soluzioni . . . . . . . . . . . 3.12.7 Calcolare i seguenti integrali 3.12.8 Soluzioni . . . . . . . . . . . 3.12.9 Calcolare i seguenti integrali 3.12.10 Soluzioni . . . . . . . . . . . 3.12.11 Funzioni razionali improprie

. . . . . . . . .

. . . . . . . . .

. . . . . . . . .

. . . . . . . . .

. . . . . . . . .

. . . . . . . . .

. . . . . . . . .

. . . . . . . . .

. . . . . . . . .

. . . . . . . . .

. . . . . . . . .

. . . . . . . . .

. . . . . . . . .

. . . . . . . . .

. . . . . . . . .

. . . . . . . . .

. . . . . . . . .

73 74 74 77 78 81

85 85 87 90 91 95 95 98 98 103 104 104 106 107 108 109 117 117 117 126 127 134 134 143 143 150

INDICE

3

3.12.12 Calcolare i seguenti integrali . . . . . . . . . . . . . . . . . 151 3.12.13 Soluzioni . . . . . . Z . . . . . . . . . . . . . . . . . . . . . . 151 Z dx 3.13 In (x) = (x2 −1)n , Jn (x) = (x2dx . . . . . . . . . . . . . . . . 157 +1)n

3.14 Integrali di funzioni irrazionali . . . . . . . . . . . . . . . 3.14.1 Integrali del “tipo 1” . . . . . . . . . . . . . . . . 3.14.2 Calcolare i seguenti integrali . . . . . . . . . . . . 3.14.3 Soluzioni . . . . . . . . . . . . . . . . . . . . . . . 3.14.4 Integrali del “tipo 2” . . . . . . . . . . . . . . . . 3.14.5 Calcolare i seguenti integrali . . . . . . . . . . . . 3.14.6 Integrali del “tipo 3” . . . . . . . . . . . . . . . . 3.14.7 Calcolare i seguenti integrali . . . . . . . . . . . . 3.14.8 Soluzioni . . . . . . . . . . . . . . . . . . . . . . . 3.14.9 Integrali del “tipo 4” . . . . . . . . . . . . . . . . 3.14.10 Calcolare i seguenti integrali . . . . . . . . . . . . 3.14.11 Soluzioni . . . . . . . . . . . . . . . . . . . . . . . 3.15 Esercizi riepilogativi sugli integrali di funzioni irrazionali 3.15.1 Calcolare i seguenti integrali . . . . . . . . . . . . 3.15.2 Soluzioni . . . . . . . . . . . . . . . . . . . . . . . 3.16 Integrali di funzioni trigonometriche . . . . . . . . . . . . 3.16.1 Integrali del “tipo 1” . . . . . . . . . . . . . . . . 3.16.2 Integrali del “tipo 2” . . . . . . . . . . . . . . . . 3.16.3 Esercizi . . . . . . . . . . . . . . . . . . . . . . . 3.16.4 Soluzioni . . . . . . . . . . . . . . . . . . . . . . . 3.16.5 Integrali del “tipo 3” . . . . . . . . . . . . . . . . 3.16.6 Esercizi . . . . . . . . . . . . . . . . . . . . . . . 3.16.7 Soluzioni . . . . . . . . . . . . . . . . . . . . . . . 3.16.8 Integrali del “tipo 4” . . . . . . . . . . . . . . . . 3.16.9 Esercizi . . . . . . . . . . . . . . . . . . . . . . . 3.16.10 Soluzioni . . . . . . . . . . . . . . . . . . . . . . . 3.16.11 Esercizi riepilogativi sugli integrali trigonometrici 3.17 Integrazione delle funzioni iperboliche . . . . . . . . . . . 3.17.1 Esercizi . . . . . . . . . . . . . . . . . . . . . . . 3.18 Sostituzioni trigonometriche ed iperboliche . . . . . . . .

. . . . . . . . . . . . . . . . . . . . . . . . . . . . . .

. . . . . . . . . . . . . . . . . . . . . . . . . . . . . .

. . . . . . . . . . . . . . . . . . . . . . . . . . . . . .

. . . . . . . . . . . . . . . . . . . . . . . . . . . . . .

. . . . . . . . . . . . . . . . . . . . . . . . . . . . . .

161 161 162 162 170 172 175 175 176 178 179 180 185 185 185 198 198 204 205 205 215 215 215 217 218 219 228 234 235 238

4 Integrali definiti 4.1 Somme integrali . . . . . . . . . . . . . . . . . . . . . . . . . . . . 4.1.1 Soluzioni . . . . . . . . . . . . . . . . . . . . . . . . . . . . 4.2 Teorema fondamentale del calcolo integrale. . . . . . . . . . . . .

239 239 239 243

5 Estensione del concetto di integrale 257 5.1 Introduzione . . . . . . . . . . . . . . . . . . . . . . . . . . . . . . 257 5.2 Rettangoloide generalizzato . . . . . . . . . . . . . . . . . . . . . 260

INDICE A Integrali notevoli Z Z 2 A.1 sin xdx, cos2 xdx . . . Z dx . . . . . . . . . . . A.2 (x2 −1)n Z dx A.3 . . . . . . . . . . . (x2 +1)n Z Z dx ; (cosdxx)n . . . . . . A.4 (sin x)n Z Z n A.5 (tan x) dx; (cot x)n dx

4 285 . . . . . . . . . . . . . . . . . . . . . 285 . . . . . . . . . . . . . . . . . . . . . 285 . . . . . . . . . . . . . . . . . . . . . 286 . . . . . . . . . . . . . . . . . . . . . 287 . . . . . . . . . . . . . . . . . . . . . 287

Parte I Teoria dell’integrazione di una funzione reale di variabile reale

Capitolo 1 Integrale definito 1.1

Funzioni primitive

Sia f (x) una funzione reale definita in (a, b) ⊂ R.

Definizione. Dicesi funzione primitiva o semplicemente primitiva di f (x), ogni funzione reale F (x) definita in (a, b) e tale che: d F (x) = f (x) (1.1) dx Il problema fondamentale del calcolo integrale consiste nel determinare eventuali primitive di una assegnata funzione f (x). Osserviamo innanzitutto che se F (x) è una primitiva di f (x), per ogni c ∈ R, F (x) + c è ancora una primitiva di f (x): d [F (x) + c] = f (x) (1.2) dx In altri termini se una funzione è dotata di una primitiva, tale funzione ammette infinite primitive. Inoltre: (F1 (x) , F2 (x) primitive di f (x)) =⇒ (F1′ (x) = f (x) , F2′ (x) = f (x)) da cui: F1′ (x) − F2′ (x) = 0 =⇒ F1 (x) − F2 (x) = C, essendo C una costante reale

Cioè due primitive di un’assegnata funzione f (x) differiscono per una costante additiva. Si conclude che se f (x) ammette una primitiva F (x) in (a, b), la totalità delle primitive di f (x) è: G (x) = F (x) + c, con c variabile da −∞ a +∞.

(1.3)

1.1 Funzioni primitive

7 ***

Supponiamo ora che f (x) sia continua in [a, b] e non negativa. Consideriamo il grafico di f (x), cioè la curva di equazione Γ)y = f (x)1 . Sussite la seguente Definizione. Dicesi rettangoloide di base [a, b] relativo alla funzione f (x), il sottoinsieme di R2 :  R = (x, y) ∈ R2 | a ≤ x ≤ b, 0 ≤ y ≤ f (x) (1.4) Ciò premesso, consideriamo il sottoinsieme di R:

R (ξ) = {(x, y) ∈ R | a ≤ x ≤ ξ, 0 ≤ y ≤ f (x)} ,

(1.5)

F : ξ → misR (ξ) ,

(1.6)

essendo ξ ∈ [a, b]. Quindi la funzione reale di variabile reale: essendo misR (ξ) l’area di R (ξ). Incrementiamo la variabile ξ:

ξ → ξ + ∆ξ | ( ξ + ∆ξ) ∈ [a, b]

Da cui l’incremento di F (ξ):

Siano:

∆F = F (ξ + ∆ξ) − F (ξ)

def

m (ξ) =

min f (x)

[ξ,ξ+∆ξ]

def

M (ξ) = max f (x) [ξ,ξ+∆ξ]

Senza perdita di generalità supponiamo che sia ∆ξ > 0: m (ξ) ∆ξ ≤ ∆F ≤ M (ξ) ∆ξ =⇒ m (ξ) ≤

∆F ≤ M (ξ) ∆ξ

Per una nota proprietà delle funzioni continue in un intervallo chiuso: 

   ∆F ∆F m (ξ) ≤ ≤ M (ξ) =⇒ ∃ξ∗ ∈ [ξ, ξ + ∆ξ] | f (ξ∗ ) = ∆ξ ∆ξ f (x) è continua in [ξ, ξ + ∆ξ] 1

Tale curva giace nel semipiano y ≥ 0, giacché f (x) è non negativa per ipotesi.

1.2 Misura degli insiemi piani

8

o, ciò che è lo stesso: ∃θ ∈ [0, 1] |

F (ξ + ∆ξ) − F (ξ) = f (ξ + θ∆ξ) ∆ξ

Da ciò segue (tenendo conto della continuità di f (x)): F (ξ + ∆ξ) − F (ξ) = f (ξ) ∆ξ→0 ∆ξ lim

(1.7)

Poiché ξ è una variabile muta, la (1.7) può scriversi: F ′ (x) = f (x)

(1.8)

Cioè la funzione (1.6) che associa ad ogni ξ ∈ [a, b] l’area di R (ξ) è una primitiva di f (x). Da tale conclusione si evince che il problema della ricerca delle funzioni primitive di un’assegnata funzione f (x) è direttamente connesso al problema della misura di un insieme piano.

1.2

Misura degli insiemi piani

Fissiamo un riferimento cartesiano ortogonale (Oxy). Siano A (a′ , a′′ ), B (b′ , b′′ ) punti del piano, essendo a′ ≤ a′′ , b′ ≤ b′′ . Definizione. Dicesi rettangolo chiuso di estremi A, B, l’insieme di punti:  (x, y) ∈ R2 | a′ ≤ x ≤ b′ , a′′ ≤ y ≤ b′′ (1.9) Definizione. Dicesi rettangolo aperto di estremi A, B, l’insieme di punti:  (x, y) ∈ R2 | a′ < x < b′ , a′′ < y < b′′ (1.10) Le suddette definizioni si specializzano nel caso in cui vale la disuguaglianza in senso stretto a sinistra o a destra (reattangolo semiaperto a sinistra e rettangolo semiaperto a destra, rispettivamente). Definizione. Il centro del rettangolo (1.9 o 1.10) è il punto C (x0 , y0 ) tale che: x0 =

a′′ + b′′ a′ + b ′ , y0 = 2 2

Le dimensioni del rettangolo (1.9 o 1.10) sono i numeri reali non negativi: 2α = b′ − a′ , 2β = b′′ − a′′ Le semidimensioni del rettangolo sono i numeri reali non negativi α, β.

1.2 Misura degli insiemi piani

9

Assegnato il rettangolo chiuso (1.9): 2x0 = a′ + b′ 2α = b′ − a′ , da cui:

o, ciò che è lo stesso:

x0 − α ≤ x ≤ x0 + α, |x − x0 | ≤ α

Similmente:

|y − y0 | ≤ β

Si conclude che il rettangolo chiuso (1.9) può essere scritto come: 

(x, y) ∈ R2 | |x − x0 | ≤ α, |y − y0 | ≤ β

mentre il rettangolo chiuso (1.10): 

(x, y) ∈ R2 | |x − x0 | < α, |y − y0 | < β



Definizione. Dicesi intorno rettangolare di centro P0 , ogni rettangolo aperto di centro P0 . *** Assegnato un punto P0 (x0 , y0 ) e un r ∈ (0, +∞): Definizione. Dicesi cerchio chiuso di centro P0 e raggio r, l’insieme:  (x, y) ∈ R2 | (x − x0 )2 + (y − y0 )2 ≤ r2 Definizione. Dicesi cerchio aperto di centro P0 e raggio r, l’insieme:  (x, y) ∈ R2 | (x − x0 )2 + (y − y0 )2 < r2

Definizione. Dicesi intorno circolare di centro P0 , ogni cerchio aperto di centro P0 . Nel seguito ci riferiremo indifferentemente ad un intorno rettangolare o circolare di un assegnato punto P del piano. Indicheremo tale intorno con I (P ). ***

1.2 Misura degli insiemi piani

10

Sia A ⊂ R2 . def

Definizione. A è limitato ⇐⇒ ∃ cerchio γ ⊃ A di centro O (0, 0) def

A è non limitato ⇐⇒ ∀ cerchio γ di centro O (0, 0), (R − γ) ∩ A 6= ∅. def

Definizione. P0 è interno ad A ⇐⇒ ∃I (P0 ) ⊂ A def

P0 è esterno ad A ⇐⇒ ∃I (P0 ) : I (P0 ) ∩ A = ∅ def

P0 è punto di frontiera per A ⇐⇒ ∀I (P0 ) , I (P0 ) * A, I (P0 ) ∩ A 6= ∅ Definizione. Dicesi frontiera di A l’insieme di punti: ∂A = {P | P è di frontiera per A} Indichiamo con il simbolo Å l’insieme dei punti interni di A: def

Å = {P ∈ A | P è punto interno}

Ciò premesso, sia u l’unità di misura dei segmenti. Conseguentemente l’unità di misura delle aree è u2 . Indichiamo con P la classe dei poligoni. Ad ogni poligono π ∈ P possiamo associare la sua misura µ (π) ∈ [0, +∞) rispetto all’unità di misura u. Resta perciò definita la seguente funzione non negativa: µ : P −→ R+ 0

(1.11)

∀π1 , π2 ∈ P : π 1 ∩ π 2 = ∅ =⇒ µ (π1 ∪ π2 ) = µ (π1 ) + µ (π2 )

(1.12)

La (1.11) verifica le seguenti proprietà: 1. Proprietà additiva: ◦



2. Invarianza per conguenza: π1 , π2 ∈ P : π1 , π2 congruenti =⇒ µ (π1 ) = µ (π2 ) Consideriamo ora un insieme A ⊂ R2 limitato. Se A ∈ / P, si pone il problema della definizione di µ (A). A tale scopo costruiamo gli insiemi: Σ1 = {π ∈ P | π ⊂ A} Σ2 = {Π ∈ P | Π ⊃ A} Osservazione. Se A è privo di punti interni, allora Σ1 = ∅.

(1.13)

1.2 Misura degli insiemi piani

11

Poniamo: def

α (A) = {µ (π) | π ∈ Σ1 } def

β (A) = {µ (Π) | Π ∈ Σ2 } Evidentemente: (∀π ∈ Σ1 , ∀Π ∈ Σ2 , π ⊆ Π) =⇒ µ (π) ≤ µ (Π) ,

cioè α (A) e β (A) sono insiemi separati.

Definizione. Si definisce misura interna di A il numero reale non negativo µi (A) = sup α (A) Definizione. Si definisce misura esterna di A il numero reale non negativo µe (A) = inf β (A) Risulta: µi (A) ≤ µe (A) Osservazione. Se A è privo di punti interni: def

Σ1 = ∅ =⇒ α (A) = ∅ =⇒ µi (A) = 0 Definizione. L’insieme A è misurabile (secondo Peano - Jordan) se risulta µi (A) = µe (A). In tal caso si pone: def

µ (A) = µi (A) = µe (A) , essendo µ (A) la misura di A. Osservazione. Se A è misurabile ed è privo di punti interni, segue necessariamente che µ (A) = 0, poiché è µi (A) = 0. Dalla definizione di misurabilità segue che A è misurabile se e solo se gli insiemi α (A) e β (A) sono contigui: ∀ε > 0, ∃πε ∈ Σ1 , ∃Πε ∈ Σ2 | µ (Πε ) − µ (πε ) < ε

(1.14)

Nel caso speciale in cui A è misurabile e privo di punti interni, la (1.14) si scrive: ∀ε > 0, ∃Πε ∈ Σ2 | µ (Πε ) < ε

1.2 Misura degli insiemi piani

12

Resta così definita la misura µ degli insiemi limitati A ⊂ R2 non appartenenti a P: µ : M → R+ 0

(1.15)

 M = A ⊂ R2 | A è limitato e misurabile

(1.16)

essendo:

Teorema 1. P ⊂ M

Dimostrazione. Osserviamo che i poligoni sono particolari insiemi limitati e misurabili, donde: ∀π ∈ P, π ∈ M (1.17)

Consideriamo ora un qualunque cerchio di centro (x0 , y0 ) e raggio r:  γ = (x, y) ∈ R2 : (x − x0 )2 + (y − y0 )2 ≤ r2

Abbiamo: µ (γ) = πr2 ; cioè γ è limitato e misurabile =⇒ γ ∈ M. Ma γ ∈ / P, donde (tenendo conto della (1.15)) l’asserto. *** La misura (1.15) conserva le proprietà (additività e congruenza) della misura (1.11). Rispetto a quest’ultima possiede la proprietà di monotonia: ∀A, B ∈ M | A ⊆ B =⇒ µ (A) ≤ µ (B)

Altre proprietà. Se A, B ∈ M

A ∪ B ∈ M, A − B ∈ M, A ∩ B ∈ M

(1.18)



A ∩ B = ∅ =⇒ µ (A + B) = µ (A) + µ (B) A ⊆ B =⇒ µ (A − B) = µ (A) − µ (B) ∀A, B ∈ M, µ (A ∪ B) = µ (A) + µ (B) − µ (A ∩ B) Dall’ultima delle (1.18) segue: A, B ∈ M, µ (A ∪ B) ≤ µ (A) + µ (B)

Ciò si esprime dicendo che la misura (1.15) è subadditiva. ***

(1.19)

1.2 Misura degli insiemi piani

13

La nozione di misura di un insieme si estende facilmente al caso di insiemi non limitati. A tale scopo sia A ⊂ R2 un insieme non limitato. Definizione.

def

A è misurabile ⇐⇒ ∀X ∈ M, A ∩ X ∈ M

In tal caso, la misura di A è:

µ (A) = sup µ (A ∩ X) ≤ +∞ X∈M

Precisamente: µ (A) < +∞ =⇒ A è di misura finita µ (A) = +∞ =⇒ A è di misura infinita Indichiamo con R la classe dei rettangoli Θ (α, β) di centro l’origine e dimensioni α, β:  Θ (α, β) = (x, y) ∈ R2 | |x| ≤ α, |y| ≤ β

Ciò premesso, sussiste il seguente Teorema 2.

(A è misurabile) ⇐⇒ (∀Θ ∈ R, A ∩ Θ ∈ M)

Se A è misurabile:

µ (A) = sup µ (A ∩ Θ) Θ∈R

Dimostrazione. Implicazione diretta Abbiamo: (A è misurabile) =⇒ (∀X ∈ M, A ∩ X ∈ M) =⇒ (∀Θ ∈ R, Θ ∩ A ∈ M) R⊂M

Implicazione inversa L’ipotesi è: ∀Θ ∈ R, A ∩ Θ ∈ M Preso ad arbitrio X ∈ M scegliamo Θ ∈ R tale che Θ ⊃ X, donde: A ∩ X = (A ∩ Θ) ∩ X Quindi A ∩ Θ ∈ M =⇒ A ∩ X ∈ M

cioè la tesi. Inoltre, nelle medesime ipotesi:

sup µ (A ∩ Θ) ≤ µ (A) ≤ µ (A ∩ Θ) =⇒ µ (A) = sup µ (A ∩ Θ)

Θ∈R

Θ∈R

1.3 Area del rettangoloide

1.3

14

Area del rettangoloide

Premettiamo la seguente Definizione. Sia A 6= ∅. Gli insiemi non vuoti A1 , A2 , ..., An costituiscono una partizione di A se: n [

Ak = A

k=1 ◦

\◦ Ak Ak′ = ∅, per k, k ′ ∈ {1, 2, ..., n} con k 6= k ′

Sia f (x) continua in [a, b] ⊂ R e ivi non negativa. Eseguiamo una partizione dell’intervallo [a, b] attraverso n + 1 punti: x0 , x1 , ...., xn ∈ [a, b]

Precisamente:

a = x0 < x1 < x2 < ... < xn−1 < xn = b Si tratta di una partizione, poiché: n−1 [ k=0

[xk , xk+1 ] = [a, x1 ] ∪ [x1 , x2 ] ∪ ... ∪ [xn−1 , b] = [a, b] ∀k, k ′ ∈ {0, 1, ..., n − 1} con k 6= k ′ , (xk , xk+1 ) ∩ (xk′ , xk′ +1 ) = ∅

Indichiamo tale partizione con il simbolo convenzionale D (x0 , x1 , ..., xn ). Poniamo per definizione: δ = max (xk+1 − xk ) , N = {0, 1, 2, ..., n − 1} k∈N

Il numero reale δ > 0 si chiama ampiezza della partizione. Inoltre, se f (x) non è identicamente nulla consideriamo il minimo e il massimo di f (x) in [xk , xk+1 ]: mk = min f (x) [xk ,xk+1 ]

Mk = max f (x) [xk ,xk+1 ]

Quindi:  (x, y) ∈ R2 | xk ≤ x ≤ xk+1 , 0 ≤ y ≤ mk def  Rk = (x, y) ∈ R2 | xk ≤ x ≤ xk+1 , 0 ≤ y ≤ Mk def

rk =

(1.20)

1.3 Area del rettangoloide

15

Cioè rk è il rettangolo di base [xk , xk+1 ] e altezza mk , mentre Rk è il rettangolo di base [xk , xk+1 ] e altezza Mk . Restano così definiti i seguenti poligoni:

π (D) = Π (D) =

n−1 [

(1.21)

rk

k=0 n−1 [

Rk

k=0

Definizione. π (D) è il plurirettangolo inscritto a R associato alla partizione D. Π (D) è il plurirettangolo circoscritto a R associato alla partizione D. Evidentemente:

def

sD = µ [π (D)] =

n−1 X

µ (rk ) =

k=0

def

SD = µ [Π (D)] =

n−1 X

n−1 X

µ (Rk ) =

k=0

mk (xk+1 − xk )

k=0 n−1 X k=0

(1.22)

Mk (xk+1 − xk )

Inoltre: ∀D, D′ , π (D) ⊆ Π (D′ ) =⇒ ∀D, D′ , sD ≤ SD′ Teorema 3. Nelle suddette ipotesi il rettangoloide  R = (x, y) ∈ R2 : a ≤ x ≤ b, 0 ≤ y ≤ f (x) è misurabile, risultando:

µ (R) = supsD = inf SD D

D

Dimostrazione. Se f (x) è identicamente nulla, l’asserto è banale:  R = (x, y) ∈ R2 : a ≤ x ≤ b, y = 0 =⇒ µ (R) = 0 ∀D, sD = SD = 0 =⇒ supsD = inf SD = 0 D

D

Consideriamo quindi il caso non banale, cioè f (x) non identicamente nulla in [a, b]. Per il teorema di Heine-Cantor, la funzione è ivi uniformemente continua, per cui: ∀ε > 0, ∃δε > 0 : x′ , x′′ ∈ [a, b] , |x′ − x′′ | < δε =⇒ |f (x′ ) − f (x′′ )| <

ε b−a

1.3 Area del rettangoloide

16

¯ (x0 , x1 , ..., xn ) dell’intervallo [a, b] di ampiezza δ¯ < δε . Eseguiamo una partizione D Siano x¯k , x¯′k ∈ [xk , xk+1 ] : f (¯ xk ) = mk , f (¯ x′k ) = Mk Quindi:

|¯ x′k − x¯k | ≤ xk+1 − xk ≤ δ¯ < δε =⇒ Mk − mk < Inoltre:

ε b−a

n−1 ε X (xk+1 − xk ) = ε SD¯ − sD¯ = (Mk − mk ) (xk+1 − xk ) < b − a k=0 k=0 n−1 X

Ma SD¯ e sD¯ sono le aree di due poligoni:  ¯ = πε π D  ¯ = Πε Π D

Perciò:

∀ε > 0, ∃Πε , πε : µ (Πε ) − µ (πε ) < ε

ciò implica la misurabilità di R. Infine:

sD¯ ≤ µ (R) , sD¯ > SD − ε, cioè: SD¯ − µ (R) < ε

Similmente:

(1.23)

µ (R) ≤ SD¯ , SD¯ < sD¯ + ε

per cui: Dalle (1.23)-(1.24) segue:

µ (R) − sD¯ < ε µ (R) = supsD¯ = inf SD¯ D

D

*** Dal teorema appena dimostrato segue che ∀n ∈ N, ∀D (x0 , x1 , ..., xn ): sD = valore approssimato per difetto di µ (R) SD = valore approssimato per eccesso di µ (R)

(1.24)

1.3 Area del rettangoloide

17

Scegliere come valore approssimato di µ (R) la somma sD , equivale ad approssimare ∀k ∈ N , il rettangoloide: 

(x, y) ∈ R2 | xk ≤ x ≤ xk+1 , 0 ≤ y ≤ f (x)

(1.25)

con il rettangolo rk in esso inscritto. Viceversa, scegliere come valore approssimato di µ (R) la somma SD , equivale ad approssimare ∀k ∈ N , il rettangoloide (1.25) con il rettangolo Rk ad esso circoscritto. Ora poniamo: def

∀k ∈ N , τk =

essendo ηk ∈ [mk , Mk ]. Il poligono:



(x, y) ∈ R2 | xk ≤ x ≤ xk+1 , 0 ≤ y ≤ ηk ,

τ (D) =

n−1 [

τk

k=0

è un plurirettangolo che non è inscritto ad R e al tempo stesso non è circoscritto ad R. def

σD = µ (τ (D)) =⇒ sD ≤ σD ≤ SD

Il numero reale σD è comunque un valore approssimato di µ (R). Inoltre: (mk ≤ ηk ≤ Mk )

(∃ξk ∈ [xk , xk+1 ] | f (ξk ) = ηk ) =⇒ f (x) è continua in [xk , xk+1 ]

Quindi:

µ (τk ) = f (ξk ) (xk+1 − xk ) =⇒ σD =

n−1 X k=0

n−1 X µ (τk ) = f (ξk ) (xk+1 − xk )

(1.26)

k=0

Dalla (1.26) segue che σD dipende da ξk per ogni k ∈ N . ∀ξk ∈ [xk , xk+1 ] (con k ∈ N ), sD ≤ σD ≤ SD =⇒ |σD − µ (R)| ≤ SD − sD Dalla dimostrazione dell’ultimo teorema segue Teorema 4. ∀ε > 0, ∃δε > 0 | ∀D (δ < δε ) , |σD − µ (R)| < ε

(1.27)

1.3 Area del rettangoloide

18

La (1.27) può essere scritta nela forma simbolica: limσD = µ (R)

δ→0

(1.28)

Si osservi che la (1.28) non è l’usuale operazione di passaggio al limite, giacché σD non una funzione ad un sol valore di δ. Infatti, assegnato un numero reale positivo δ < b − a, esistono infinite partizioni di ampiezza δ, e per ciascuna partizione esistono infiniti valori di σD , giacché questi ultimi dipendono dai punti ξk (che possono essere scelti in infiniti modi). Da ciò si conclude che σD è una funzione ad infiniti valori di δ. Pertanto, la (1.28) andrebbe riscritta nella forma: σD −→ µ (R) , δ→0

e cioè le somme σD tendono all’area del rettangoloide, quando la loro ampiezza tende a zero. *** Sia f (x) continua in [a, b] e ivi non positiva. In tal caso il rettangoloide di base [a, b], relativo a f (x), si ridefinisce: def

R =

 (x, y) ∈ R2 | a ≤ x ≤ b, f (x) ≤ y ≤ 0

Indichiamo con R′ il rettangoloide di base [a, b], relativo a −f (x). È facile convincersi che R′ è simmetrico a R rispetto all’asse x. Per i teoremi precedenti si ha che R′ è misurabile: ′ µ (R′ ) = limσD , δ→0

(1.29)

essendo:

′ σD

=

n−1 X

[−f (ξk )] (xk+1 − xk )

k=0 n−1 X

=−

k=0

(1.30)

f (ξk ) (xk+1 − xk )

= −σD , per una generica partizione D di ampiezza δ, e per ogni ξk ∈ [xk , xk+1 ]. Dalle (1.29)-(1.30) segue µ (R′ ) = −limσD δ→0

Dalla misurabilità di R e dalla simmetria tra R′ e R, segue che R è misurabile: ′

1.4 Definizione di integrale definito

19

µ (R) = µ (R′ ) , donde: limσD = −µ (R)

δ→0

1.4

(1.31)

Definizione di integrale definito

Sia f (x) continua in [a, b]. Eseguiamo ad arbitrio una partizione D (x0 , x1 , ..., xn ) di ampiezza δ dell’intervallo [a, b]. Per ogni ξk ∈ [xk , xk+1 ], consideriamo le somme: σD =

n−1 X k=0

Posto

f (ξk ) (xk+1 − xk )

λ = limσD , δ→0

(1.32)

risulta: λ=



µ (R) , se f (x) ≥ 0 , −µ (R) , se f (x) ≤ 0

(1.33)

essendo R il rettangoloide di base [a, b] relativo a f (x). Teorema 5. ∃!λ ∈ R | limσD = λ δ→0

(1.34)

Dimostrazione. Poniamo: f + (x) =

f (x) − |f (x)| f (x) + |f (x)| − , f (x) = 2 2

(1.35)

Le funzioni continue f ± (x) sono rispettivamente la parte non negativa (+) e la parte non positiva (−) di f (x). Evidentemente: f (x) = f + (x) + f − (x) Se Γ+ )y = f + (x): x ∈ [a, b] : f (x) > 0 =⇒ f + (x) = f (x) x ∈ [a, b] : f (x) < 0 =⇒ f + (x) = 0

(1.36)

1.4 Definizione di integrale definito

20

Cioè Γ+ è composto dalle parti del grafico Γ)y = f (x) di ordinata positiva, e dai punti dell’asse x le cui ordinate sono negative. Similmente Γ− )y = f − (x) x ∈ [a, b] : f (x) < 0 =⇒ f − (x) = f (x) x ∈ [a, b] : f (x) > 0 =⇒ f + (x) = 0 ∀D, ∀ξk σD = ± σD =

n−1 X

k=0 n−1 X k=0

f (ξk ) (xk+1 − xk )

f ± (ξk ) (xk+1 − xk )

Dalle (1.36): + − σD = σD + σD

Inoltre: + − limσD = µ (R+ ) , limσD = −µ (R− ) ,

δ→0

δ→0

essendo R± il rettangoloide di base [a, b], relativo a f ± (x). Per una nota proprietà del valore assoluto: + − |σD − [µ (R+ ) − µ (R− )]| ≤ σD − µ (R+ ) + σD − µ (R− ) = ε {z } | {z } | <ε/2

<ε/2

Posto

segue:

λ = µ (R+ ) − µ (R− ) ,

(1.37)

|σD − λ| ≤ ε =⇒ limσD = λ δ→0

La (1.37) implica esistenza e unicità di λ. *** Per il teorema appena dimostrato, assegnata una funzione f (x) continua in [a, b], esiste ed è unico il limite λ dato dall’equazione (1.32). Tale numero reale si chiama integrale della funzione f (x) esteso all’intervallo [a, b]: λ=

Zb a

Cioè:

f (x) dx

(1.38)

1.4 Definizione di integrale definito

21

b

Z n−1 X lim f (ξk ) (xk+1 − xk ) = f (x) dx

δ→0

k=0

(1.39)

a

L’interpretazione geometrica dell’integrale (1.39) è: Zb

f (x) dx = µ (R) , se f (x) ≥ 0

Zb

f (x) dx = −µ (R) , se f (x) ≤ 0

Zb

f (x) dx = µ (R+ ) − µ (R− ) , altrimenti

(1.40)

a

a

a

Se f (x) è una funzione definita nell’intervallo X ed è ivi continua, per ogni a, b ∈ X con a < b, risulta definito l’oggetto: Zb

f (x) dx

(1.41)

Zb f (x) dx = − f (x) dx

(1.42)

a

Poniamo quindi per definizione: Za b

def

a

Da ciò segue che (1.41) ha senso per ogni coppia di punti x′ , x′′ ∈ X: Zx′′ f (x) dx

(1.43)

x′

Il numero reale (1.43) si chiama integrale definito della funzione f (x) esteso all’intervallo orientato di estremi x′ e x′′ . Seguono le denominazioni:

1.5 Proprietà dell’integrale definito

Z

22

segno di integrale

x′ , x′′ limiti di integrazione x′ limite inferiore x′′ limite superiore f (x) funzione integranda Osservazione. Il numero reale (1.41) dipende da a, b e da f (x), ma non dalla variabile x. Ciò si esprime dicendo che x è una variabile muta, donde possiamo scrivere: Zx′′ Zx′′ Zx′′ Zx′′ Zx′′ f (x) dx = f (y) dy = f (t) dt = f (ξ) dξ = f (η) dη = ...

x′

1.5

x′

x′

x′

(1.44)

x′

Proprietà dell’integrale definito

Sia f (x) funzione continua nell’intervallo X e ivi non negativa. Sussistono le seguenti proprietà di cui omettiamo le dimostrazioni:

Zb

∀a, b ∈ X, f (x) dx = 0 ⇐⇒ f (x) ≡ 0

a

a < b =⇒

Zb

f (x) dx ≥ 0

Zb

f (x) dx ≤ 0

a

a > b =⇒

a ′



a < b, ∀ [a , b ] ⊂ [a, b] ,

Zb′

a′

f (x) dx ≤

Zb a

Qualunque sia il segno di f (x): b b Z Z ∀a, b ∈ X, f (x) dx ≤ |f (x)| dx a

a

f (x) dx

1.6 Teorema della media

1.5.1

23

Proprietà additiva Zb

∀a, b, c ∈ X,

f (x) dx =

f (x) dx +

Zb

f (x) dx

(1.45)

c

a

a

1.5.2

Zc

Proprietà distributiva

f1 (x), f2 (x) , ..., fn (x) funzioni continue in X: (1.46)

∀c1 , c2 , ...cn ∈ R, Zb [c1 f1 (x) + c2 f2 (x) + ... + cn fn (x)] dx a

= c1

Zb

f1 (x) dx + c2

Zb

f2 (x) dx + ... + cn

fn (x) dx

a

a

a

Zb

Nel caso particolare: fk (x) ≡ 0, per ogni k ∈ {2, 3, ...n}, la (1.46) si scrive: Zb

c1 f1 (x) dx = c1

Zb

f1 (x) dx,

a

a

che in generale si riscrive: Zb

cf (x) dx = c

a

Zb

f (x) dx

a

Ciò si esprime dicendo che ogni costante moltiplicativa può essere portata fuori dal segno di integrale.

1.6

Teorema della media

Teorema della media.

f (x) è continua in [a, b] =⇒ ∃ξ ∈ [a, b] | f (ξ) =

Zb

f (x) dx

a

b−a

(1.47)

1.6 Teorema della media

24

Dimostrazione. Poniamo:

µ=

Zb

f (x) dx

a

b−a Eseguiamo quindi una partizione D (x0 , x1 , ..., xn ) di [a, b] con ampiezza δ. Presi ad arbitrio i punti ξk ∈ [xk , xk+1 ], costruiamo la somma σD : σD =

n−1 X k=0

(1.48)

f (ξk ) (xk+1 − xk )

La (1.48) verifica la doppia disuguaglianza: (1.49)

m (b − a) ≤ σD ≤ M (b − a) ,

essendo m = minf (x), M = maxf (x). Eseguendo nella (1.49) l’operazione di [a,b]

passaggio al limite per δ → 0: m (b − a) ≤

Zb

[a,b]

f (x) dx ≤ M (b − a) ⇐⇒ m ≤ µ ≤ M

a

=⇒ f (x) è continua in [a, b]

=⇒ ∃ξ ∈ [a, b] : f (ξ) = µ, da cui l’asserto. Definizione. Il numero reale µ si chiama media integrale. Interpretazione geometrica. Eseguiamo una equipartizione di [a, b], cioè una partizione Dn attraverso n punti equidistanti. Ad esempio: xk = a +

k (b − a) , k ∈ N = {0, 1, ..., n − 1} n

Abbiamo: b−a b−a =⇒ δn = max (xk+1 − xk ) = k∈N n n Determiniamo una somma σDn assumendo ξk = xk+1 : xk+1 − xk =

σDn =

n−1 X k=0

n

f (xk+1 ) (xk+1 − xk ) =

Passiamo dalle somme all’integrale:

b − aX f (xk ) n k=1

1.7 Teorema fondamentale del calcolo integrale

Zb

25

f (x) dx = lim σDn δn →0

a

= lim σDn n→+∞

= (b − a) lim

n→+∞

n X

f (xk )

k=1

n

Da cui: n X f (xk )

f (x1 ) + f (x2 ) + ... + f (xn ) n→+∞ n→+∞ n n [f (x1 ) + f (x2 ) + ... + f (xn )] /n è la media aritmetica dei valori assunti da f (x) in n punti equidistanti di [a, b], e µ si presenta come il limite di tale media per n → +∞. Da qui la denominazione di media integrale. µ = lim

1.7

k=1

= lim

Teorema fondamentale del calcolo integrale

Siamo ora in grado di risolvere il problema della ricerca della primitiva di una funzione continua f (x) in un intervallo X. Assegnato un punto x0 ∈ X, consideriamo: ∀x ∈ X, F (x) =

Zx

f (ξ) dξ

(1.50)

x0

La funzione F (x) dicesi funzione integrale della funzione f (x) di punto iniziale x0 . Proposizione (Teorema fondamentale del calcolo integrale) 6. La funzione integrale (1.50) è derivabile in X, risultando: ∀x ∈ X, F ′ (x) = f (x) , cioè F (x) è una primitiva di f (x).

1.7 Teorema fondamentale del calcolo integrale

26

Dimostrazione.

F (x + ∆x) − F (x) = ∆x

=

x+∆x Z

f (ξ) dξ −

Zx

f (ξ) dξ

x0

x0

∆x Zx0 f (ξ) dξ + f (ξ) dξ

x+∆x Z x0

x

∆x x+∆x Z f (ξ) dξ

=

x

∆x

Per il teorema della media: x+∆x Z

f (ξ) dξ

∃θ ∈ [0, 1] : f (x + θ∆x) =

x

=

∆x

F (x + ∆x) − F (x) ∆x

La continuità di f (x) implica: F (x + ∆x) − F (x) = f (x) , ∆x→0 ∆x lim

donde l’asserto. *** Per l’equazione (1.3) la famiglia delle primitive di una funzione f (x) continua nell’intervallo X, è: G (x) =

Zx

f (ξ) dξ + c,

con c ∈ (−∞, +∞)

(1.51)

x0

In particolare, nella (1.51) possiamo porre c = G (x0 ), onde: G (x) = G (x0 ) +

Zx

f (ξ) dξ

(1.52)

x0

La differenza tra la (1.51) e la (1.52) è evidente: mentre nella (1.51) la primitiva è indeterminata, in quanto definita a meno di una costante additiva, nella (1.52)

1.7 Teorema fondamentale del calcolo integrale

27

essa è univocamente determinata. Inoltre, ponendo nella (1.52) x0 = a, x = b, si ottiene: Zb

f (x) dx = G (b) − G (a)

(1.53)

a

La (1.52) è la formula fondamentale del calcolo integrale, poiché fornisce l’integrale definito della funzione f (x) tra a e b, attraverso la differenza dei valori assunti in a e in b da una qualunque primitiva. La (1.52) viene spesso scritta con la notazione simbolica: Zb a

f (x) dx = G (x)|ba

(1.54)

Capitolo 2 Integrale indefinito 2.1

Definizione di integrale indefinito

Definizione. Dicesi integrale indefinito, la totalità dele primitive di una assegnata funzione f (x) continua in un intervallo X, e si indica con il simbolo: Z f (x) dx (2.1) Una qualunque primitiva di f (x) è una determinazione dell’integrale indefinito (2.1). Se F (x) è una primitiva, in forza della (1.3) si ha: Z f (x) dx = F (x) + c, con c ∈ (−∞, +∞) (2.2)

Il numero reale c si chiama costante di integrazione. Osservazione. L’integrale definito Zb

f (x) dx,

a

è un numero reale. Viceversa, l’integrale indefinito: Z f (x) dx,

è un insieme di funzioni.

L’integrale indefinito rappresenta l’operazione inversa della derivazione. precisamente l’operazione di derivazione è definita da: Df (x) = f ′ (x) ,

Più (2.3)

2.2 Integrali indefiniti fondamentali

29

essendo D l’operatore di derivazione e f (x) una qualunque funzione derivabile. L’integrazione indefinita esegue l’operazione inversa della (2.3), giacché: Z D f (x) dx = f (x) Si osservi che il risultato dell’applicazione dell’operatore di derivazione su una qualunque funzione dotata di espressione elementare, è a sua volta una funzione dotata di espressione elementare. Ci si può chiedere se tale circostanza si verifica per l’operatore di integrazione indefinita. La risposta è negativa, nel senso che esistono funzioni le cui primitive non sono dotate di espressioni elementari.

2.2

Integrali indefiniti fondamentali

Introdotta la nozione di integrale indefinito, si pone il problema della ricerca delle funzioni primitive di una assegnata funzione dotata di espressione elementare (quando ciò è possibile, secondo quanto esposto nella sezione precedente). Il punto di partenza per la soluzione di tale problema è fornito dalla tabella degli integrali indefiniti fondamentali:

1) 2) 3) 4) 5) 6) 7) 8) 9)

Z

Z

Z

Z

Z

Z

Z

n

x dx = dx x

1 xn+1 n+1

+ C, per n 6= −1

= ln |x| + C

 arctan xa + C, a 6= 0 dx 1 + C, a 6= 0 = 2a ln x−a x2 −a2 x+a a+x 1 dx + C, a 6= 0 = ln 2 2 a −x 2a a−x √ x + x2 ± a2 + C, (a 6= 0) √ dx = ln 2 2 x ±a  √ dx = arcsin xa + C, (a > 0) a2 −x2 dx x2 +a2

=

1 a

Z

ax dx =

Z

ex dx = ex + C

ax ln a

+ C, (a > 0)

10) 11) 12) 13) 14) 15) 16) 17)

Z

sin xdx = − cos x + C

Z

dx cos2 x

Z

dx sin x

Z

cos xdx = sin x + C

Z

dx sin2 x

Z

dx cos x

Z

= tan x + C

= − cot x + C  = ln tan x2 + C  = ln tan x + π 2

4

sinh dx = cosh x + C

Z

cosh dx = sinh x + C  Z dx   = tanh x + C cosh2 x Z 18)  dx  = coth x + C sinh2 x

Una coppia di integrali notevoli è la 14)-15) che può essere riscritta nella forma:

2.2 Integrali indefiniti fondamentali

1 dx = ln − cot x + C sin x sin x Z 1 dx = ln + tan x + C cos x cos x Z

30

(2.4)

Parte II Esercizi sull’integrazione di una funzione reale di variabile reale

Capitolo 3 Integrali indefiniti 3.1

Integrali indefiniti fondamentali

Calcolare i seguenti integrali, utilizzando la tabella degli integrali indefiniti fondamentali: Z Z Z 3 x 1) x dx 7) 2 dx 13) √5xdx2 −5 Z Z Z √ dx 2) xdx 8) 3 sin xdx 14) 5+5x 2 Z Z Z 3) √dxx 9) 2 sinh xdx 15) x3dx 2 −1 Z Z √ Z √ 4) x xdx 10) x3 dx 16) √ax2a+a dx (a > 0) Z Z Z 3 5 √ √ 17) 5a2 x6 dx 11) 5) 4 3 dx 2 dx Z x Z 4−4x Z √ −3dx x/2 6) e dx 12) √x2 −1 18) 2pxdx

3.2 1. 2. 3. 4.

Soluzioni Z

Z

Z

Z

x3 dx = 41 x4 + C √

1

xdx =

dx √ x



x 2 +1 1 +1 2 1

=

x− 2 +1 − 21 +1

x xdx =

Z

+ C = 32 x3/2 + C

√ +C =2 x+C x3/2 dx = 25 x5/2 + C

3.2 Soluzioni 5. 6. 7. 8. 9. 10. 11. 12. 13. 14. 15. 16. 17. 18.

Z

Z

Z

Z

Z

Z

Z

Z

Z

Z

Z

Z

Z

Z

5 √ 4 3 dx x

33 √ x 4 = 20 √ x+C 4 3 + C = 20 x 1

1

ex/2 dx = 2e 2 x + C = 2e 2 x + C 2x dx =

1 x 2 ln 2

+C

Z 3 sin xdx = 3 sin xdx = −3 cos x + C

Z 2 sinh xdx = 2 sinh xdx = 2 cosh x + C 3 dx x

Z =3

√ 3 dx 4−4x2 √−3dx x2 −1 √ dx 5x2 −5 dx 5+5x2

3dx x2 −1 √



dx x

=

3 2

= 3 ln x + C Z

√dx 1−x

p = −3 ln x + (−1 + x2 ) + C =

√1 5

Z

√ dx x2 −1

=

√1 5

= 15 arctan x + C +C = 32 ln x−1 x+1

a dx ax2 +a

=

Z

√ 1 dx x2 +1

2 6

5a x dx == 5a √

= 23 arcsin x + C

2pxdx = 2px

2

Z

Z

√ ln x + x2 − 1 + C

√ = ln x2 + x2 + 1 + C

x6 dx = 57 a2 x7 + C

x1/2 dx =

2 3



√ 2px3/2 + C = 32 x 2px + C

Altri esercizi: 1) 2)

Z

Z

3)

dx √ nx

(nx)

1−n n

dx 4)

Z

Z

dx x2 +7 dx x2 −10

5) 6)

Z

Z

√ dx 4+x2 √ dx 8−x2

3.3 Integrali di somme di funzioni

3.2.1

2. 3. 4. 5. 6.

Soluzioni Z

1.

34

Z

Z

Z

Z

Z

3.3

dx √ nx

=

(nx) dx x2 +7

1 n x1− n n−1

1−n n

=

dx = n

√1 7

n n−1

+C = 1−n n

Z

x

1−n n

√ n

xn−1 + C

dx = n

1−n 1−n x n +1 n 1−n +1 n

+ C = n1/n x1/n =

√ n

nx

arctan √x7 + C √ x−√10 ln x+ +C 10

√ dx 8−x2

=

√ dx 4+x2

= ln x +

√ dx 8−x2

= arcsin √x8 + C

√1 10



 x2 + 4 + C

Integrali di somme di funzioni

Calcolare i seguenti integrali, utilizzando la proprietà additiva:

1) 2) 3) 4) 5) 6)

Z

7)

(sin x + cos x) dx Z √  3 1 dx x2 − √ 3 2 x Z 3x5 −2x3 +1 dx x Z 3x5 −2x3 +1 dx x3 Z  3 + sin22 x dx cos2 x Z sin 2x+2 cos x dx cos x

3.3.1 1.

Z

8) 9)

Z  Z

Z

√ 3 1−x2



2 1+x2

2x4 −3x2 +5x dx x2



dx

14) 15)

cos 2x dx sin x−cos x

Z

13)



Z

cos 2x dx cos x+sin x

Z

(2x + 1)4 dx

Z

Z

(ax + b)3 dx



2



2

2+x √ − 2−x dx dx 16) 2 sin x + 4−x4 Z Z  17) (6x2 + 8x + 3) dx e3x + x5 dx 11) Z Z   1 2 − √1−x2 + cos2 x dx 18) x (x + a) (x + b) dx 12)

10)

sin 2x sin x

Soluzioni (sin x + cos x) dx =

Z

Z sin xdx+ cos xdx = (− cos x + C1 )+(sin x + C2 )

= sin x − cos x + C Z √ Z √ Z  3 3 1 1 2 2 dx = 2. x − √ x dx − √ 3 2 3 2 dx x x   2 +1 √ √ − 2 +1 3 = x23+1 + C1 − x− 23+1 + C2 = 35 x x2 − 3 3 x + C 3

3

3.3 Integrali di somme di funzioni 3. 4. 5.

Z

Z

3x5 −2x3 +1 dx x 3x5 −2x3 +1 dx x3

Z

3 cos2 x

+

35

Z Z Z 4 2 = 3 x dx − 2 x dx +

dx x

= 53 x5 − 23 x3 + ln |x| + C

Z Z Z 2 = 3 x dx − 2 dx + x−3 dx = x3 − 2x −

2 sin2 x



Z dx = 3

Z

dx +2 cos2 x

dx sin2 x

1 2x2

+C

= 3 (tan x + C1 )+2 (− cot x + C2 )

= 3 tan x − 2 cot x + C Z Z Z sin 2x sin 2x+2 cos x dx = dx + 2 dx = 2 (− cos x + C1 ) + 2 (x + C2 ) = 6. cos x cos x

−2 (cos x − x) + C Z Z   2 3 √ − 1+x2 dx = 3 7. 1−x2 8.

Z

2x4 −3x2 +5x dx x2

√ dx 1−x2

Z −2

dx 1+x2

Z Z Z 2 = 2 x dx − 3 dx + 5

= 3 arcsin x − 2 arctan x + C

dx x

 + C1 − 3 (x + C2 ) + 5 (ln |x| + C3 ) = 32 x3 − x + 5 ln |x| + C Z 2 x−sin2 x cos 2x 9. dx = cos dx sin x−cos x sin x−cos x Z Z = − sin xdx − cos xdx = cos x − sin x + C =2 Z

10.

Z

1 3 x 3

2 sin x +

sin 2x sin x



Z Z dx = 2 sin xdx + 2 cos xdx = −2 cos x + 2 sin x

= 2 (sin x − cos x) + C Z Z Z  5 3x 3x e + x dx = e dx + 5 11.

= 31 e3x + 5 ln x + C Z  Z  2 1 − √1−x2 + cos2 x dx = −2 12. 13. 14.

Z

cos 2x dx cos x+sin x

=

Z

Z

(ax + b) dx =

=

16 5 x 5

3

cos2 x−sin2 x dx cos x+sin x

Z

dx x

= 13 e3x + C1 + 5 ln |x| + C2

√ dx 1−x2

=

Z

+

Z

dx cos2 x

cos xdx−

Z

= 2 arccos x + tan x + C sin xdx = cos x+sin x+C

(a3 x3 + 3a2 bx2 + 3b2 ax + b3 ) dx

= 41 a3 x4 + a2 bx3 + 32 b2 ax2 + b3 x + C Z Z 4 15. (2x + 1) dx = (16x4 + 32x3 + 24x2 + 8x + 1) dx + 8x4 + 8x3 + 4x2 + x + C

3.4 Integrali di una f (ξ (x)), con ξ (x) funzione lineare 16.

Z =

17. 18.

Z

Z



Z

√ 2 2 2+x √ − 2−x dx 4−x4 √ dx 2−x2



Z

=

Z

√ dx 2+x2





2+x2 dx (2−x2 )(2+x2 )



Z



= arcsin √x2 − ln x +

36



2−x2 dx (2−x2 )(2+x2 )

√  x2 + 2 + C

(6x2 + 8x + 3) dx = 2x3 + 4x2 + 3x + C x (x + a) (x + b) dx =

Z

3

x dx + (a + b)

= 41 x4 + 13 (a + b) x3 + 21 abx2

Z

2

x dx + ab

Z

xdx

Integrali di una f (ξ (x)), con ξ (x) funzione lineare

3.4

Calcolare gli integrali: Z 1) sin (ax + b) dx Z 2) (ax + b)n dx Z 3) eax+b dx Z 4) cos (ax + b) dx

3.4.1

5) 6) 7) 8)

Z

9)

dx (ax+b)n

Z

Z

Z

Z

(3 − 5x)3 dx

10)

cos (5x − 2) dx

11)

3e−2x+5 dx

Z

Z

5 3

sinh (5x) dx √ dx 1−4x2 adx a−x

Soluzioni

1. I (x) =

Z

sin (ax + b) dx; poniamo ξ = ax + b =⇒ dξ = adx,

donde:

1 I (ξ) = a

Ripristinando la variabile x:

Z

1 sin ξdξ = − cos ξ + C, a

1 I (x) = − cos (ax + b) + C a 2. I (x) =

Z

(ax + b)n dx; eseguendo il cambio di varibile (3.1): 1 I (ξ) = a

Z

ξ n dξ =

1 ξ n+1 + C, a (n + 1)

(3.1)

3.4 Integrali di una f (ξ (x)), con ξ (x) funzione lineare Ripristinando la variabile x: I (x) = 3. I (x) =

Z

(ax + b)n+1 + C, per n 6= −1 a (n + 1)

eax+b dx; eseguendo il cambio di varibile (3.1): 1 I (ξ) = a

Z

1 eξ dξ = eξ , a

Ripristinando la variabile x: 1 I (x) = eax+b + C a 4. I (x) =

Z

cos (ax + b) dx; eseguendo il cambio di varibile (3.1): 1 I (ξ) = a

Z

cos ξdξ =

1 sin ξ, a

Ripristinando la variabile x: 1 +C a sin (ax + b)

I (x) = 5. I (x) =

Z

dx ; (ax+b)n

eseguendo il cambio di varibile (3.1): 1 I (ξ) = a

Z

dξ 1 1 1−n = ξ + C, n ξ a1−n

Ripristinando la variabile x: (ax + b)1−n + C, per n 6= 1 I (x) = a (1 − n) 6. I (x) =

Z

(3 − 5x)3 dx; eseguendo il cambio di varibile (3.1): 1 I (ξ) = − 5

Z

ξ 3 dξ = −

1 ξ4 + C, 54

Ripristinando la variabile x: I (x) = −

(3 − 5x)4 +C 20

37

3.4 Integrali di una f (ξ (x)), con ξ (x) funzione lineare 7. I (x) =

Z

cos (5x − 2) dx; eseguendo il cambio di variabile del tipo (3.1): 1 I (ξ) = 5

Z

cos ξdξ =

1 sin ξ + C, 5

Ripristinando la variabile x: sin (5x − 2) +C 5

I (x) = 8. I (x) =

Z

3e−2x+5 dx; eseguendo il cambio di varibile del tipo (3.1): 3 I (ξ) = − 2

Z

3 eξ dξ = − eξ + C, 2

Ripristinando la variabile x: 3 I (x) = − e−2x+5 + C 2 9. I (x) =

Z

5 3

sinh (5x) dx; eseguendo il cambio di varibile del tipo (3.1): 1 I (ξ) = 3

Z

sinh ξdξ =

1 cosh ξ + C, 3

Ripristinando la variabile x: I (x) = 10. I (x) =

Z

√ dx ; 1−4x2

1 cosh (2x) + C 3

eseguendo il cambio di varibile: ξ = 2x,

segue:

38

1 I (ξ) = 2

Z

Ripristinando la variabile x:

dξ 1 p = arcsin ξ + C, 2 1 − ξ2

I (x) =

1 arcsin (2x) + C 2

3.5 Integrazione per introduzione sotto il segno di integrale 11. I (x) =

Z

adx ; a−x

39

eseguendo il cambio di varibile: ξ = a − x,

segue: I (ξ) = −a

Z

dξ = −a ln |ξ| + C1 , ξ

Ripristinando la variabile x: I (x) = −a ln |a − x| + C1 Poniamo: C1 = a ln |C| , da cui:

3.5

C I (x) = a ln a − x

Integrazione per introduzione sotto il segno di integrale

Esempio 1: Z

xdx √ = 1 + x4

Poniamo

Z

xdx q 1 + (x2 )2

ξ = x2 , donde: 1 xdx = dξ =⇒ 2

Z

Quindi:

Esempio 2

Z

xdx q 1 + (x2 )2

1 = 2

Z

dξ p

1 + ξ2

=

 p 1  ln ξ + 1 + ξ 2 + C 2

 1  2 √ xdx 2 √ = ln x + 1 + x + C 2 1 + x4

3.5 Integrazione per introduzione sotto il segno di integrale

In (x) = Poniamo:

Z

n

xn−1 ex dx

ξ = xn =⇒ xn−1 dx = donde: 1 In (ξ) = n

3.5.1 1) 2) 3) 4) 5) 6) 7) 8) 9)

eξ dξ =

1 n 1 ξn e + C =⇒ In (x) = ex + C n n ***

Calcolare gli integrali: Z

11)

xdx cos2 x2

Z

12)

tan xdx

Z

cot xdx

Z

x cot (x + 1) dx 15)

Z

cos ax dx sin5 ax

Z

√ sin x cos x

Z

tan

Z

Z

10)

3.5.2

Z

dξ , n

Z



13) 14)

x √dxx 2

3

sin 6x cos 6xdx

17) 18)

sin 3x dx 3+cos 3x

cos2 x−sin2 x

16)

dx

19) 20)

√ x dx x2 +1

Z



x+ln x dx x

21)

Z

2x−5 dx 3x2 −2

22)

3−2x dx 5x2 +7

23)

Z

xdx x2 −5

24)

xdx 2x2 +3

25)

Z

ax+b dx a2 x2 +b2

27)

Z

√ xdx a4 −x4 x2 dx 1+x6

28)

Z

Z

Z

Z q

Z

arcsin x dx 1−x2

arctan x2 dx 4+x2

26)

Z

Z

√ x− arctan 2x dx 1+4x2

e−(x

2 +1

) dx

Z

x7x dx

Z

5

Z

2x −1 a√ dx ax

Z

2

e1/x dx x2 √

dx x√ x

Z

(ax −bx )2 dx a x bx

Z

ex dx ex −1

Z

√ ex a − bex dx Z 1/3 x/a ex/a + 1 e dx 30) 29)

Soluzioni

1. I (x) =

Z

xdx ; cos2 x2

eseguiamo il cambio di variabile 1 ξ = x2 =⇒ xdx = dξ, 2

quindi: 1 I (ξ) = 2

Z

dξ 1 1 = tan ξ + C =⇒ I (x) = tan x2 + C 2 cos ξ 2 2

40

3.5 Integrazione per introduzione sotto il segno di integrale 2. I (x) =

Z

Z

tan xdx =

41

eseguiamo il cambio di variabile:

sin x dx; cos x

ξ = cos x =⇒ dξ = − sin xdx =⇒ Z dξ = − ln |ξ| + C =⇒ I (x) = − ln |cos x| + C =⇒ I (ξ) = − ξ 3. I (x) =

Z

cot xdx =

Z

=⇒ I (ξ) =

Z

dξ = − ln |ξ| + C =⇒ I (x) = ln |sin x| + C ξ

=

Z

cos x dx; sin x

eseguiamo il cambio di variabile: ξ = sin x =⇒ dξ = sin xdx =⇒

4. I (x) =

Z

tan



x √dxx

√ sin x dx √ √ ; cos x x

eseguiamo il cambio di variabile:

√ √ 1 ξ = cos x =⇒ dξ = − √ sin xdx, 2 x quindi:

5.

Z

2

x cot (x + 1) dx =

7. I (x) =

Z

cos(x2 +1) x sin(x2 +1) dx; eseguiamo il cambio di variabile:

  ξ = sin x2 + 1 =⇒ dξ = 2x cos x2 + 1 dx,

quindi:

6. I (x) =

Z dξ = −2 ln |ξ| + C I (ξ) = −2 ξ √ I (x) = −2 ln cos x + C

Z

Z

Z 1 1 dξ = ln |ξ| + C I (ξ) = 2 ξ 2  2 I (x) = −2 ln sin x + 1 + C

3

sin 6x cos 6xdx = cos ax dx sin5 ax

=

1 a

Z

dξ ξ

1 6

=

Z

1 a

sin3 6xd (sin 6x) =

1 24

sin4 6x

 · − 41 ξ −4 + C =⇒ I (x) = − 4a sin1 4 ax + C

3.5 Integrazione per introduzione sotto il segno di integrale Z

8. I (x) =

sin 3x dx; 3+cos 3x

42

eseguiamo il cambio di variabile:

ξ = 3 + cos 3x =⇒ dξ = −3 sin 3xdx Quindi: Z 1 1 1 dξ = − ln |ξ| + C =⇒ I (x) = − ln |3 + cos 3x| + C I (ξ) = − 3 ξ 3 3 Z Z 1 sin x cos x 2x dx = 2 √sin 9. I (x) = √ 2 dx; eseguiamo il cambio di variabile: 2 cos 2x cos x−sin x

ξ = cos 2x =⇒ dξ = −2 sin 2xdx Quindi: 1 I (ξ) = − 4 10. I (x) =

Z

Z

√ x dx; x2 +1

dξ 1p 1√ √ =− ξ + C =⇒ I (x) = − cos 2x + C 2 2 ξ eseguiamo il cambio di variabile: ξ = x2 + 1 =⇒ dξ = 2xdx

Quindi: Z

1√ 2 dξ 1p √ =− ξ + C =⇒ I (x) = x +1+C 2 2 ξ Z Z √ Z Z   √ x+ln x ln x dx 1 √ √ + x = + lnxx dx = (2 x + C1 ) + 11. I (x) = dx = x x x Z  √ ln xd (ln x) = 12 4 x + ln2 x + C 1 I (ξ) = 2

12. I (x) =

Z

2x−5 dx 3x2 −2

2I1 (x) − 5I2 (x)

=

Z

2x 3x2 −2



5 3x2 −22



dx = 2

Z

xdx − 3x2 −2

Z 5

dx 3x2 −2

Calcolo di I1 (x) Eseguiamo il cambio di variabile: ξ = 3x2 − 2 =⇒ dξ = 6xdx Quindi: 1 I1 (ξ) = 6

Z

dξ 1 1 = ln |ξ| + C1 =⇒ I1 (x) = ln 3x2 − 2 + C1 ξ 6 6

=

3.5 Integrazione per introduzione sotto il segno di integrale Calcolo di I2 (x) Eseguiamo il cambio di variabile: √ √ ξ = 3x =⇒ dξ = 3dx Quindi:

donde:

Z 1 dξ I2 (ξ) = √ √ 2 3 ξ2 − 2 √ 1 1 ξ − 2 √ + C2 = √ · √ · ln ξ + 2 3 2 2 ξ − √2 1 √ + C2 = √ ln 2 6 ξ + 2 √ 3x − √2 1 √ + C2 =⇒ I1 (x) = √ ln √ 2 6 3x + 2

√ 3x − √2 1 2 5 √ +C I (x) = ln 3x − 2 − √ ln √ 3 2 6 3x + 2 Z Z Z 3−2x 1 13. I (x) = 5x2 +7 dx = 3 5x2 +7 dx − 2 5x2x+7 dx = 3I1 (x) − 2I2 (x) Z Z 1 1 Calcolo di I1 (x) = 5x2 +7 dx = √ 2 dx √ 2 5x ( ) +( 7) Eseguiamo il cambio di variabile: √ √ ξ = 5x =⇒ dξ = 5dx Quindi: 1 I1 (ξ) = √ 5

Calcolo di I2 (x) =

1 5

Z

Z

1 1 ξ dξ √ 2 = √ · √ arctan √ + C1 5 7 7 ξ2 + 7 r ! 1 5 =⇒ I1 (x) = √ arctan x + C1 7 35

d(5x2 +7) 5x2 +7

=

3 I (x) = √ arctan 35

1 10

ln |5x2 + 7| + C2 , donde:

r

5 x 7

!



1 2 ln 5x + 7 + C 5

43

3.5 Integrazione per introduzione sotto il segno di integrale 14. I (x) = 15. I (x) = 16. I (x) =

Z

Z

Z

xdx x2 −5

=

xdx 2x2 +3

=

Z

1 2

1 4

ax+b dx a2 x2 +b2

Calcolo di I1 (x) =

d(x2 −5) x2 −5

Z

= 21 ln |x2 − 5| + C

d(2x2 +3) 2x2 +3

=a Z

Z

= 41 ln (2x2 + 3) + C

xdx a2 x2 +b2

+b

Z

dx a2 x2 +b2

= aI1 (x) + bI2 (x)

xdx a2 x2 +b2

Eseguiamo il cambio di variabile: ξ = a2 x2 + b2 =⇒ dξ = 2a2 xdx Quindi:

Calcolo di I2 (x) =

Z

1 dξ = 2 ln |ξ| + C1 ξ 2a 1 =⇒ I1 (x) = 2 ln a2 x2 + b2 + C1 2a Z 1 I1 (ξ) = 2 2a

dx a2 x2 +b2

Eseguiamo il cambio di variabile:

ξ = ax =⇒ dξ = adx Quindi: 1 I2 (ξ) = 2 a

donde:

17. I (x) =

Z

Z

  1 1 dξ ξ = · arctan + C2 2 2 ξ +b a b b a  1 arctan x + C2 =⇒ I2 (x) = ab b

a  2 2 1 1 2 I (x) = 2 ln a x + b + arctan x + C 2a a b

√ xdx . a4 −x4

Eseguiamo il cambio di variabile: ξ = x2 =⇒ dξ = 2xdx

Quindi: 1 I (ξ) = 2

Z

= arcsin



ξ a2



+C

1 =⇒ I (x) = arcsin 2



x2 a2



+C

dξ q

(a2 )2 − ξ 2

44

3.5 Integrazione per introduzione sotto il segno di integrale 18. I (x) =

Z

x2 dx 1+x6

=

Z

x2 dx . 1+(x3 )2

Eseguiamo il cambio di variabile:

ξ = x3 =⇒ dξ = 3x2 dx Quindi:

19. I (x) =

Z q

Z

dξ 1 = arctan ξ + C 2 1+ξ 3  1 =⇒ I (x) = arctan x3 + C 3

1 I (ξ) = 3

arcsin x dx. 1−x2

Eseguiamo il cambio di variabile: ξ = arcsin x =⇒ dξ = √

dx 1 − x2

Quindi: Z p 2 I (ξ) = ξdξ = ξ 3/2 + C 3 2 =⇒ I (x) = (arcsin x)3/2 + C 3 20. I (x) =

Z

arctan x2 dx. 4+x2

Eseguiamo il cambio di variabile:

ξ = arctan

x dx 1 dx 1 =⇒ dξ = = dξ 2 =⇒ 2 x 2 21+ 1 + 4x 2 2

Quindi:

Z 1 1 ξdξ = ξ 2 + C I (ξ) = 2 4 x 2 1 arctan +C =⇒ I (x) = 4 2 Z √ Z Z √ x− arctan 2x arctan 2x x 21. I (x) = dx = 1+4x2 dx − dx = I1 (x) − I2 (x) 1+4x2 1+4x2 Z Z d(1+4x2 ) 1 x dx = 81 ln (1 + 4x2 ) + C1 Calcolo di I1 (x) = 1+4x2 dx = 8 1+4x2 Z √ arctan 2x Calcolo di I2 (x) = dx 1+4x2

45

3.5 Integrazione per introduzione sotto il segno di integrale Eseguiamo il cambio di variabile: ξ = arctan 2x =⇒ dξ =

2 dx 1 dξ dx =⇒ = 1 + 4x2 1 + 4x2 2

Quindi: Z p 1 ξdξ = ξ 3/2 + C1 3 1 =⇒ I2 (x) = (arctan 2x)3/2 + C2 , 3 1 I2 (ξ) = 2

donde: I (x) = 22. I (x) =

Z

e−(x

2 +1

 1 1 ln 1 + 4x2 − (arctan 2x)3/2 + C 8 3

) dx. Eseguiamo il cambio di variabile:

 1 ξ = − x2 + 1 =⇒ dξ = −2xdx =⇒ xdx = − dξ 2

Quindi:

Z

1 eξ dξ = − ξ 2 + C 2 1 −(x2 +1) =⇒ I (x) = − e +C 2

1 I (ξ) = 2

23. I (x) = 24. I (x) =

Z

Z

x2

x7 dx = e1/x dx. x2

1 2

Z

2

7x d (x2 ) =

2

7x 2 ln 7

+C

Eseguiamo il cambio di variabile: ξ=

dx dx 1 =⇒ dξ = − 2 =⇒ 2 = −dξ x x x

Quindi: Z I (ξ) = − eξ dξ = −eξ + C 1 =⇒ I (x) = − e1/x + C 2

46

3.5 Integrazione per introduzione sotto il segno di integrale 25. I (x) =

Z



5

dx x√ . x

47

Eseguiamo il cambio di variabile: ξ=



dx dx x =⇒ dξ = √ =⇒ √ = 2dξ 2 x x

Quindi: Z 5ξ +C I (ξ) = − 5ξ dξ = 2 ln 5 √ 5 x =⇒ I (x) = 2 +C ln 5 Z Z  (ax −bx )2 ax bx dx = I1 (x) − 2 (x − C2 ), essendo: dx = − 2 + 26. I (x) = x x x x a b b a I1 (x) =

Z

ax dx + bx

Qui è: J1 (x) = Risulta:

Z

Z

bx def dx = J1 (x) + J2 (x) x a

ax dx, J2 (x) = bx

Z

bx dx ax

 Z  x a x a b J1 (x) = + K1 dx = b ln a − ln b

Scambiando a con b:

J2 (x) =

 b x a

ln b − ln a

+ K2

Quindi: ax b−x − a−x bx ax b−x − a−x bx + C1 =⇒ I (x) = − 2x + C ln a − ln b ln a − ln b Z Z Z    2x −1 a√ a2x 2x− x2 −x/2 −x/2 a dx = − a − a dx 27. I (x) = dx = x/2 x a a Z Z 3 x 2 = a dx − a−x/2 dx = I1 (x) − I2 (x) , I1 (x) =

essendo:

  Z 3 3 3 2 a2x 2 x x = + C1 a2 d I1 (x) = 3 2 3 ln a Z x x a− 2 − x2 + C2 = −2 I2 (x) = −2 a d 2 ln a

3.5 Integrazione per introduzione sotto il segno di integrale Quindi: I (x) = 28. I (x) =

Z

ex dx. ex −1

 x 2  3x a 2 + 3a− 2 + C 3 ln a

Eseguiamo il cambio di variabile: ξ = ex =⇒ ex dx = dξ

Quindi: I (ξ) =

29. I (x) =

Z

Z

dξ = ξ−1

Z

d (ξ − 1) = ln |ξ − 1| + C ξ−1 =⇒ I (x) = ln |ex − 1| + C

√ ex a − bex dx. Eseguiamo il cambio di variabile: 1 ξ = a − bex =⇒ ex dx = − dξ b

Quindi: Z p 1 1 3/2 ξdξ = − · ξ +C b 1 + 12 q 2 =⇒ I (x) = − (a − bex )3 + C 3b

1 I (ξ) = − b

30. I (x) =

Z

1/3 x/a ex/a + 1 e dx. Eseguiamo il cambio di variabile: 1 ξ = ex/a =⇒ dξ = ex/a dx a

Quindi: I (ξ) = a

Z

3a (ξ + 1)4/3 + C 4 4/3 3a x/a =⇒ I (x) = e +1 +C 4

(ξ + 1)1/3 dξ =

***

48

3.5 Integrazione per introduzione sotto il segno di integrale 3.5.2.1 1) 2)

Z

Calcolare gli integrali

ax dx 1+a2x

Z

11)

e−bx dx 1−e−2bx

12)

Z

Z

(2 sinh 5x − 3 cosh 5x) dx 21)

tanh dx Z Z √ ex 13) x n n − x2 dx 3) √1−e 2x dx Z Z 2 2 4) (cos ax + sin ax) dx 14) xe−x dx Z Z √ dx 2+3x2 5) sin (ln x) x 15) 3−2+3x dx 2 Z Z √ tan x dx 16) √dxex 6) cos2 x Z Z 1−sin x 2 7) x sin (1 − x ) dx 17) x+cos dx x Z Z x)2/3 dx 8) (cot dx 18) x ln 2 sin2 x x Z Z 3x 9) 1+sin dx 19) cos2 x√dx cos3 3x 2−tan2 x Z Z  10) b−adx 2 + 2x2x+1 2xdx 20) 2 +1 cot 3x

3.5.3

49

22) 23) 24) 25) 26) 27) 28) 29) 30)

Soluzioni

1. I (x) =

Z

ax dx . 1+a2x

Eseguiamo il cambio di variabile: ξ = ax =⇒ 1 + a2x = 1 + ξ 2 ; dξ = axdx

Quindi: Z

dξ = arctan ξ + C 1 + ξ2 =⇒ I (x) = arctan (ax ) + C

I (ξ) =

2. I (x) =

Z

e−bx dx; 1−e−2bx

Eseguiamo il cambio di variabile:

ξ = e−bx =⇒ 1 − e−2bx = 1 − ξ 2 ; dξ = −be−bxdx

Quindi: dξ 1 1 + ξ = − ln +C 1 − ξ2 2b 1 − ξ 1 1 + e−bx +C =⇒ I (x) = − ln 2b 1 − e−bx

1 I (ξ) = − b

Z

Z

asin x cos xdx

Z

tan2 axdx

Z

Z

Z

Z

x√n−1 dx dx n n x +1

√ 3

1+ln x dx x

√ tan x − 1 √dx x−1

earctan x +x ln(1+x2 )+1 dx 1+x2

Z

sin x−cos x dx sin x+cos x

Z

esin

Z

Z

x2 dx x2 −2 2

x

sin 2xdx

√5−3x dx 4−3x2

3.5 Integrazione per introduzione sotto il segno di integrale 3. I (x) =

Z

x √ e dx; 1−e2x

50

Eseguiamo il cambio di variabile: ξ = ex =⇒ ex dx = dξ

Quindi: I (ξ) =

Z

Z

dξ p = arcsin ξ + C 1 − ξ2 =⇒ I (x) = arcsin (ex ) + C Z

2



Z

cos ax + sin ax + sin 2ax dx = dx+ 4. I (x) = (cos ax + sin ax) dx = Z 1 1 1 + 2a sin 2axd (2ax) = (x + C1 ) + 2a (− cos 2ax + C2 ) = x − 2a cos 2ax + C 5. I (x) =

Z

2

2

. Eseguiamo il cambio di variabile: sin (ln x) dx x ξ = ln x =⇒ dξ =

dx x

Quindi: I (ξ) =

Z

sin ξdξ = − cos ξ + C

=⇒ I (x) = − cos (ln x) + C 6. I (x) =

Z



tan x dx. cos2 x

Eseguiamo il cambio di variabile: ξ = tan x =⇒ dξ =

dx cos2 x

Quindi: Z p 2 ξdξ = ξ 3/2 + C I (ξ) = 3 √ 2 =⇒ I (x) = tan3 x + C 3 7. I (x) =

Z

x sin (1 − x2 ) dx. Eseguiamo il cambio di variabile: 1 ξ = 1 − x2 =⇒ xdx = − dξ 2

3.5 Integrazione per introduzione sotto il segno di integrale Quindi: 1 I (ξ) = − 2

8. I (x) =

Z

Z

1 cos ξ + C 2  1 =⇒ I (x) = cos 1 − x2 + C 2

(cot x)2/3 dx. sin2 x

sin ξdξ =

Eseguiamo il cambio di variabile: ξ = cot x =⇒ dξ = −

dx sin2 x

Quindi: Z 3 I (ξ) = − ξ 2/3 dξ = − ξ 5/3 + C 5 3 =⇒ I (x) = − (cot x)5/3 + C 5 9. I (x) =

Z

1+sin 3x dx cos3 3x

=

Z

dx + cos3 3x

I1 (x) =

Z

Z

sin 3x dx cos3 3x

= I1 (x) + I2 (x)

1 dx = tan 3x + C1 3 cos 3x 3 Z sin 3x dx I2 (x) = cos3 3x

Per calcolare I2 (x) eseguiamo il cambio di variabile: ξ = cos 3x =⇒ dξ = −3 sin 3xdx Quindi: Z 1 dξ 11 I2 (ξ) = − = + C2 3 ξ2 3ξ   1 1 1 + C2 =⇒ I (x) = tan 3x + +C =⇒ I2 (x) = 3 cos 3x 3 cos 3x 10. I (x) =

Z

dx . b−a cot 3x

Eseguiamo il cambio di variabile: ξ = cot 3x =⇒

dx 1 = − dξ 2 3 sin 3x

51

3.5 Integrazione per introduzione sotto il segno di integrale

52

Quindi: Z

Z

d (b − aξ) 1 = ln |b − aξ| + C b − aξ 3a 1 =⇒ I (x) = ln |b − a cot 3x| + C 3a Z Z Z 1 3 11. I (x) = (2 sinh 5x − 3 cosh 5x) dx = 2· 5 sinh 5xd (5x)− 5 cosh 5xd (5x) 1 I (ξ) = − 3

dξ 1 = b − aξ 3a

= 25 cosh 5x − 35 sinh 5x + C = 15 (2 cosh 5x − 3 sinh 5x) + C Z Z Z x) sinh x 12. I (x) = tanh dx = cosh x dx = d(cosh dx = ln (cosh x) + C cosh x

13. In (x) =

Z

√ x n n − x2 dx. Eseguiamo il cambio di variabile:

1 ξ = n − x2 =⇒ dξ = −2xdx =⇒ xdx = − dξ 2 Quindi: 1+n n ξ n +C 2 (1 + n) q 3n (n − x2 )1+n =⇒ In (x) = − 5 Z Z 2 2 1 −x2 14. I (x) = xe dx = − 2 e−x d (−x2 ) = − 12 e−x + C

In (ξ) = −

15. I (x) =

Z

I1 (x) = √1 6

Z

√ 3− 2+3x2 dx 2+3x2 dx 2+3x2

q

= 3 x 2

Z



Z =3

dx 2+3x2

dx √ 2 √ 2 ( 2) +( 3x)



Z

=

√ dx 2+3x2

√1 3

Z

= 3I1 (x) − I2 (x)

√ d( 3x) √ 2 √ 2 ( 2) +( 3x)

+ C1 arctan Z Z dx q √ dx √ I2 (x) = √2+3x 2 = 2 2 = ( 2) +( 3x) Z √ √ √ d( 3x) 1 2 + C , √ q √1 ln 3x + = 3 2 + 3x √ 2 √ 2 2 = 3 ( 2) +( 3x) donde: r r ! √ √ 3 3 1 I (x) = arctan x − √ ln 3x + 2 + 3x2 + C 2 2 3 =

3.5 Integrazione per introduzione sotto il segno di integrale 16. I (x) = 17. I (x) = 18. I (x) =

Z

Z

Z

√dx ex

=

Z

1−sin x dx x+cos x dx . x ln2 x

− 21 x

e

=

Z

Z  1 dx = −2 e− 2 x d − 21 x = 2e−x/2 + C d(x+cos x) x+cos x

= ln |x + cos x| + C

Eseguiamo il cambio di variabile: ξ = ln x =⇒ dξ =

dx x

Quindi: Z

1 dξ =− +C ξ ξ 1 =⇒ I (x) = − +C ln x

I (ξ) =

19. I (x) =

Z

√dx . cos2 x 2−tan2 x

Eseguiamo il cambio di variabile: ξ = tan x =⇒ dξ =

dx cos2 x

Quindi:  ξ I (ξ) = p +C = arcsin √ 2 2 − ξ2   tan x +C =⇒ I (x) = arcsin √ 2 Z Z  dx x dx 2 + 2x2 +1 2x2 +1 = 2 2x2 +1 + (2xxdx 2 +1)2 = 2I1 (x) + I2 (x) Z

20. I (x) = Qui è:

Z





Z

dx I1 (x) = 2x2 + 1 Z dx = √ 2 1+ 2x √ 2 Z d 2x 1 =√ 2x2 + 1 2 √  1 2x + C1 ; = √ arctan 2 Z 1 d (2x2 + 1) 1 + C2 I2 (x) = =− 2 2 4 2x + 1 (2x2 + 1)

53

3.5 Integrazione per introduzione sotto il segno di integrale Quindi: I (x) = 21. I (x) =

Z



2 arctan

√

 2x −

1 +C 4 (2x2 + 1)

asin x cos xdx. Eseguiamo il cambio di variabile: ξ = cos x =⇒ dξ = cos xdx

Quindi: Z

aξ +C ln a asin x +C =⇒ I (x) = − ln a

I (ξ) =

22. I (x) =

Z

x√n−1 dx dx. n n x +1

aξ dξ =

Eseguiamo il cambio di variabile: ξ = xn + 1 =⇒ dξ = nxn−1 dx

Quindi: Z

1 dξ 1 1 1− n = +C 1ξ 1/n ξ n1− n 1 n 1 (xn + 1)1− n + C =⇒ In (x) = · n n −q 1 1 n n (x + 1)n−1 + C = n−1 Z Z sin2 ax 2 23. I (x) = tan axdx = cos 2 ax dx = Z Z Z dx 1−cos2 ax dx = cos2 ax − dx = a1 tan ax − x = cos2 ax

1 In (ξ) = n

24. I (x) =

Z

√ 3

1+ln x dx. x

Eseguiamo il cambio di variabile: ξ = ln x =⇒ dξ =

dx x

Quindi: I (ξ) =

Z

q 33 (1 + ξ)4 + C (1 + ξ)1/3 dξ = 4 q 33 =⇒ I (x) = (1 + ln x)4 + C 4

54

3.5 Integrazione per introduzione sotto il segno di integrale 25. I (x) =

Z

√ tan x − 1 √dx . Eseguiamo il cambio di variabile: x−1 ξ=



dx x − 1 =⇒ dξ = √ 2 x−1

Quindi:

26. I (x) =

Z

Z Z sin ξ d (− cos ξ) I (ξ) = 2 dξ = −2 = −2 ln |sin ξ| + C cos ξ cos ξ √ =⇒ I (x) = −2 ln sin x − 1 + C earctan x +x ln(1+x2 )+1 dx 1+x2

= I1 (x) + I2 (x) + I3 (x), essendo: Z

earctan x dx I1 (x) = 1 + x2 Z x ln (1 + x2 ) I2 (x) = dx 1 + x2 Z dx I3 (x) = dx 1 + x2 Calcolo di I1 (x) Eseguiamo il cambio di variabile: ξ = arctan x =⇒

dx = dξ 1 + x2

Quindi: Z I1 (ξ) = 2 eξ dξ = eξ + C1 =⇒ I1 (x) = earctan x + C1

Calcolo di I2 (x) Eseguiamo il cambio di variabile:

Quindi:

 2xdx ξ = ln 1 + x2 =⇒ dξ = 1 + x2 Z 1 1 ξdξ = ξ 2 + C2 I2 (ξ) = 2 4  1 2 =⇒ I2 (x) = ln 1 + x2 + C2 4

55

3.5 Integrazione per introduzione sotto il segno di integrale Calcolo di I3 (x) I3 (x) =

Z

dx = arctan x + C3 1 + x2

Quindi: I (x) = earctan x + 27. I (x) =

Z

Z

sin x−cos x dx sin x+cos x x2 dx x2 −2

28. I (x) = Z Z = dx + 2 29. I (x) =

Z

esin

2

=

dx x2 −2 x

Z

Z =−

d(sin x+cos x) sin x+cos x

x2 −2+2 dx x2 −2

=x+2

 1 2 ln 1 + x2 + arctan x + C 4



=

1 √ 2 2

Z

= − ln |sin x + cos x| + C

1+

2 x2 −2



dx

√  √2 ln x− +C =x+ x+ 2

Quindi: I (ξ) =

Z

eξ dξ = eξ + C

=⇒ I (x) = esin 30. I (x) =

2 2

√ √2 ln x− x+ 2

sin 2xdx. Eseguiamo il cambio di variabile: ξ = sin2 x =⇒ dξ = sin 2xdx

Z



√5−3x dx 4−3x2

2

x

+C

= 5I1 (x) − 3I2 (x), essendo: I1 (x) =

Z

1 I1 (x) = √ 3

Z

dx √ 2 Z 4 − 3x dx I2 (x) = √ 4 − 3x2

Calcolo di I1 (x) r

dx 2

√2 3

− x2

√ ! 3 1 + C1 = √ arcsin x 2 3

56

3.5 Integrazione per introduzione sotto il segno di integrale

57

Calcolo di I2 (x) Eseguiamo il cambio di variabile: 1 ξ = 4 − 3x2 =⇒ xdx = − dξ 6 Quindi: 1 I2 (ξ) = − 6

Z

dξ 1p √ =− ξ + C2 3 ξ 1√ 4 − 3x2 + C2 =⇒ I2 (x) = − 3

Da ciò segue: √ ! 3 1√ 5 4 − 3x2 + C + I (x) = √ arcsin x 2 3 3

3.5.4 1) 2) 3) 4)

Calcolare gli integrali: Z

sin

2π T



+ φ0 dt 5)

Z

dx x(4−ln2 x)

Z

e− tan x cosdx2 x

Z

arccos( x2 ) √ dx 4−x2

1. I (t) =

Z

sin

6) 7) 8)

2π T

Z

Z

Z

9)

sin x cos x √ dx 4

x2 cosh (x3 + 3) dx Z 3tanh x 10) cosh 2 dx x

2−sin x

arcsin √ x+x dx 1−x2 cos 2x dx 4+cos2 2x

Z r

Z

√ ln(x+ x2 +1) dx 1+x2

 + φ0 dt. Eseguiamo il cambio di variabile:

τ=

2π 2π T + φ0 =⇒ dτ = dt =⇒ dt = dτ T T 2π

Quindi: 2π I (τ ) = T 2. I (x) =

Z

Z

T T sin τ dτ = − cos τ + C =⇒ I (t) = − cos 2π 2π

dx . x(4−ln2 x)

Eseguiamo il cambio di variabile: ξ = ln x =⇒

dx = dξ x



 2π + φ0 + C T

3.5 Integrazione per introduzione sotto il segno di integrale Quindi: Z

3. I (x) =

Z

arccos( x2 ) √ dx. 4−x2

dξ I (ξ) = 4 − ξ2 Z dξ = 2 2 − ξ2 1 2 + ξ +C = ln 4 2 − ξ 1 2 + ln x I (x) = ln +C 4 2 − ln x

Eseguiamo il cambio di variabile:

ξ = arccos

x 2

=⇒ √

dx = −dξ 4 − x2

Quindi: Z I (ξ) = − ξdξ

4. I (x) =

Z

1 = − ξ2 + C 2  x i2 1h +C I (x) = − arccos 2 2

e− tan x cosdx2 x . Eseguiamo il cambio di variabile: ξ = − tan x =⇒ dξ = −

dx cos2 x

Quindi: Z I (ξ) = − eξ dξ

= −eξ + C I (x) = −etan x + C 5. I (x) =

Z

sin x cos x √ dx. Eseguiamo il cambio di variabile: 4 2−sin x

1 ξ = sin2 x =⇒ sin x cos xdx = dξ 2

58

3.5 Integrazione per introduzione sotto il segno di integrale Quindi: Z dξ 1 p I (ξ) = 2 2 − ξ2   1 ξ = arcsin √ +C 2 2  2  sin x 1 √ +C I (x) = arcsin 2 2 6. I (x) =

Z

arcsin √ x+x dx 1−x2

=

Poniamo: def

I1 (x) =

Z

Z

arcsin x √ dx 1−x2

+

Z

√ x dx 1−x2

arcsin x def √ dx; I2 (x) = 2 1−x

Z



x dx 1 − x2

Calcolo di I1 (x) Eseguiamo il cambio di variabile: ξ = arcsin x =⇒ √

dx = dξ 1 − x2

Quindi: Z 1 I1 (ξ) = ξdξ 2 1 = ξ 2 + C1 2

I1 (x) =

1 (arcsin x)2 + C1 2

Calcolo di I2 (x) Eseguiamo il cambio di variabile: ξ = 1 − x2 =⇒ √

dξ xdx =− 2 2 1−x

Quindi: Z 1 dξ √ I2 (ξ) = − 2 ξ p = − ξ + C2 √ I1 (x) = − 1 − x2 + C2

59

3.5 Integrazione per introduzione sotto il segno di integrale L’integrale I (x) è: I (x) = 7. I (x) =

Z

cos 2x dx 4+cos2 2x

=

Z

√ 1 (arcsin x)2 − 1 − x2 + C 2

cos 2x dx. 5−sin2 2x

Eseguiamo il cambio di variabile:

ξ = sin 2x =⇒ cos 2xdx =

dξ 2

Quindi: Z dξ 1 I (ξ) = 2 5 − ξ2 √ 5 + ξ 1 = √ ln √ +C 4 5 5 − ξ ! √ 1 5 + sin 2x I (x) = ln √ +C 2 5 − sin 2x 8. I (x) =

Z r

√ ln(x+ x2 +1) dx. 1+x2



Eseguiamo il cambio di variabile:

ξ = ln x + Quindi:



x2

 dx = dξ + 1 =⇒ √ 1 + x2 Z p I (ξ) = ξdξ

2p 3 ξ +C 3 r   √ 2 I (x) = ln3 x + x2 + 1 + C 3 =

9. I (x) =

Z

x2 cosh (x3 + 3) dx. Eseguiamo il cambio di variabile: ξ = x2 + 3 =⇒ x2 dx =

dξ 3

Quindi: Z 1 I (ξ) = cosh ξdξ 3 1 = sinh ξ + C 3  1 I (x) = sinh x3 + 3 + C 3

60

3.6 Integrazione per sostituzione 10. I (x) =

Z

3tanh x dx. cosh2 x

61

Eseguiamo il cambio di variabile: ξ = tanh x =⇒

dx = dξ cos2 x

Quindi: I (ξ) = = I (x) =

3.6

Z

3ξ dξ

3ξ +C ln 3

3tanh x +C ln 3

Integrazione per sostituzione

Assegnato l’integrale:

si esegue la sostituzione:

Z

f (x) dx

(3.2)

x = φ (ξ)

(3.3)

In tal modo l’integrale (3.2) diventa: Z f [φ (ξ)] φ′ (ξ) dξ

(3.4)

La scelta della funzione φ (ξ) deve essere tale che (3.4) è riconducibile agli integrali fondamentali. Esempio 1 Z √ I (x) = x x − 1dx Poniamo:

ξ= da cui:



x − 1,

x = ξ2 + 1 dx = 2ξdξ

3.6 Integrazione per sostituzione

62

L’integrale diventa: Z I (ξ) = 2

Ripristinando la variabile x: I (x) =

 ξ + 1 ξ 2 dξ = 2 2



1 5 1 3 ξ + ξ 5 3



+C

2 2 (x − 1)5/2 + (x − 1)3/2 + C 5 3

Esempio 2 I (x) = Poniamo:

Z



dx 5x − 2

ξ = 5x − 2

da cui:

dξ = 5dx L’integrale diventa: 1 I (ξ) = 5 Ripristinando la variabile x:

Z

I (x) =

dξ 2p √ = ξ+C 5 ξ 2√ 5x − 2 + C 5

Esempio 3 I (x) = Poniamo:

Z



xdx 1 + x4

ξ = x2 da cui: dξ = 2xdx L’integrale diventa: 1 I (ξ) = 2

Z

Ripristinando la variabile x:

p 1 2 p = ln ξ + ξ + 1 + C 2 2 1+ξ dξ

3.6 Integrazione per sostituzione

63

 1  2 √ 4 I (x) = ln x + x + 1 + C 2 ***

In molti casi è conveniente eseguire le sostituzioni trigonometriche. Precisamente, se l’integrando contiene uno dei seguenti radicali: √ si eseguono le sostituzioni:

a2 − x 2 ,



x 2 − a2 ,



x 2 + a2 ,



√ a2 − x2 , x = a sin ξ =⇒ a2 − x2 = a cos ξ √ √ a x 2 − a2 , x = =⇒ x2 − a2 = a tan ξ cos ξ √ √ a x2 + a2 , x = a tan ξ =⇒ x2 + a2 = cos ξ

Esempio 4 I (x) = Poniamo:

Z √

x2 + 1 dx x2

x = tan ξ da cui: √

x2 + 1 =

1 cos ξ

L’integrale diventa: Z

dξ sin ξ cos ξ Z sin2 ξ + cos2 ξ dξ = sin2 ξ cos ξ = I1 (ξ) + I2 (ξ) ,

I (ξ) =

2

essendo: 1 dξ I1 (ξ) = = ln + tan ξ + C1 cos ξ cos ξ Z Z d (sin ξ) 1 cos ξ =− + C2 , I1 (ξ) = 2 dξ = 2 sin ξ sin ξ sin ξ Z

3.6 Integrazione per sostituzione

64

Quindi: 1 1 I (ξ) = ln + tan ξ − +C cos ξ sin ξ

Ripristinando la variabile x e osservando che

p √ 1 = 1 + tan2 ξ = 1 + x2 cos ξ r r 1 1 1 = 1+ = 1 + 2, 2 sin ξ tan ξ x si ottiene: √1 + x2 √ 2 +C I (x) = ln x + 1 + x − x

3.6.1

Calcolare gli integrali: 1) 2) 3) 4) 5)

3.6.2

Z

Z

√dx x x2 −2

6)

dx ex +1

7)

Z

x (2x + 5) dx 11)

Z

√dx x 2x+1

Z

1−x √ dx 1+ x

Z

√ dx ex −1

Z

x (5x − 3) dx

8) 9)

Z

√xdx x+1

√cos xdx

10)

Z

2

1+sin2 x

7

10

Z

ln 2x dx ln 4x x

Soluzioni

1. I (x) =

Z

√dx . x x2 −2

Procediamo per sostituzione: ξ=

1 dx dξ =⇒ =− x x ξ

12) 13)

Z

√ex dx e +1

Z

√dx dx x 1+x2

Z

2x

sin3 x √ dx cos x

3.6 Integrazione per sostituzione

65

Quindi: Z dξ I (ξ) = − q √ 2 1− ξ 2 √  Z d ξ 2 1 q = −√ √ 2 2 1− ξ 2  √  1 = − √ arcsin ξ 2 + C 2 √ ! 1 2 I (x) = − √ arcsin +C x 2 2. I (x) =

Z

dx . ex +1

Procediamo per sostituzione: x = − ln ξ =⇒

dx dξ = − ex + 1 ξ+1

Quindi: Z I (ξ) = −

3. I (x) =

Z

7

dξ ξ+1 = − ln |ξ + 1| + C  I (x) = − ln 1 + e−x + C

x (5x2 − 3) dx. Procediamo per sostituzione: 5x2 − 3 = ξ =⇒ xdx =

dξ 10

Quindi:

4. I (x) =

Z

Z 1 I (ξ) = ξ 7 dξ 10 1 = ξ8 + C 80 8 1 I (x) = 5x2 − 3 + C 80 √xdx . x+1

Procediamo per sostituzione: ξ=



x + 1 =⇒ √

dx = 2dξ x+1

3.6 Integrazione per sostituzione

66

Quindi: Z I (ξ) = 2

 ξ 2 − 1 dξ

2 = ξ 3 − 2ξ + C 3 q √ 2 I (x) = (x + 1)3 − 2 x + 1 + C 3 5. I (x) =

Z

√cos xdx . Procediamo per sostituzione: 2 1+sin x

ξ = sin ξ =⇒ cos xdx = dξ Quindi:

6. I (x) =

Z

Z dξ I (ξ) = 2 p 1 + ξ2 p 2 = ln ξ + 1 + ξ + C p I (x) = ln sin x + 1 + sin2 x + C

x (2x + 5)10 dx. Poniamo

ξ = 2x + 5 =⇒ dξ = 2dx Quindi: Z 

 ξ 5 10 − ξ dξ 2 2 1 5 = ξ 12 − ξ 11 + C 24 44   1 1 5 I (x) = (2x + 5)12 − (2x + 5)11 + C 4 12 11 1 I (ξ) = 2

7.

Z

1−x √ dx 1+ x

8. I (x) =

Z

=

Z





(1− x)(√1+ x)

√dx . x 2x+1

1+ x

dx =

Z

dx −

Z



xdx = x −

Poniamo ξ=



2x + 1 =⇒ dξ = √

dx 2x + 1

3 2



x3 + C

3.6 Integrazione per sostituzione

67

Quindi:

9. I (x) =

Z

√ dx . ex −1

Z dξ I (ξ) = 2 2 ξ −1 ξ − 1 +C = ln ξ + 1 √ 2x + 1 − 1 +C I (x) = ln √ 2x + 1 + 1

Poniamo ξ=



ex − 1 =⇒ √

dξ dx =2 2 x ξ +1 e −1

Quindi: Z I (ξ) = 2

10. I (x) =

Z

ln 2x dx ln 4x x

=

Z

dξ +1 = arctan ξ + C √  I (x) = 2 arctan ex − 1 + C ln 2+ln x dx . ln 4+ln x x

ξ2

Poniamo

ξ = ln x =⇒

dx = dξ x

Quindi: Z

ln 2 + ξ I (ξ) = dξ ln 4 + ξ  Z  ln 2 − ln 4 dξ 1+ = ln 4 + ξ Z d (ξ + ln 4) = ξ − ln 2 ln 4 + ξ = ξ − ln 2 ln |ξ + ln 4| + C I (x) = ln x − ln 2 ln |ln x + ln 4| + C 11. I (x) =

Z

2x

√ex dx. e +1

Poniamo

ξ=



 e2x dx = 2 ξ 2 − 1 dξ ex + 1 =⇒ √ x e +1

3.6 Integrazione per sostituzione

68

Quindi: Z  ξ 2 − 1 dξ I (ξ) = 2   1 3 =2 ξ −ξ +C 3 √ 2 I (x) = (ex − 2) ex + 1 + C 3 12. I (x) =

Z

sin3 x √ dx. cos x

Poniamo sin3 x 1 − ξ2 ξ = cos x =⇒ √ dx = − √ dξ cos x ξ

Quindi: I (ξ) =

Z

 ξ 3/2 − ξ −1/2 dξ

2 = ξ 5/2 − 2ξ 1/2 + C 5  √ 1 2 I (x) = 2 cos x cos x − 1 + C 5 13. I (x) =

Z

√dx dx. x 1+x2

Poniamo ξ=

dx dξ 1 =⇒ =− x x ξ

Quindi: Z dξ I (ξ) = − p 1 + ξ2 p = − ln ξ + ξ 2 + 1 + C x +C √ I (x) = − ln 1 + x2 + 1 ***

3.6 Integrazione per sostituzione

3.6.3

69

Calcolare i seguenti integrali utilizzando le sostituzioni trigonometriche 1) 2) 3) 4)

1. I (x) =

Z

2 √x dx . 1−x2

Z

Z

Z

Z

2 √x dx 1−x2

5)

3 √x dx 2−x2 √ x2 −a2 dx x

6)

√dx x x2 −1

7) 8)

Z



Z



Z

Z

x2 +1 dx x

√dx x2 4−x2

1 − x2 dx dx x(1−x)



Poniamo x2 dx = sin2 ξdξ x = sin ξ =⇒ √ 2 1−x

Quindi: Z

I (ξ) = sin2 ξdξ Z 1 = (1 − cos 2ξ) dξ 2 Z  Z 1 1 = dξ − cos 2ξd (2ξ) 2 2 ξ 1 = − sin 2ξ 2 4 x√ 1 − x2 + C I (x) = arcsin x − 2 2. I (x) =

Z

3 √x dx . 2−x2

Poniamo ξ=



x3 dx 2 sin ξ =⇒ √ = 23/2 sin3 ξdξ 2 − x2

3.6 Integrazione per sostituzione

70

Quindi: 3/2

3. I (x) =

Z

Z

I (ξ) = 2 sin2 ξ sin ξdξ Z  3/2 cos2 ξ − 1 d (cos ξ) =2   1 3 3/2 cos ξ − cos ξ + C =2 3   q q 3/2 1 2 1 − sin2 ξ − 1 − sin2 ξ + C =2 1 − sin ξ 3 "  # r r 2 2 2 1 x x x I (x) = 23/2 1− 1− +C − 1− 3 2 2 2 √ 1 = − 4 + x2 2 − x2 + C 3



x2 −a2 dx. x

x=

Poniamo

1√ 2 a a x − a2 ; dx = =⇒ tan ξ = sin ξdξ cos ξ a cos2 ξ

Quindi: Z

4. I (x) =

Z

sin2 ξ I (ξ) = a dξ cos ξ Z 1 − cos2 ξ dξ =a cos ξ  Z Z dξ − cos ξdξ =a cos ξ   1 − sin ξ + C = a ln tan ξ + cos ξ √ 2  √ 2  x − a2 + x x − a2 I (x) = a ln +C − a x

√dx . x x2 −1

Anziché eseguire una sostituzione trigonometrica, è

conveniente porre:

dx dξ 1 =⇒ =− ξ x ξ Z dξ I (ξ) = − p = − arccos ξ 1 − ξ2 1 I (x) = − arccos + C x x=

3.6 Integrazione per sostituzione 5. I (x) =

Z



x2 +1 dx. x

71

Poniamo x = tan ξ =⇒ dx =

dξ cos2 ξ

Quindi: Z

dξ I (ξ) = sin ξ cos2 ξ Z sin2 ξ + cos2 ξ dξ = sin ξ cos2 ξ Z Z sin ξ dξ = dξ + 2 cos ξ sin ξ Z Z d (cos ξ) dξ =− + 2 cos ξ sin ξ 1 1 + ln − cot ξ + C = cos ξ sin ξ

Osservando che:

√ p 1 x2 + 1 = 1 + tan2 ξ = sin ξ x √ 1 = x2 + 1, cos ξ

si ottiene: I (x) = 6. I (x) =

Z

√dx . x2 4−x2



√ x2 + 1 − 1 x2 + 1 + ln +C x

Anziché eseguire una sostituzione trigonometrica, è

conveniente porre: 1 dx =⇒ 2 = −dξ ξ x Z ξ I (ξ) = − dξ p 4ξ 2 − 1 Z 1 d (4ξ 2 − 1) p =− 8 4ξ 2 − 1 1p 2 4ξ − 1 + C =− 4 √ 4 − x2 I (x) = − +C 4x x=

3.7

Z

2

sin xdx,

7. I (x) =

Z

Z √

cos2 xdx

72

1 − x2 dx. Poniamo   √ x = sin ξ =⇒ dx = cos ξdξ, 1 − x2 = cos ξ

Quindi: Z

I (ξ) = cos2 dξ Z cos 2ξ + 1 dξ = 2  Z  Z 1 1 cos 2ξd (2ξ) + dξ = 2 2   1 1 = sin 2ξ + ξ + C 2 2 1 x√ 1 − x2 + C I (x) = arcsin x + 2 2 8. I (x) =

Z



dx . x(1−x) 2

Poniamo 

x = sin ξ =⇒ dx = 2 sin ξ cos ξdξ, Quindi:

p

I (ξ) = 2

Z

x (1 − x) = sin ξ cos ξ





= 2ξ + C √ I (x) = 2 arcsin x + C

3.7

Z

sin2 xdx,

Z

cos2 xdx

Dagli esercizi precedenti, risulta: Z

1 1 (x − sin x cos x) + C = (2x − sin 2x) + C 2 4 Z 1 1 cos2 xdx = (x + sin x cos x) + C = (2x + sin 2x) + C 2 4 sin2 xdx =

(3.5)

3.8 Integrazione per parti

3.8

73

Integrazione per parti

Proposizione. Se f (x) e g (x) sono funzioni derivabili, sussiste la formula di integrazione per parti: Z Z ′ f (x) g (x) dx = f (x) g (x) − g (x) f ′ (x) dx, (3.6) che può essere riscritta come: Z Z f (x) dg (x) = f (x) g (x) − g (x) df (x)

(3.7)

Dimostrazione. È immediata: basta applicare l’operatore di derivazione ad ambo i membri della (3.7). Esempio 1 Z



 Z x2 x2 1 dx x ln xdx = ln xd = ln x − x2 2 2 2 x 1 x2 ln x − x2 + C = 2 4 x2 = (2 ln x − 1) + C 4 Z

Esempio 2 Z

x

Z

x

Z

e d (sin x) = e sin x − ex sin xdx Z x = e sin x − ex d (− cos x) Z x x = e sin x + e cos x − ex cos xdx,

e cos xdx =

x

R risolvendo rispetto a ex cos xdx: Z ex (sin x + cos x) + C ex cos xdx = 2

3.8 Integrazione per parti

3.8.1 1) 2) 3) 4) 5)

3.8.2

74

Calcolare gli integrali: Z

Z

Z

Z

Z

6)

ln xdx arctan xdx arcsin xdx

7) 8) 9)

x sin xdx x cos 3xdx

Z

Z

Z

Z

11)

x dx ex

12)

−x

x · 2 dx

13)

2 3x

x e dx 3 −x/3

xe dx Z 10) x sin x cos xdx

14) 15)

Z

(x2 + 5x + 6) cos 2xdx

Z

x2 ln xdx

Z

ln x dx x3

Z

Z

ln2 xdx

ln √ x dx x

Soluzioni

1. I (x) = 2. I (x) = 3. I (x) =

Z

Z

ln xdx = x ln x −

Z

xd (ln x) = x ln x −

arctan xdx = x arctan x−

Z

Z

Z

Z

dx = x (ln x − 1) + C

xdx 1+x2

= x arctan x − 12 ln (1 + x2 ) + C

√xdx 1−x2

1 2

= x arcsin x + arcsin xdx = x arcsin x− √ = x arcsin x + 1 − x2 + C Z Z Z 4. I (x) = x sin xdx = xd (− cos x) = −x cos x + cos xdx

Z

d(1−x2 ) √ 1−x2

= −x cos x + sin x + C Z Z Z  1 1 1 5. I (x) = x cos 3xdx = xd 3 sin 3x = 3 x sin 3x − 9 sin 3xd (3x)  = 31 x sin 3x + 13 cos 3x + C Z Z Z Z x −x −x −x 6. I (x) = ex dx = xe dx = xd (−e ) = −xe + e−x dx = −xe−x − Z e−x d (−x) Z Z −x −x −x = xe + e dx = −xe − e−x d (−x) = −xe−x − e−x + C = − x+1 +C ex 7. I (x) = C

Z

−x

x·2 dx =

Z

Z  −x  1 1 2 −x x·d − ln 2 = − ln 2 x·2 + ln 2 2−x dx = − 2x+1 x ln 2 +

3.8 Integrazione per parti 8. I (x) =

Z

75

x2 e3x dx. Anzichè integrare per parti, è conveniente utilizzare il

metodo dei coefficienti indeterminati. Assegnato l’integrale: Z In,m (x) = pn (x) emx dx, essendo pn (x) un polinomio di grado n: pn (x) =

n X

ak x k

k=0

risulta: In,m (x) = qn (x) emx , qui è: qn (x) =

n X

(3.8)

bk xk

k=0

Applichiamo l’operatore di derivazione ad ambo i membri della (3.8): Z D pn (x) emx dx = D [qn (x) emx ] , ottenendo: pn (x) = qn′ (x) + mqn (x)

(3.9)

Il principio di identità dei polinomi applicato alla (3.9) conduce ad un sistema di equazioni lineari che permette di ricavare i coefficienti indeterminati bk , e quindi l’integrale In,m (x). Nel caso in esame è: I2,3 (x) =

Z

 x2 e3x dx = b2 x2 + b1 x + b0 e3x

x2 = 3b2 x2 + (3b1 + 2b2 ) x + b2 + 3b0 Affinchè sia verificata l’ultima, deve essere: 3b2 = 1 3b1 + 2b2 = 0 b2 + 3b0 = 0, da cui i coeffcienti indeterminati: (b2 , b1 , b0 ) =



1 2 2 ,− , 3 9 27



,

3.8 Integrazione per parti

76

quindi l’integrale: I2,3 (x) = 9. I (x) =

Z

 e3x 9x2 − 6x + 2 27

x3 e−x/3 dx. Procedendo come nell’esercizio precedente: Z

donde:

 x x3 e−x/3 dx = b3 x2 + b2 x2 + b1 x + b0 e− 3 ,

  1 b3 x3 + b2 x2 + b1 x + b0 e−x/3 x3 e−x/3 = 3b3 x2 + 2b2 x + b1 e−x/3 − 3    b3 3 b1 b2 b2 2 = − x + 3b3 − x + 2b2 − x + b1 − 3 3 3 3 da cui: b3 + 3 = 0 b2 =0 3b3 − 3 b1 2b2 − =0 3 b0 − 3b1 = 0

Z

Z

 x3 e−x/3 dx = −3 x3 + 9x2 + 54x + 162 e−x/3 Z

x sin 2xdx 10. I (x) = x sin x cos xdx =   Z Z  1 1 1 1 cos 2x = 2 xd − 2 = 2 − 2 x cos 2x + 4 cos 2xd (2x) 1 2

= 41 (sin 2x − 2x cos 2x) + C Z 11. I (x) = (x2 + 5x + 6) cos 2xdx. Anziché procedere per un’integrazione

per parti, conviene applicare il metodo dei coefficienti indeterminati. Più in generale, consideriamo l’integrale: Z def In,α (x) = pn (x) cos (αx) dx (3.10) L’integrale (3.10) ha la forma: In,α (x) = qn (x) cos (αx) + rn (x) sin (αx) ,

(3.11)

3.8 Integrazione per parti

77

essendo: pn (x) = qn (x) =

n X

ak x k

k=1 n X

bk xk

k=1

rn (x) =

n X

c k xk

k=1

Applicando l’operatore di derivazione ad ambo i membri della (3.11) ed eseguendo le dovute semplificazioni:   1  In,α (x) = 3 (a1 + 2a2 x) α cos (αx) + (a0 + a1 x) α2 + a2 −2 + α2 x2 sin (αx) α

Nel caso in esame è:   1 (5 + 2x) cos (2x) + 2x2 + 10x + 11 sin (2x) I2,2 (x) = 4 Z Z Z  3 3 x3 1 x 2 12. I (x) = x ln xdx = ln xd 3 = 3 ln x− 3 x2 dx = x9 (3 ln x − 1)+C 13. I (x) = 14. I (x) =

Z

Z

Z  ln xdx = x ln x − 2 ln xdx = x ln2 x − 2 ln x + 2 + C 2

ln x dx x3

2

=

Z

ln xd

− 2x12



=

Z

ln xd

− 2x12



=

− 2x12

ln x +

x +C = − 2x12 ln x − 41 x12 + C = − 1+ln 4x2 Z Z Z √ √ dx √ ln x √ dx = ln xd (2 15. I (x) = x) = 2 x ln x − 2 xx x Z √ √ = 2 x ln x − 2 x−1/2 dx = 2 x (ln x − 2) + C

***

3.8.3

Calcolare gli integrali: 1) 2) 3)

Z

Z

Z

ln √ x dx x

x arctan xdx

x arcsin xdx Z √  4) ln x + 1 + x2 dx

5) 6) 7) 8)

Z

xdx sin2 x

9)

10)

Z

x cos xdx sin2 x

ex sin xdx

Z Z

3x cos xdx

Z

R

eax sin bxdx sin (ln x) dx

1 2

Z

1 dx x2 x

3.8 Integrazione per parti

3.8.4

78

Soluzioni Z

Z

Z √ √ dx √ ln xd (2 x) = 2 x ln x − 2 xx

= 1. I (x) = √ = 2 x (ln x − 2) + C Z Z  2 2. I (x) = x arctan xdx = arctan xd x2 = ln √ x dx x

x2 2

arctan x −

1 2

Poniamo:

Z

x2 dx x2 +1

Z

x2 dx J (x) = x2 + 1 Z 2 x +1−1 = dx x2 + 1 Z Z dx = dx − 2 x +1 = x − arctan x + C1 , donde:

x2 + 1 x arctan x − + C 2 2 Z Z Z  2 x2 1 x 3. I (x) = x arcsin xdx = arcsin xd 2 = 2 arcsin x − 2 I (x) =

2 √x dx 1−x2

Poniamo:

Z

x2 dx J (x) = √ 1 − x2 Z 2 x −1+1 √ = dx 1 − x2 Z Z x2 − 1 dx = √ dx + √ x2 1 − x2 Z1 − √ =− 1 − x2 dx + arcsin x Calcoliamo a parte

1 = 2



R√

1 ξ − sin 2ξ 2

1 − x2 dx:



Z √

+ C1

=

ξ=arcsin x

1−

x2 dx 1 = 2

=

x=sin ξ

Z

Z

sin2 ξdξ

(1 − cos 2ξ) dξ

 √ 1 arcsin x − x 1 − x2 + C1 , 2

3.8 Integrazione per parti

79

donde:

 √ 1 x2 arcsin x − arcsin x − x 1 − x2 + C 2 4 Z Z √ √   x 2 2 4. I (x) = ln x + 1 + x dx = x ln x + 1 + x − √1+x 2 dx Z √ √ √   d(1+x2 ) 2 2 − 2 1 + x2 + C √ = x ln x + 1 + x = x ln x + 1 + x − 2 1+x I (x) =

5. I (x) = Z cos x dx sin x

Z

xdx sin2 x

=

Z

xd (− cot x) = −x cot x +

Z

cot xdx = −x cot x +

= −x cot x + ln |sin x| + C Z Z Z x cos xdx 6. I (x) = = x cos xd (− cot x) = −x cos x cot x+ cot x (cos x − x sin x) dx sin2 x Poniamo:

J (x) =

Z

Z

cos2 x − x cos x sin x dx sin x Z Z cos2 x = dx − x cos xdx sin x   Z Z 1 − sin2 x dx − x sin x − sin xdx = sin x Z x dx − x sin x = ln tan − x sin x + C, = sin x 2 =

donde:

cot x (cos x − x sin x) dx

x I (x) = −x cos x cot x + ln tan − x sin x + C 2 x x =− + ln tan + C sin x 2 Z Z Z x x x 7. I (x) = e sin xdx = e d (− cos x) = −e cos x + cos xd (ex ) Poniamo:

J (x) = =

Z

Z

ex cos xdx

ex d (sin x) = ex sin x − I (x) ,

3.8 Integrazione per parti

80

da cui:

ex (sin x − cos x) + C 2 Z Z Z x x x 8. I (x) = 3 cos xdx = 3 d (sin x) = 3 sin x − sin xd (3x ) = 3x sin x − Z 1 3x sin xdx ln 3 I (x) =

Poniamo:

J (x) =

Z

x

Z

3x d (− cos x) Z 1 x = −3 cos x + 3x cos xdx ln 3 3 sin xdx =

Quindi I (x) = 3x sin x − ln 3 [−3x cos x + I (x) ln 3] Risolvendo rispetto a I (x): I (x) =

9. Ia,b (x) = Abbiamo

Z

3x (cos x ln 3 + sin x) +C 1 + ln3

eax sin bxdx



 1 Ia,b (x) = e d − cos bx b Z 1 a = − eax cos bx + eax cos bxdx b b Z

ax

Poniamo: Ja,b (x) =

donde:

Z

 1 sin bx e cos bxdx = e d b 1 a = eax sin bx − Ia,b (x) , b b ax

Z

ax



eax (a sin bx − b cos bx) Ia,b (x) = +C a2 + b 2

3.8 Integrazione per parti 10. I (x) =

R

essendo:

81

sin (ln x) dx = x sin (ln x) − J (x) =

Z

R

cos (ln x) dx = x sin (ln x) − J (x)

cos (ln x) dx

= x cos (ln x) + I (x) , quindi: I (x) =

3.8.5

x [x sin (ln x) − cos (ln x)] + C 2

Formule ricorrenti

In questa sezione determiniamo alcune formule ricorrenti utilizzando l’integrazioni per parti. Z dx (±) In (x) = (3.12) 2 (a ± x2 )n (+)

Calcoliamo In (x):

In(+)

essendo:

Z 1 a2 + x 2 − x 2 (x) = 2 dx a (a2 + x2 )n i 1 h (+) = 2 In−1 (x) − Jn (x) , a

Jn (x) = Osserviamo che:

Z

x2 dx (a2 + x2 )n

  d 1 1 x = − , (a2 + x2 )n dx 2 (n − 1) (a2 + x2 )n−1

quindi:

Jn (x) = − Perciò: In(+)

1 x 1 (+) In−1 (x) n−1 + 2 2 2 (n − 1) (a + x ) 2 (n − 1)

  x 1 1 2n − 3 (+) I (x) (x) = 2 + a 2 (n − 1) (a2 + x2 )n−1 2 (n − 1) n−1 (−)

Similmente si perviene a In (x):

3.8 Integrazione per parti

In(±)

82

  x 1 2n − 3 (±) 1 I (x) + (x) = 2 a 2 (n − 1) (a2 ± x2 )n−1 2 (n − 1) n−1

(3.13)

Si osservi che la (3.13) è valida per n 6= 1:

1 x arctan + C a a 1 x (−) I1 (x) = arctan h + C a a (+)

I1

(x) =

Esempi: (+) I2

(x) =

(+)

(x) =

I3

= Z = Z

dx

(1 + x2 )3/2 Z dx (1 +

x2 )5/2

= =

  x 1 x 1 + arctan +C a2 2 (a + x2 ) 2a a   1 x 3 (+) + I (x) a2 4 (a + x2 )2 4 2   x  1 x (3x2 + 5a2 ) 3 +C + arctan 8a4 a a (a + x2 )2 dx (1 + x2 )5/2 x 3 (1 + x

x2 )3/2

(1 + x2 )1/2 x 3 (1 +

2 + 3

Z

dx (1 + x2 )3/2

+ C1 =⇒

x2 )3/2

+

x 2 +C 3 (1 + x2 )1/2

Calcoliamo: In(±)

(x) =

Z

a2 ± x 2

n

dx

(x) =

Z

a2 + x 2

n

dx

(+)

Calcoliamo In (x): In(+)

Sviluppiamo l’integrando nel seguente modo: a2 + x 2

n

n−1 a2 + x 2 n−1 n−1 a2 + x 2 + x 2 a2 + x 2 ,

= a2 + x 2 = a2



(3.14)

3.8 Integrazione per parti

83

donde: (+)

In(+) (x) = a2 In−1 (x) + Jn (x) , essendo: Jn (x) = Osserviamo che: 2

x a +x

Z

 2 n−1

x 2 a2 + x 2

n−1

dx

   d 1 2 2 n = a +x dx 2n

Ciò implica che possiamo eseguire un’integrazione per parti: Jn (x) = Quindi:

n x 2 1 a + x2 − In(+) (x) 2n 2n

n 2n + 1 (+) x 2 (+) In (x) = a + x2 + a2 In−1 (x) 2n 2n (−)

Analogo risultato per In (x). In definitiva abbiamo ottenuto la seguente relazione: In(±) (x) = Esempio: (+) I3/2

(x) =

Z

n x 2na2 (±) 1 a2 ± x 2 + In−1 (x) , ∀n 6= − 2n + 1 2n + 1 2

9 + x2

3/2

dx

3/2 27 (+) x 9 + x2 + I1/2 (x) 4 Z4  1/2 (+) 9 + x2 dx I1/2 (x) = 1/2 i 1/2 9 h x = + C1 =⇒ 9 + x2 + ln 9 + 9 + x2 2 2 3/2 x 1/2 9 h 1/2 i x (+) 9 + x2 + 9 + x2 + ln 9 + 9 + x2 +C I3/2 (x) = 4 2 2 =

In maniera simile si dimostrano le seguenti:

(3.15)

(3.16)

3.8 Integrazione per parti

84

  Z 1 dx x 1 dx 2n − 3 =− 2 + (x2 − a2 )n a 2 (n − 1) (x2 − a2 )n−1 2 (n − 1) (x2 − a2 )n−1 (3.17) Z n−1 n n 2n x x 2 − a2 x 2 − a2 − dx x2 − a2 dx = 2n + 1 2n + 1

Z Z

Ricaviamo ora una formula ricorrente per l’integrale: Z In (x) = xn eax dx

Abbiamo:



 1 ax In (x) = x d e a Z 1 n ax n = x e − xn−1 eax dx a a Z

n

Cioè: n 1 In (x) = xn eax − In−1 (x) a a Ad esempio:

I3 (x) =

Z

x3 eax dx

1 3 = x3 eax − I2 (x) a a 1 2 ax 2 I2 (x) = x e − I1 (x) a a 1 2 ax 2 I1 (x) = x e − I0 (x) a a 1 ax I0 (x) = xe + C0 a da cui: I3 (x) =

 eax 3 3 2 2 a x − 3a x + 6ax − 6 +C a4

(3.18)

3.8 Integrazione per parti

3.8.6 1) 2) 3) 4) 5)

85

Calcolare i seguenti integrali applicando il metodo più opportuno Z

Z

Z

Z

7)

3

x sin xdx √

8)

x ln (2x) dx

3 −x2

xe √

e

x

9)

dx

dx

Z

(x2 − 2x + 3) ln xdx Z  1−x dx 6) x ln 1+x

3.8.7

Z

Z

Z

ln2 x dx x2

13)

ln(ln x) dx x

14)

2

x arctan 3xdx Z 10) x (arctan x)2 dx Z 11) (arcsin x)2 dx Z x 12) arcsin dx x2

15)

Z

Z

Z

√ arcsin x √ dx 1−x

x tan2 2xdx sin2 x dx ex

Soluzioni

1. I (x) =

Z

x3 sin xdx

Poniamo:

Z

3

x sin xdx = sin x

3 X

k

bk x + cos x

k=1

3 X

c k xk

k=1

Derivando primo e secondo membro, e risolvendo rispetto ai coefficienti bk , c k : Z   x3 sin xdx = −x x2 − 6 cos x + 3 x2 − 2 sin x + C 2. I (x) =

Z



Z

x ln (2x) dx = ln (2x) d √ = 32 x3/2 ln (2x) − 49 x3 + C,

2 3/2 x 3



=

2 3/2 x 3

donde:

ln (2x) −

2 3

Z

x1/2 dx

2√ 3 x [3 ln (2x) − 2] + C 9 Z Z Z Z 2 1 1 3 −x2 2 −x2 2 −x2 3. I (x) = x e dx = x xe dx = 2 x xe dx = 2 x2 e−x d (x2 ) Z = 2 ξe−ξ dξ. Calcoliamo a parte quest’integrale utilizzando il metodo dei I (x) =

ξ=x

coefficienti indeterminati: Z ξe−ξ dξ = (b0 + b1 ξ) e−ξ + C

3.8 Integrazione per parti

86

Derivando primo e secondo membro: ξe−ξ = (b0 + b1 ξ) e−ξ , da cui: (b0 , b1 ) = (−1, −1) =⇒

Z

ξe−ξ dξ = − (1 + ξ) e−ξ + C

Ripristinando la variabile x: I (x) = − 4. I (x) =

Z



e

x

 1 2 1 + x2 e−x + C 2

dx. Eseguiamo il cambio di variabile: ξ=



x =⇒ dx = 2ξdξ,

donde:   Z Z ξ ξ ξ I (ξ) = 2 ξe dξ = 2 ξe − e dξ = 2 (ξ − 1) eξ + C, ripristinando la variabile x: I (x) = 2 5. 6. 7. 8. 9. 10. 11.

Z

Z

Z

Z

Z

Z

Z



 √ x−1 e x+C

(x2 − 2x + 3) ln xdx = 13 x3 ln x − 91 x3 − (ln x) x2 + 12 x2 + 3x ln x − 3x + C x ln

1−x 1+x

ln2 x dx x2



dx =

=−−

ln(ln x) dx x

1 x

x2 −1 2

ln

1−x 1+x



+C

 ln2 x + 2 ln x + 2 + C

= ln x [ln (ln x) − 1] + C

x2 arctan 3xdx = 13 x3 arctan 3x − x (arctan x)2 dx =

x2 +1 2

1 18



 x2 − 91 ln (1 + 9x2 ) + C

(arctan x)2 − x arctan x + 21 ln (1 + x2 ) + C

√  (arcsin x)2 dx = arcsin x x arcsin x + 2 1 − x2 − 2x + C

3.9 Integrali contenenti un trinomio di secondo grado 12. 13. 14. 15.

Z

arcsin x dx x2

Z

Z

=

√ arcsin x √ dx 1−x

− x1

87

√x arcsin x + ln 1+ 1−x2 + C

√ √ √ = −2 1 − x arcsin x + 2 x + C

x tan2 2xdx = 12 x tan 2x − 12 x2 − 18 ln (1 + tan2 2x) + C

Z

sin2 x dx ex

3.9

=

1 − sin x−2 cos x 5 ex

sin x −

2 5ex

+C

Integrali contenenti un trinomio di secondo grado

Consideriamo gli integrali del tipo: Z

mx + n dx, (3.19) ax2 + bx + c essendo m, n, a, b, c ∈ R. Senza perdita di generalità, consideriamo a > 0. Poniamo: F (x) =

mx + n = A (2ax + b) + B,

(3.20)

da cui: A=

m mb , B =n− , 2a 2a

(3.21)

donde: Z

 Z Z mx + n 2ax + b m mb dx dx = dx + n − 2 2 2 ax + bx + c 2a ax + bx + c 2a ax + bx + c   m 2 mb G (x) , = ln ax + bx + c + C1 + n − 2a 2a

essendo:

G (x) =

Z

ax2

dx + bx + c

Per calcolare G (x) procediamo nel modo seguente: ax2 + bx + c = a (x + k)2 + l, da cui si ricava:

(3.22)

3.9 Integrali contenenti un trinomio di secondo grado

k=

88

b2 ∆ b , l =c− =− , 2a 4a 4a

(3.23)

essendo ∆ = b2 − 4ac,

il discriminante del trinomio. Distinguiamo i tre casi:

(3.24)

1. ∆ < 0 2. ∆ > 0 3. ∆ = 0 Iniziamo con il caso 1: 1 G (x) = l Eseguiamo il cambio di variabile: r

Z

dx 2 p a (x + k) 1+ l r

a (x + k) = ξ =⇒ dx = l

L’integrale G (ξ) diventa:

G (ξ) =

r

Ripristinando la variabile x:

1 al

Z

l dξ a

(3.25)

dξ 1 = √ arctan ξ + C2 2 1+ξ al

1 G (x) = √ arctan al Osserviamo che:

 r  a b x+ + C2 l 2a

a 4a2 =− l ∆

a·l =−

quindi: 2 arctan G (x) = √ −∆ Finalmente l’integrale (3.19):  m 2 F (x) = ln ax2 + bx + c + √ 2a −∆





∆ , 4

2ax + b √ −∆

mb n− 2a





(3.26) (3.27)

+ C2

arctan



2ax + b √ −∆



+ C (3.28)

3.9 Integrali contenenti un trinomio di secondo grado

89

Esempio 7. Calcoliamo: F (x) =

Z

x2

x+2 dx − 3x + 10

Qui è ∆ = −31, quindi applichiamo la (3.28):

 7 1 F (x) = ln x2 − 3x + 10 + √ arctan 2 31



2x − 3 √ 31



+C

Nel caso speciale m = 0, n = 1:   Z dx 2ax + b 2 F (x) = +C arctan √ =√ ax2 + bx + c −∆ −∆

(3.29)

***

Nel caso 2, la (3.22) si scrive:

giacché l < 0. Quindi

ax2 + bx + c = a (x + k)2 − |l| , Z

dx a (x + k)2 − |l| Z 1 d (x + k) = a (x + k)2 − |l| a q |l| x + k − 1 a q + C2 = p ln 2 a |l| x + k + |l| a √ 1 2ax + b − ∆ √ + C2 , = √ ln ∆ 2ax + b + ∆

G (x) =

Esempio 8.

G (x) = Qui è: ∆ = 16, quindi:

cioè:

(3.30)

Z

x2

dx − 2x − 3

1 x − 3 +C G (x) = ln 4 x + 1

m 2 1 F (x) = ln ax + bx + c + √ 2a ∆



mb n− 2a



2ax + b − √∆ √ +C ln 2ax + b + ∆

(3.31)

3.9 Integrali contenenti un trinomio di secondo grado

90

*** Caso 3 (∆ = 0): l = 0, per cui Z

dx a (x + k)2 Z 1 d (x + k) = a (x + k)2 2 =− + C2 , 2ax + b

G (x) =

cioè:    2 m mb 2 F (x) = ln ax + bx + c − n − +C 2a 2a 2ax + b

(3.32)

Esempio 9.

F (x) =

Z

= Riassumendo:

x2

3x + 2 dx − 4x + 4

 8 3 ln x2 − 4x + 4 − +C 2 x−2

     mb 2 2ax + b m 2 n− +C ln ax + bx + c + √ arctan √ ∆ < 0 =⇒ F (x) = 2a 2a −∆ −∆ √   m 2 ∆ mb 2ax + b − 1 √ +C ∆ > 0 =⇒ F (x) = n− ln ln ax + bx + c + √ 2ax + b + ∆ 2a 2a ∆    m 2 mb ∆ = 0 =⇒ F (x) = ln ax2 + bx + c − n − +C 2a 2a 2ax + b

3.9.1

Calcolare gli integrali: 1) 2) 3)

Z

x2 −1 dx x2 −x−1

4)

dx x2 +2x+5

5)

Z

dx x2 +2x

Z

6)

Z

3x−2 dx x2 −4x+5

Z

x2 dx x2 −6x+10

Z

(x−2)2 dx x2 +3x+4

7)

Z

dx 3x2 −x+1

3.9 Integrali contenenti un trinomio di secondo grado

3.9.2

91

Soluzioni

1. F (x) =

Z

2. F (x) =

Z

x2 −1

R

x



dx = x + h √ i 2x−1−√ 5 = x + 12 ln |x2 − x − x| + √15 ln 2x−1+ +C 5

3. F (x) = 4. F (x) = 5. F (x) = 6. F (x) = 7. F (x) =

Z

Z

Z

Z

Z

x2 −x−1

dx =

dx x2 +2x+5

dx x2 +2x

1+

x2 −x−1

= 21 arctan

x+1 2

x +C = 21 ln x+2

3x−2 dx x2 −4x+5

(x−2)2 dx x2 +3x+4

dx

=

x dx x2 −x−1

+C

= 23 ln (x2 − 4x + 5) + 4 arctan (x − 2) + C =x−

x2 dx x2 −6x+10

3x2 −x+1



R

7 2

h

ln (x2 + 3x + 4) −

√6 7

arctan



2x+3 √ 7

i

+C

= x + 3 ln (x2 − 6x + 10) + 8 arctan (x − 3) + C

√ 2 11 11

arctan



6x−1 √ 11



+C

*** Consideriamo integrali del tipo: H (x) = Qui vanno distinti i casi:

Z



mx + n dx, ax2 + bx + c

1. a > 0 2. a < 0 Consideriamo a > 0 Applicando l’artificio (3.20):  Z Z mb 2ax + b dx m √ √ dx + n − H (x) = 2a 2a ax2 + bx + c ax2 + bx + c   m√ 2 mb = K (x) , ax + bx + c + C1 + n − a 2a essendo:

(3.33)

3.9 Integrali contenenti un trinomio di secondo grado

Z

92

dx + bx + c Per calcolare K (x) applichiamo l’artificio (3.22): Z dx , K (x) = q 2 a (x + k) + l K (x) =



ax2

cosicchè vanno distinti i tre casi: 1. ∆ < 0 2. ∆ > 0 3. ∆ = 0 Iniziamo con il caso 1:

Z

1 K (x) = √

dx p a 2 l 1+ (x + k) l p Eseguendo il cambio di variabile x → ξ = al (x + k): Z p 1 1 dξ 2 p K (ξ) = √ = √ ln ξ + ξ + 1 + C2 2 a a 1+ξ q

Ripristinando la variabile x:

Per le (3.26):

r r a 1 a 2 K (x) = √ ln (x + k) + 1 + (x + k) + C2 l l a 2ax + b , ξ= √ −∆

donde:

s 2 2ax + b 1 (2ax + b) + 1+ K (x) = √ ln √ + C2 |∆| a −∆

Finalmente l’integrale (3.33):

H (x) =

m√ a

1 ax2 + bx + c + √ a



n−

mb 2a



s 2 2ax + b (2ax + b) + 1+ ln √ +C |∆| −∆ (3.34)

3.9 Integrali contenenti un trinomio di secondo grado

93

Ad esempio: Z

x+3 √ dx x2 + 2x + 2 √ √ 2 2 = x + 2x + 2 + 2 ln x + 1 + x + 2x + 2 + C

H (x) =

Caso 2 (∆ > 0) vale la (3.30), donde: Z 1 s K (x) = p q |l|

Eseguiamo il cambio di variabile:

ξ=

r

dx a |l|

2 (x + k) − 1

a (x + k) |l|

Z dx 1 p K (ξ) = √ a ξ2 − 1 p 1 2 = √ ln ξ + ξ − 1 + C2 a

Per le (3.26):

ξ=

2ax + b √ , ∆

donde:

Quindi:

H (x) =

s 2 1 2ax + b (2ax + b) + − 1 + C2 K (x) = √ ln √ ∆ a ∆ 

m√ 2 mb ax + bx + c + n − a 2a

Caso 3 (∆ = 0):



s 2 2ax + b 1 (2ax + b) √ ln √ + − 1 + C ∆ a ∆ (3.35)

3.9 Integrali contenenti un trinomio di secondo grado

K (x) =

Z

94

dx q a (x + k)2

1 = √ ln |x + k| + C2 a 2ax + b 1 + C2 = √ ln 2a a

Quindi:

  2ax + b m√ 2 1 mb +C √ ln H (x) = ax + bx + c + n − a 2a 2a a Consideriamo a < 0   m√ 2 mb H (x) = ax + bx + c + C1 + n − K (x) a 2a Qui è: Z dx K (x) = q − |a| (x + k)2 + l

(3.36)

con k e l dati dalla (3.23). Consideriamo solo il caso ∆ > 0, giacché per ∆ ≤ 0 l’integrando è immaginario. Z dx 1 s K (x) = √  2 q l |a| (x + k) 1− l Eseguendo il cambio di variabile

ξ= otteniamo:

r

|a| (x + k) l

1 K (ξ) = p arcsin ξ + C2 |a| Ripristinando la variabile x: r

2ax + b 4a2 2ax + b =− √ =⇒ ∆ 2a ∆  2ax + b 1 √ + C2 , =⇒ K (x) = − p arcsin ∆ |a|

ξ=

(3.37)

3.9 Integrali contenenti un trinomio di secondo grado

95

donde: m√ 2 1 H (x) = ax + bx + c − p a |a|



mb n− 2a



arcsin



2ax + b √ ∆



(3.38)

+C

Esempi

1. H (x) = 2. H (x) =

3.9.3

Z

Z



2x−3 dx −2x2 +3x+2



dx −x2 +2x+5

= arcsin



x−1 √ 6



4x−3 5



+C

+C

Calcolare gli integrali 1) 2) 3)

3.9.4

√ √ = − −2x2 + 3x + 2 − 43 2 arcsin

Z

Z

Z

√ dx x−x2

4)

√ 2x−8 dx 1−x−x2

5)

√ 3x+2 dx 1−4x−6x2

6)

Z

√ −x+2 dx 2−x−x2

Z

√ −2x+3 dx 14+5x−7x2

Z

√ 10x−4 dx 2+5x−7x2

Soluzioni

1. H (x) = 2. H (x) = 3. H (x) = 4. H (x) = 5. H (x) =

Z

Z

Z

Z

Z

√ dx x−x2

= arcsin (2x − 1) + C

√ 2x−8 dx 1−x−x2

  √ √ = −2 1 − x − x2 − 9 arcsin 2x+1 +C 5

√ 3x+2 dx 1−4x−6x2 √ −x+2 dx 2−x−x2

=

√ 10x−4 dx 2+5x−7x2

Z

=

− 12





1 − 4x −

6x2

+



2 − x − x2 + 25 arcsin

√ = − 10 2 + 5x − 7x2 − 7

−2x+3 6. H (x) = √14+5x−7x 2 dx p = 72 (14 + 5x − 7x2 ) +

16 49



6 6

arcsin

2x+1 3 √ 3 7 49

√ 14 7 arcsin 417 417 x −

Esercizio 10. Calcolare:

K (x) =

Z

dx p

x2

+ px + q



h√



i (3x + 1) + C

+C

arcsin

5 14

10 5

+C

14x−5 9



+C

3.9 Integrali contenenti un trinomio di secondo grado 2 2 Soluzione 11. x2 + px + q = x + p2 + 4q−p 4 Risulta ∆ = p2 − 4q, per cui se ∆ < 0 abbiamo: Z dx K (x) = v ( u  u 4q−p2 t 1+ √ 2 4

4q−p2

Poniamo:

x+

 p 2

2 )

 p , ξ=p x+ 2 4q − p2 2

donde: Cioè: "

p K (ξ) = ln ξ + 1 + ξ 2

# p p K (x) = ln p x + + x2 + px + q + C1 2 2 4q − p p p 2 = ln x + + x + px + q + C, 2   essendo C = ln √ 2 2 + C1 . 2

4q−p

Se ∆ > 0:

donde:

 p 2 |4q − p2 | x2 + px + q = x + , − 2 4 Z dx K (x) = v ( u 2 )  u |4q−p2 |  t √ 2 2 x + p2 − 1 4 |4q−p |

Eseguendo il cambio di variabile:

ξ=p

otteniamo: In funzione di x:

 p x+ , 2 |4q − p2 | 2

p K (ξ) = ln ξ + ξ − 1 + C1

# p p 2 K (x) = ln p x + + x + px + q + C1 2 |4q − p2 | p p = ln x + + x2 + px + q + C, 2 "

2

96

3.9 Integrali contenenti un trinomio di secondo grado essendo C = ln Se ∆ = 0: donde:



√ 2 2 |4q−p |



97

+ C1 .

 p 2 x2 + px + q = x + , 2 Z p dx K (x) = 2 = ln x + + C 2 x+ p 2

***

Consideriamo gli integrali del tipo: Z dx √ L (x) = , (mx + n) ax2 + bx + c

(3.39)

essendo m, n, a, b, c ∈ R tali che (m, n) 6= (0, 0). Gli integrali di questo tipo si riducono a quelli del tipo (3.33) eseguendo il cambio di variabile: ξ=

1 mx + n

Esempio L (x) = Esegiamo il cambio di variabile: x+1=

Z

dx √ (x + 1) x2 + 1

dξ 1 =⇒ dx = − 2 , ξ ξ

donde: Z L (ξ) = − p

dξ 2ξ 2

− 2ξ + 1 p 1 = − √ ln 2ξ − 1 + 4ξ 2 − 4ξ + 2 + C 2

Ripristinando la variabile indipendente: 1 − x + p2 (1 + x2 ) 1 L (x) = − √ ln +C 1 + x 2

3.9 Integrali contenenti un trinomio di secondo grado

3.9.5

Calcolare gli integrali 1) 2)

3.9.6

Z

Z

√dx x 1−x2

3)

√ dx x x2 +x−1

4)

Z

Z

dx √ (x−1) x2 −2 dx √ (x+1) x2 +2x

Soluzioni

1. L (x) =

Z

√dx . x 1−x2

Eseguiamo il cambio di variabile: 1 x= , ξ

donde: Z dξ L (ξ) = − p ξ2 − 1 p 2 = − ln ξ + ξ − 1 + C

Ripristinando la variabile x:

2. L (x) =

Z

√ dx . x x2 +x−1

x +C √ L (x) = ln 1 + 1 − x2

Eseguiamo il cambio di variabile: 1 x= , ξ

donde:

Applicando la (3.37):

Z L (ξ) = − p L (ξ) = arcsin



dξ −ξ 2 + ξ + 1

−2ξ + 1 √ 5



+C

Ripristinando la variabile x: L (x) = arcsin



x−2 √ x 5



+C

98

3.9 Integrali contenenti un trinomio di secondo grado 3. L (x) =

Z

dx √ . (x−1) x2 −2

99

Eseguiamo il cambio di variabile: 1 x−1= , ξ

donde:

Applicando la (3.37):

Z L (ξ) = − p L (ξ) = arcsin

dξ −ξ 2 + 2ξ + 1



−ξ + 1 √ 2



+C

Ripristinando la variabile x:  x−2 +C L (x) = arcsin √ 2 (x − 1) 

4. L (x) =

Z

dx √ . (x+1) x2 +2x

Eseguiamo il cambio di variabile: 1 x+1= , ξ

donde: Z dξ L (ξ) = − p 1 − ξ2 = − arcsin ξ + C Ripristinando la variabile x: L (x) = − arcsin



1 x+1



+C

*** Consideriamo gli integrali del tipo: M (x) = Qui vanno distinti i casi:

Z √

ax2 + bx + cdx

(3.40)

3.9 Integrali contenenti un trinomio di secondo grado

100

1. a > 0 2. a < 0 Consideriamo a > 0 Ricaviamo dal trinomio un quadrato completo applichando l’artificio (3.22): Z q M (x) = a (x + k)2 + ldx Eseguiamo il cambio di variabile:

ξ =x+k

(3.41)

Quindi: M (ξ) =



a

Z r

l ξ 2 + dξ a

(3.42)

Calcoliamo l’integrale a secondo membro della (3.42), ponendo α = l/a. Integrando per parti: Z p

ξ2

donde:

Z

ξ 2 dξ p ξ2 + α Z p (ξ 2 + α) − α p dξ = ξ ξ2 + α − ξ2 + α Z p Z p dξ 2 2 =ξ ξ +α− ξ + αdξ + α p , ξ2 + α

+ αdξ = ξ

p

ξ2

+α−

Z p ξp 2 ξ 2 + αdξ = ξ +α+ 2 ξp 2 = ξ +α+ 2

Osserviamo che:

ξ =x+k = α=− Quindi:

Z α dξ p 2 ξ2 + α p α ln ξ + ξ 2 + α + C1 2

2ax + b 2a

∆ ax2 + bx + c 2 =⇒ ξ + α = 4a2 a

3.9 Integrali contenenti un trinomio di secondo grado

101

r √ ∆ a 2ax + b 2ax + b √ 2 ax2 + bx + c ln + M (x) = ax + bx + c − + C2 2a 4a 8a2 a  √  √ 2ax + b √ √ 2ax + b √ 2 ∆ a 2 ax + bx + c − = ln a + ax + bx + c − ln a + C2 4a 8a2 2a √ Incorporando ln a nella costante di integrazione: √ ∆ a √ ln a, C = C2 + 8a2 otteniamo: √ 2ax + b √ 2 ∆ a √ 2ax + b √ 2 +C M (x) = ax + bx + c − a ax + bx + c ln + 4a 8a2 2a (3.43) Esempi Z √ √ √  2 + 2x − 3 − 2 ln 1 + x + 2 + 2x − 3 M (x) = x2 + 2x − 3dx = x+1 x x 2 +C

Z

√ M (x) = 3x2 + x + 2dx i h √ √ √ 1+6x 1 2 2 √ = 72 6 (6x + 1) 3x + x + 2 + 23 3 ln 2 3 + 3x + x + 2 + C Z √ 25x2 + 4x + 7dx M (x) =  √ √  1 = 250 5 (2 + 25x) 25x2 + 4x + 7 + 171 ln 52 + 5x + 25x2 + 4x + 7 + C Consideriamo a < 0 L’integrale diventa (3.40) p Z q β 2 − (x + k)2 dx, M (x) = |a| essendo

β2 =

l ∆ = 2 |a| 4a

da ciò segue affinchè l’integrando sia reale, deve essere ∆ > 0. Eseguendo il cambio di variabile (3.41): p Z p (3.44) M (ξ) = |a| β 2 − ξ 2 dξ Integriamo per parti:

3.9 Integrali contenenti un trinomio di secondo grado

da cui:

102

Z p Z p ξ2 2 2 2 2 dξ β − ξ dξ = ξ β − ξ + p β2 − ξ2 Z p (β 2 − ξ 2 ) − β 2 2 2 p dξ =ξ β −ξ − β2 − ξ2 Z p Z p dξ 2 p = ξ β2 − ξ2 − β 2 − ξ 2 dξ + β β2 − ξ2   Z p p ξ 2 2 2 2 2 β − ξ dξ + β arcsin , =ξ β −ξ − β   Z p 2 p ξ β ξ β 2 − ξ 2 dξ = β2 − ξ2 + arcsin + C1 2 2 β

Sostituendo nella (3.44) e ripristinando la variabile x: p   ∆ |a| 2ax + b 2ax + b √ 2 √ ax + bx + c − +C arcsin M (x) = 4a 8a2 ∆ Esempi Z



x+1 2

−x2





−x2

1+x √ 2



+C − 2x + 1dx = − 2x + 1 + arcsin √ √ M (x) = x − x2 dx = 2x−1 x − x2 + 18 arcsin (2x − 1) + C 4 Z √ √  2 + 9 arcsin 2x+1 + C M (x) = 2 − x − x2 dx = 2x+1 2 − x − x 4 8 3 M (x) =

Z

***

Riassumendo: Z

mx + n dx ax2 + bx + c

(3.45)    







√ arctan 2ax+b + C, se ∆ < 0 −∆ √ m 2  2ax+b−√∆ ln ax + bx + c + = se ∆ > 0 ln 2ax+b+ + √1∆ n − mb + C, 2a  2a ∆   2 − 2ax+b + C, se ∆ = 0 2 + √−∆ n−

mb 2a

3.10 Coppia di integrali notevoli

Z

=



mx + n dx ax2 + bx + c

103

(3.46)         

m√ 2 ax + bx + c +  a       

2ax+b q (2ax+b)2 n− ln √−∆ + 1 + |∆| + C, se a > 0, ∆ < 0 2ax+b q 4a(ax2 +bx+c)  mb √1 + C, se a > 0, ∆ > 0 ln √∆ + n − 2a ∆ a 2ax+b  mb √1 n − 2a se a > 0, ∆ = 0 ln 2a + C, a    √ arcsin 2ax+b − √1 n − mb + C, se a < 0, ∆ > 0 2a ∆ √1 a

mb 2a



|a|

Z

dx √ (mx + n) ax2 + bx + c si riconduce alla forma (3.46) mediante la sostituzione 1 ξ= mx + n Z √

(3.47)

(3.48)

ax2 + bx + cdx

√ √ 2ax+b √ ∆ a 2 + bx + c + C, se a > 0 a ax − + 2 ln 2ax + b √ 2 8a 2a √   ax + bx + c + =  − ∆8a2|a| arcsin 2ax+b √ 4a + C, se a < 0 ∆

3.10

 

Coppia di integrali notevoli

Dai calcoli precedenti segue l’esistenza della coppia di integrali notevoli: Z √ √ x√ 2 α x2 + αdξ = x + α + ln x + x2 + α + C 2 2   Z p p x β2 x 2 2 2 2 β − x dξ = β −x + +C arcsin 2 2 β

(3.49)

3.11 Esercizi riepilogativi sugli integrali contenenti un trinomio di secondo grado 104

3.11

Esercizi riepilogativi sugli integrali contenenti un trinomio di secondo grado

3.11.1

Calcolare i seguenti integrali

1)

Z

√dx x x2 −1

11)

dx 2x2 +2x+5

12)

Z

x+1 dx x2 −4x+8

13)

dy y 2 +10y+30

14)

Z

√x+3 dx 1−x2

Z

2) 3)

Z

4) 5) 6)

Z

2x−7 dx x2 +9

Z



Z

7) 8)

Z

9)

10)

Z

√ dx 20+8x−x2

15) 16) 17)

dx 28−12x−x2

18)

√ x+3 dx 5−4x−x2

19)

2x+3 dx 9x2 −12x+8

20)

Z

√ x+2 dx 4x−x2

21) 22)

Z

√x+2 dx x2 +9 2x−3 dx 4x2 −11

23)

√ x+2 dx x2 +2x−3

24)

2−x dx 4x2 +4x−3

25)

Z

Z

Z

Z



Z



Z

Z

Z



√ √

25 − 3−

x2 dx

4x2 dx

26) 27)

− 36dx

28)

3x2 + 5dx

29)

x2

3 − 2x − x2 dx 30)

Z



4x2 − 4x + 5dx

Z

√dx x x2 −5

Z

x−1 dx 3x2 −4x+3 √

Z

x dx 27+6x−x2



5−4x dx 12x−4x2 −8

Z

2x−3 dx x2 +6x+13

Z

Z



Z



Z



Z



16 − 9x2 dx x2 − 16dx 4x2 + 9dx x2 − 2x − 3dx

3.11.1.1

Soluzioni Z 1. I (x) = x√dx . Eseguiamo il cambio di variabile: x2 −1 1 x= , ξ donde:

Z dξ I (ξ) = − p = − arcsin ξ + C 1 − ξ2

Ripristinando la variabile x:

  1 I (x) = − arcsin +C x 2.

Z

dx 2x2 +2x+5

= 13 arctan

2x+1 3



3.11 Esercizi riepilogativi sugli integrali contenenti un trinomio di secondo grado 105 3. 4. 5. 6. 7. 8. 9. 10. 11. 12. 13. 14. 15.

Z

Z

Z

Z

Z

Z

Z

Z

Z

Z

Z

Z

Z

x+1 x2 −4x+8

dx = 12 ln |x2 − 4x + 8| + 32 arctan

dy

=

y 2 +10y+30

√x+3 dx 1−x2 2x−7 dx x2 +9

√1 5

arctan



y+5 √ 5



x−2 2



+C

√ = − 1 − x2 + 3 arcsin x + C

= ln (x2 + 9) − 73 arctan



dx 20+8x−x2



dx 28−12x−x2

= arcsin = arcsin

x−4 6



x+6 8

x 3



+C

+C



+C

√ x+3 dx 5−4x−x2

√ = − 5 − 4x − x2 + arcsin

2x+3 dx 9x2 −12x+8

= 19 ln |9x2 − 12x + 8| +

√ x+2 dx 4x−x2

= 4 arcsin

1 x 2

2x−3 dx 4x2 −11

= 14 ln |4x2 − 11| −

2−x dx 4x2 +4x−3



+C

arctan

√3 4 11

√ √11 ln 2x− +C 2x+ 11

=

5 16

+C ln 2x−1 2x+3 5 16

(ln |2x − 1| − ln |2x + 3|) + C

3 7 ln |2x + 3| + 16 ln |2x − 1| + C = − 16 Z √ √ 16. 25 − x2 dx = x2 25 − x2 + 25 arcsin x5 + C 2



 −1 +C

√ √ = ln 1 + x + x2 + 2x − 3 + x2 + 2x − 3 + C

= − 18 (ln |2x + 3| + ln |2x − 1|) +

17.

3 x 2

√ √ x2 + 9 + 2 ln x + x2 + 9 + C

=

√ x+2 dx x2 +2x−3

13 18

x+2 3

 √ − 1 − 4x − x2 + C

√x+2 dx x2 +9

= − 18 ln |4x2 + 4x − 3| +

Z

+C

3 − 4x2 dx =

x 2



3 − 4x2 + 43 arcsin



2x √ 3



+C

3.11 Esercizi riepilogativi sugli integrali contenenti un trinomio di secondo grado 106 18. 19. 20. 21. 22. 23. 24. 25. 26. 27. 28. 29. 30.

Z

Z

Z

Z

Z

Z

Z

Z

Z

Z

Z

Z

Z

3.11.2

√ √ √ √

x2 − 36dx =

x 2

3x2 + 5dx =

x 2

√ √

√ x2 − 36 − 18 ln x + x2 − 36 + C

3x2 + 5 +

3 − 2x − x2 dx =

x+1 2

4x2 − 4x + 5dx =

√dx x x2 −5

=

√1 5

arcsin



2x−1 4

5 x

√ √ ln 3x + 3x2 + 5 + C

x−1 dx 3x2 −4x+3

= 61 ln |3x2 − 4x + 3| −

5−4x dx 12x−4x2 −8

√ √ √ √



5 15

x+3 2

arctan

√ = − 27 + 6x − x2 + 3 arcsin



+C

+C

= ln |x2 + 6x + 13| − 29 arctan

x dx 27+6x−x2



√ 4x2 − 4x + 5+ln 2x − 1 + 4x2 − 4x + 5 +C

2x−3 dx x2 +6x+13



1+x 2

3 − 2x − x2 + 2 arcsin



√ 

√ 5 3 6





+C

3x−2 √ 5

x−3 6





+C

+C

√ = 21 arcsin (3 − 2x) + 2 3x − x2 − 2 + C

16 − 9x2 dx =

x 2



16 − 9x2 + 38 arcsin

3 x 4



+C

√ √ x2 − 16dx = 21 x x2 − 16 − 8 ln x + x2 − 16 + C 4x2 + 9dx = 21 x

p

x2 − 2x − 3dx =

√ (4x2 + 9) + 94 ln 2x + 4x2 + 9 + C

x−1 2



√ x2 − 2x − 3 − 2 ln −1 + x + x2 − 2x − 3 + C

Calcolare i seguenti integrali 1) 2) 3) 4)

Z

Z

Z

Z

√ √ √ √

12 + 4x − x2 + 4xdx x2

x2 dx

5) 6)

− 8xdx

7)

x2 dx

8)

6x −

Z

√ dx 4−9x2

Z

xdx x4 −4x2 +3

Z Z

sin 8x dx 9+sin4 4x

cos x dx sin2 x−6 sin x+12

3.11 Esercizi riepilogativi sugli integrali contenenti un trinomio di secondo grado 107

3.11.3 1. 2. 3. 4. 5.

Z

Z

Z

Z

Z

Soluzioni √ √ √ √

12 + 4x − x2 dx = x2 + 4xdx =

x+2 2

x2 − 8xdx =

x−4 2

6x − x2 dx =

x−3 2

√ dx 4−9x2

6. I (x) =

Z

x−2 2



12 + 4x − x2 + 8 arcsin

x−2 4

√ √

√ x2 − 8x − 8 ln x − 4 + x2 − 8x + C  6x − x2 + 29 arcsin −1 + 13 x + C

= 13 arcsin 32 x + C sin 8x dx 9+sin4 4x

Z =2

sin 4x cos 4x dx. 9+sin4 4x

Eseguiamo il cambio di variabile:

donde: I (ξ) = = =

=

Z 1 ξdξ 2 9 + (ξ 2 )2 Z 1 d (ξ 2 ) 4 9 + (ξ 2 )2 Z 1 d (ξ 2 )  2 2 36 1 + ξ3  2 1 ξ +C arctan 12 3

Ripristinando la variabile x: 1 arctan I (x) = 12 7. I (x) =

+C

√ √ x2 + 4x − 2 ln 2 + x + x2 + 4x + C

ξ = sin 4x,

Z



xdx . x4 −4x2 +3



sin2 4x 3



+C

Eseguiamo il cambio di variabile: ξ = x2 ,

3.12 Integrazione delle funzioni razionali

108

donde: Z 1 dξ I (ξ) = 2 2 ξ − 4ξ + 3 1 ξ − 3 +C = ln 4 ξ − 1

Ripristinando la variabile x:

8. I (x) =

Z

1 x2 − 3 I (x) = ln 2 +C 4 x − 1

cos x dx. sin2 x−6 sin x+12

Eseguiamo il cambio di variabile: ξ = sin x,

donde: Z dξ 1 I (ξ) = 2 ξ 2 − 6ξ + 12   1 ξ−3 = √ arctan √ +C 3 3 Ripristinando la variabile x:

3.12

sin x − 3 1 +C I (x) = √ arctan √ 3 3

Integrazione delle funzioni razionali

Consideriamo l’integrale indefinito: Z

(3.50)

f (x) dx,

essendo:

p (x) , q (x) con p (x), q (x) polinomi in x di grado m e n rispettivamente: f (x) =

p (x) =

m X k=0

k

ak x , q (x) =

n X

bk xk

(3.51)

(3.52)

k=0

Definizione. La funzione f (x) espressa dalla (3.51) è una funzione razionale. Più precisamente, è una funzione razionale propria se m < n; una funzione razionale impropria se m ≥ n.

3.12 Integrazione delle funzioni razionali

3.12.1

109

Funzioni razionali proprie

Sotto ragionevoli ipotesi, la funzione (3.51) ammette una decomposizione in frazioni parziali, cioè somme di fattori del tipo: Ai Ax + B Ai x + Bi Ai,j , , νj , 2 x − ξi (x − ξi ) ax + bx + c (ax2 + bx + c)i

Ad esempio, se q (x) ha r radici reali distinte: ξ1 , ξ2 , ..., ξr Se νk la molteciplità di ξk : r X

νk = n,

k=1

donde:

q (x) =

r Y

k=1

(x − ξk )νk

(3.53)

In tal caso la decomposizione in frazioni parziali è f (x) =

νi r X X

Ai,j j, (x − ξ ) i i=1 j=1

(3.54)

essendo Ai,j coefficienti indeterminati. Esplicitando il secondo membro della (3.54): A1,2 A1,1 A1,ν1 + 2 + ... + x − ξ1 (x − ξ1 ) (x − ξ1 )ν1 A2,1 A2,2 A2,ν2 + + 2 + ... + x − ξ2 (x − ξ2 ) (x − ξ1 )ν2 Ar,1 Ar,2 Ar,νr ... + + 2 + ... + x − ξr (x − ξr ) (x − ξr )νr

f (x) =

(3.55)

Nel caso speciale r = n (cioè q (x) ammette n radici reali e distinte ciascuna di molteplicità 1), la decomposizione (3.54) è: n X Ai A1 A2 An f (x) = = + + ... + x − ξi x − ξ1 x − ξ2 x − ξn i=1

La (3.56) esprime una decomposizione in fattori lineari distinti.

(3.56)

3.12 Integrazione delle funzioni razionali

110

Esempio 12. 1 1 = −4 (x − 2) (x + 2) A1 A1 + = x−2 x−2 (A1 + A2 ) x + 2 (A1 − A2 ) = x2 − 4

f (x) =

x2

Per il principio di identità dei polinomi:

(3.57)

A1 + A2 = 0 2 (A1 − A2 ) = 1 La soluzione del sistema (3.57) è: (A1 , A2 ) =



1 1 ,− 4 4



,

donde la demposizione in fattori lineari distinti è: f (x) =

1 1 − 4 (x − 2) 4 (x + 2)

Nel caso generale r ∈ {1, 2, ...n − 1}, si applica la (3.54) che esprime una decomposizione in fattori lineari ripetuti. Esempio 13. p (x) = x, q (x) = x3 + x2 − x − 1

Cerchiamo le radici reali di q (x):

q (x) = 0 ⇐⇒ x = 1, −1 Precisamente: ξ1 = 1, (ν1 = 1) ξ2 = −1, (ν2 = 2) , per cui: La decomposizione si scrive:

q (x) = (x − 1) (x + 1)2

A1,1 x A2,1 A2,2 + + 2 = x − 1 x + 1 (x + 1)2 (x − 1) (x + 1)

3.12 Integrazione delle funzioni razionali

111

Per non appesantire la notazione, ridefiniamo i coefficienti della decomposizione: B1 B2 x A + + 2 = x − 1 x + 1 (x + 1)2 (x − 1) (x + 1) Abbiamo: (A + B1 ) x2 + (2A + B2 − 1) x + A − B1 − B2 = 0

da cui il sistema lineare:

A + B1 + 0 = 0 2A + 0 + B2 = 1 A − B1 − B2 = 0 che è compatibile e determinato: (A, B1 , B2 ) = Perciò:



1 1 1 ,− , 4 4 2



1 1 x 1 − + 2 = 4 (x − 1) 4 (x + 1) 2 (x + 1)2 (x − 1) (x + 1)

Quindi l’integrale: Z Esempio 14.

1 1 x − 1 x − ln +C 2 = 4 x+1 2 (x + 1) (x − 1) (x + 1) I (x) =

essendo: donde:

Z

p (x) dx, q (x)

p (x) = 1, q (x) = x3 − 2x2 + x = x (x − 1)2 ,

Ciò implica:

A B1 B2 p (x) = + + q (x) x x − 1 (x − 1)2 A + B1 + 0 = 0 2A + B1 − B2 = 0 A+0+0=1

La soluzione è x − Quindi I (x) = ln x−1

(A, B1 , B2 ) = (1, −1, 1)

1 x−1

+C

3.12 Integrazione delle funzioni razionali

112

Se q (x) contiene dei fattori irriducibili ax2 + bx + c, a ciascuno di essi corrisponde una frazione parziale propria: Ax + B , ax2 + bx + c con A, B coefficienti indeterminati. In tal caso si ha una decomposizione in fattori quadratici distinti. Esempio 15. Sia:

x3 + x2 + x + 2 f (x) = x4 + 3x2 + 2

La decomposizione è: f (x) =

Ax + B Cx + D + 2 x2 + 2 x +1

Deve essere: A+0+C +0=1 0+B+0+D =1 A + 0 + 2C + 0 = 1 0 + B + 0 + 2D = 2 Tale sistema lineare è compatibile e determinato: (A, B, C, D) = (1, 0, 0, 1) Quindi: f (x) =

x2

1 x + 2 +1 x +1

L’integrale vale: Z

f (x) dx =

 1 ln x2 + 1 + arctan x + C 2 β

Se q (x) contiene dei fattori irriducibili (ax2 + bx + c) , a ciascuno di essi corrisponde una somma di β frazioni parziali proprie: β X

Ai x + Bi , (ax2 + bx + c)i i=1

con Ai , Bi coefficienti indeterminati. In tal caso si ha una decomposizione in fattori quadratici ripetuti.

3.12 Integrazione delle funzioni razionali Esempio 16. Sia: f (x) =

113

2x2 + 3 (x2 + 1)2

Abbiamo:

Cx + D Ax + B + 2 x +1 (x2 + 1)2 Otteniamo il sistema di equazioni lineari: f (x) =

A+0+0+0=0 0+B+0+0=2 A+0+C +0=0 0 + B + 0 + D = 3, che è compatibile e determinato, con soluzione: (A, B, C, D) = (0, 2, 0, 1) donde: f (x) =

1 2 + x2 + 1 (x2 + 1)2

Ciò implica che l’integrale è: Z Z I (x) = f (x) dx = 2 arctan x +

(x2

dx + 1)2

Calcoliamo l’integrale a secondo membro attraverso il cambio di variabile: x = tan ξ, per cui:

Z

Z dx 1 = cos2 ξdξ = (ξ + sin ξ cos ξ) + C 2 2 (x2 + 1) Per ripristinare la variabile x è necessario esplicitare: sin (arctan x) , cos (arctan ξ)

A tale scopo osserviamo che: cos x = quindi:

r

1 , tan x + 1 2

1 cos (arctan ξ) = √ x2 + 1 p sin (arctan x) = 1 − cos2 (arctan x) x =√ x2 + 1

(3.58)

3.12 Integrazione delle funzioni razionali Dalle (3.58) otteniamo: sin ξ cos ξ =

x2

114

x , +1

quindi l’integrale: Z Finalmente:

Z

(x2

x dx 1 arctan x + + C1 2 = 2 2 2 (x + 1) + 1)

f (x) dx =

x 5 arctan x + +C 2 2 2 (x + 1)

Esempio 17. Sia: f (x) = Abbiamo: f (x) =

x5 − x4 + 4x3 − 4x2 + 8x − 4 (x2 + 2)3

Ex + F Ax + B Cx + D + + 2 2 x +2 (x2 + 2) (x2 + 2)3

Eseguendo i calcoli: A+0+0+0+0+0=1 0 + B + 0 + 0 + 0 + 0 = −1 4A + 0 + C + 0 + 0 + 0 = 4 0 + 4B + 0 + D + 0 + 0 = −4 4A + 0 + 2C + 0 + E + 0 = 8 0 + 4B + 0 + 2D + 0 + F = −4, la cui unica soluzione è: (A, B, C, D, E, F ) = (1, −1, 0, 0, 4, 0) Quindi: f (x) =

x−1 4x + 2 x + 2 (x2 + 2)3

L’integrale: Z Z f (x) dx =

Z Z dx xdx x dx − +4 2 2 x +2 x +2 (x2 + 2)3    1 1 x 1 2 − +C = ln x + 2 − √ arctan √ 2 2 2 2 (x + 2)2

La tabella seguente riassume i vari casi:

(3.59)

3.12 Integrazione delle funzioni razionali Fattore ax + b (ax + b)α

115

Frazioni parziali

ax2 + bx + c β

(ax2 + bx + c)

A ax+b A , i= (ax+b)i Ax+B ax2 +bx+c Ax+B , (ax2 +bx+c)i

1, 2, ..., α

N 1 α 1

i = 1, 2, ..., β

β

Se il polinomio q (x) ha radici reali e complesse multiple, per il calcolo dell’integrale (3.50) si applica la seguente formula di riduzione: Z Z Y (x) p (x) X (x) dx = + dx (3.60) q (x) q1 (x) q2 (x)

d q (x), Nella (3.60) q1 (x) è il massimo comune denominatore di q (x) e q ′ (x) = dx mentre X (x), Y (x) sono polinomi a coefficienti indeterminati di grado r − 1, essendo r il grado di q1 (x). Il polinomio q2 (x) è:

q2 (x) =

q (x) q1 (x) 2

Esempio 18. p (x) = 1, q (x) = (x3 − 1) . Abbiamo q ′ (x) = 6x2 (x3 − 1), per cui q1 (x) = q2 (x) = x3 − 1. Per la (3.60) Z Z Dx2 + Ex + F dx Ax2 + Bx + C + = x3 − 1 x3 − 1 (x3 − 1)2

Derivando primo e secondo membro:

1 (2Ax + B) (x3 − 1) − 3x2 (Ax2 + Bx + C) Dx2 + Ex + F = + x3 − 1 (x3 − 1)2 (x3 − 1)2 Dx5 + (E − A) x4 + (F − 2B) x3 + (−D − 3C) x2 + (−E − 2A) x − F − B = (x3 − 1)2

Da cui:

0+0+0+D+0+0=0 −A + 0 + 0 + 0 + E + 0 = 0 0 − 2B + 0 + 0 + 0 + F = 0 0 + 0 − 3C − D + 0 + 0 = 0 −2A + 0 + 0 + 0 − E + 0 = 0 0−B+0+0+0−F =1 Dalla prima e dalla quarta equazione del sistema (3.61): D = C = 0,

(3.61)

3.12 Integrazione delle funzioni razionali

116

donde otteniamo il sistema: (3.62)

−A + 0 + E + 0 = 0 0 − 2B + 0 + F = 0 −2A + 0 − E + 0 = 0 0−B+0−F =1 Dalla prima e dalla terza: A=E=0 Quindi:

1 2 B=− ,F =− 3 3

Riassumendo: 2 1 A = 0, B = − , C = 0, D = 0, E = 0, F = − 3 3 donde: Poniamo:

2 1 x − I (x) = − 3 3x −1 3

Z

dx −1

x3

Z

dx J (x) = 3 x −1 Z p˜ (x) def dx = q˜ (x) def

Dove:

 p˜ (x) = 1, q˜ (x) = x3 − 1 = (x − 1) x2 + x + 1

cioè q˜ (x) ha radici complesse. In tal caso la riduzione è: Z dx L Mx + N = + 2 2 (x − 1) (x + x + 1) x−1 x +x+1 Da cui: (L, M, N ) = cioè:



1 1 2 ,− ,− 3 3 3

1 1 J (x) = ln |x − 1| − 3 3

Z

x2



,

x+2 dx +x+1

L’integrale a secondo membro si calcola con la (3.45):   Z  √ 2x + 1 1 x+2 2 √ + C2 dx = ln x + x + 1 + 3 arctan x2 + x + 1 2 3

3.12 Integrazione delle funzioni razionali

117

Quindi l’integrale J (x):

√    1 1 2x + 1 3 2 √ + C1 J (x) = ln |x − 1| − ln x + x + 1 − arctan 3 6 3 3

L’integrale I (x):

√  2    x 1 x +x+1 2x + 1 2 3 √ I (x) = + ln arctan + +C 3 (1 − x3 ) 9 9 3 (x − 1)2

Z

3.12.2

dx 2 (x2 +1)

Dai calcoli precedenti segue un integrale che compare spesso:   Z 1 x dx = + arctan x + C 2 x2 + 1 (x2 + 1)2 ***

3.12.3 1) 2) 3) 4)

3.12.4

Calcolare i seguenti integrali Z

dx , (x+a)(x+b)

Z

2x2 +41x−91 dx (x−1)(x+3)(x−4)

Z

Z

(a 6= b) 5) 6)

dx (x−1)(x+2)(x+3)

7) 8)

dx x(x+1)2

Z

5x2 +6x+9 dx (x−3)2 (x+1)2

9)

10)

Z

x2 −8x+7 dx (x2 −3x−10)2 2x−3 dx (x2 −3x+2)3

11)

x5 −7x+1 dx (x2 −3x+2)3

12)

Z

Z

Z

Z

Z Z

dx (x2 −4x+3)(x2 +4x+5) dx x3 +1 dx x4 +1 dx x4 +x2 +1

Soluzioni

1. I (x) =

Z

dx . (x+a)(x+b)

Abbiamo:

1 A B = + (x + a) (x + b) x+a x+b (A + B) x + (Ab + Ba) = (x + a) (x + b)

da cui: (A, B) = Quindi:



1 1 ,− b−a b−a



x + a 1 +C I (x) = ln b − a x + b

(3.63)

3.12 Integrazione delle funzioni razionali 2. I (x) =

Z

dx . (x−1)(x+2)(x+3)

118

Abbiamo:

A B C 1 = + + (x − 1) (x + 2) (x + 3) x−1 x+2 x+3 Quindi il sistema di equazioni lineari: A+B+C =0 5A + 2B − C = 0 6A − 3B − 2C = 1, che è compatibile e determinato: (A, B, C) =



1 1 1 ,− , 12 3 4



Quindi: I (x) = 3. I (x) =

Z

1 1 1 ln |x − 1| − ln |x + 2| + ln |x + 3| + C 12 3 4

2x2 +41x−91 dx. (x−1)(x+3)(x−4)

Abbiamo:

2x2 + 41x − 91 A B C = + + (x − 1) (x + 3) (x − 4) x−1 x+2 x+3 Quindi il sistema di equazioni lineari: A+B+C =2 −A − 5B + 2C = 41 −12A + 4B − 3C = −91, che è compatibile e determinato: (A, B, C) = (4, −7, 5) Quindi: I (x) = 4 ln |x − 1| − 7 ln |x + 3| + 5 ln |x − 4| + C

3.12 Integrazione delle funzioni razionali 4. I (x) =

Z

dx . x(x+1)2

Qui è: p (x) ≡ 1, q (x) ≡ x (x + 1)

La (3.54) porge: 1 A1,1 A2,1 A2,2 = + + x (x + 1) x x + 1 (x + 1)2 B1 B2 def A + + = x x + 1 (x + 1)2 Otteniamo il sistema di equazioni lineari: A + B1 + 0 = 0 2A + B1 + B2 = 0 A + 0 + 0 = 1, la cui soluzione è: (A, B1 , B2 ) = (1, −1, −1) Quindi:

5. I (x) =

Z

x 1 +C I (x) = + ln x+1 x + 1

5x2 +6x+9 dx. (x−3)2 (x+1)2

Qui è:

p (x) ≡ 5x2 + 6x + 9, q (x) ≡ (x − 3)2 (x + 1)2 La (3.54) porge: A2,1 A1,1 A1,2 A2,2 p (x) = + + 2 + q (x) x − 3 (x − 3) x + 1 (x + 1)2 C A B D def = + + 2 + x − 3 (x − 3) x + 1 (x + 1)2 Otteniamo il sistema di equazioni lineari: A+0+C +0=0 −A + B − 5C + D = 5 −5A + 2B + 3C − 6D = 6 −3A + B + 9C + 9D = 9,

119

3.12 Integrazione delle funzioni razionali

120

Tale sistema è compatibile e determinato, con soluzione   9 1 (A, B, C, D) = 0, , 0, 2 2 Quindi: I (x) = − 6. I (x) =

Z

x2 −8x+7 dx. (x2 −3x−10)2

5x + 3 +K (x − 3) (x + 1)

Qui è:

2 p (x) ≡ x2 − 8x + 7, q (x) ≡ x2 − 3x − 10

Le radici di q (x) = 0 e le relative moltepicità sono: ξ1 = 5, (ν1 = 2) ξ2 = −2, (ν2 = 2) donde:

q (x) = (x − 5)2 (x + 2)2

La (3.54) porge: 2

ν

i Ai,j p (x) XX = q (x) (x − ξi )j i=1 j=1  2  X Ai,2 Ai,1 Ai,νi = + 2 + ... + x − ξ (x − ξi )νi (x − ξ ) i i i=1 A1,2 A2,2 A1,1 A2,1 + + = 2 + x − ξ1 (x − ξ1 ) x − ξ2 (x − ξ2 )2

Per non appesantire la notazione, ridefiniamo: def

(A1,1 , A1,2 , A2,1 , A2,2 ) = (A, B, C, D) per cui: p (x) = q (x) =

A (x − 5) (x + 2)2 + B (x + 2)2 + C (x − 5)2 (x + 2) + D (x − 5)2 (x − 5)2 (x + 2)2

3.12 Integrazione delle funzioni razionali

121

cioè: (A + C) x3 + (−A + B − 8C + D) x2 + (−16A + 4B + 5C − 10D) x + (−20A + 4B + 50C + 25D) = x2 − 8x + 7 Per il principio di identità dei polinomi deve essere: A+0+C +0=0 −A + B − 8C + D = 1 −16A + 4B + 5C − 10D = −8 −20A + 4B + 50C + 25D = 7 Tale sistema è compatibile e determinato. La soluzione è:   8 30 27 30 ,− ,− ,− (A, B, C, D) = 343 49 343 49 Quindi: p (x) 30 = q (x) 343



1 1 − x−5 x+2



  1 1 8 + − 49 (x − 5)2 (x + 2)2

L’integrale:

7. I (x) =

Z

30 x − 5 19x − 151 I (x) = ln +K − 343 x+2 49 (x2 − 3x − 10)

2x−3 dx. (x2 −3x+2)3

L’integrazione è immediata: Z

d (x2 − 3x + 2) dx (x2 − 3x + 2)3 1 =− +K 2 (x2 − 3x + 2)2

1 I (x) = 2

8. I (x) =

Z

x5 −7x+1 dx. (x2 −3x+2)3

Qui è:

3 p (x) ≡ x5 − 7x + 1, q (x) ≡ x2 − 3x + 2

Le radici di q (x) = 0 e le relative moltepicità sono: ξ1 = 2, (ν1 = 3) ξ2 = 1, (ν2 = 3)

3.12 Integrazione delle funzioni razionali donde:

122

q (x) = (x − 2)3 (x − 1)3

La (3.54) porge: ν

2

i p (x) XX Ai,j = q (x) (x − ξi )j i=1 j=1  2  X Ai,1 Ai,2 Ai,νi = + + ... + x − ξi (x − ξi )2 (x − ξi )νi i=1 A1,1 A1,3 A2,1 A2,3 A1,2 A2,2 = + + 2 + 3 + 2 + x − ξ1 (x − ξ1 ) x − ξ2 (x − ξ2 ) (x − ξ1 ) (x − ξ2 )3

Per non appesantire la notazione, ridefiniamo: def

(A1,1 , A1,2 , A1,3 , A2,1 , A2,2 , A2,3 ) = (A, B, C, D, E, F ) per cui: p (x) = q (x) 1 · {A (x − 2)2 (x − 2)3 + B (x − 2) (x − 1)3 + C (x − 1)3 q (x) + D (x − 1)2 (x − 2)3 + E (x − 1) (x − 2)3 + F (x − 2)3 } =

Da cui: 19 5 p (x) 16 25 17 26 = + 3 + 2 − 3 + 2 + q (x) x − 2 (x − 1) x−1 (x − 2) (x − 2) (x − 1) L’integrale: I (x) =

9. I (x) =

9 − 14x 38 − 33x − 25 ln |−2 + x| + 26 ln |x − 1| + K 2 + 2 x − 3x + 2 − 3x + 2)

2 (x2 Z

1 . (x2 −4x+3)(x2 +4x+5)

Qui è:

  p (x) ≡ 1, q (x) ≡ x2 − 4x + 3 x2 + 4x + 5

Le radici reali di q (x) sono:

ξ1 = 3, ξ2 = 1

3.12 Integrazione delle funzioni razionali

123

con molteplicità: ν1 = ν2 = 1 Quindi: p (x) A B Cx + D = + + 2 , q (x) x − 3 x − 1 x + 4x + 5

(3.64)

giacché x2 + 4x + 5 ha radici complesse coniugate. La (3.64) implica A+B+C +0=0 3A + B − 4C + D = 0 A − 7B + 3C − 4D = 0 −5A − 15B + +0 + 3d = 1, la cui unica soluzione è: (A, B, C, D) = onde: I (x) = essendo:



1 2 3 1 ,− , , 52 20 65 26



1 1 ln |x − 3| − ln |x − 1| + J (x) , 52 20 1 J (x) = 130

Z

4x + 15 , x2 + 4x + 5

che si calcola attraverso la (3.45), ottenendo: J (x) = Finalmente:

1 2 7 ln x + 4x + 5 + arctan (x + 2) + K1 65 130

7 1 1 1 ln |x − 3|− ln |x − 1|+ ln x2 + 4x + 5 + arctan (x + 2) 52 20 65 130 Z 10. I (x) = x3dx+1 . Qui è: I (x) =

p (x) ≡ 1, q (x) ≡ x3 + 1 Risulta: donde:

 q (x) = (x + 1) x2 − x + 1 , A Bx + C p (x) = + 2 , q (x) x+1 x −x+1

3.12 Integrazione delle funzioni razionali

124

che conduce al sistema: A+B+0=0 −A + B + C = 0 A+0+C =1 compatibile e determinato: (A, B, C) = cioè



1 1 2 ,− , 3 3 3



1 1 1 x−2 p (x) = − q (x) 3 x + 1 3 x2 − x + 1

Quindi l’integrale: I (x) =

1 1 ln |x + 1| − J (x) , 3 3

essendo: J (x) =

Z

x2

x−2 dx, −x+1

che si calcola attraverso la (3.45), ottenendo:  √ 1 J (x) = ln x2 − x + 1 − 3 arctan 2



2x − 1 √ 3



+ K1

Finalmente:  1 1 1 I (x) = ln |x + 1| − ln x2 − x + 1 + √ arctan 3 6 3 11. I (x) =

Z

dx . x4 +1



2x − 1 √ 3



+K

Qui è:

   √ √ 2 2 p (x) = 1, q (x) = x + 1 = x + 2x + 1 x − 2x + 1 4

Quindi: (Ax + B) x2 − p (x) = q (x)



√   2x + 1 + (Cx + D) x2 + 2x + 1 q (x)

3.12 Integrazione delle funzioni razionali

125

da cui il sistema: A+0+C +0=0 √ √ − 2A + B + 2C + D = 0 √ √ A − 2B + C + 2D = 0 0+B+0+D =1 compatible e determinato (A, B, C, D) =



1 1 1 1 √ , ,− √ , 2 2 2 2 2 2



Quindi l’integrale: 1 I (x) = √ [J1 (x) − J2 (x)] , 2 2 essendo: J1 (x) =

Z

√ √ Z x+ 2 x− 2 √ √ dx, J2 (x) = dx x2 + 2x + 1 x2 − 2x + 1

che si calcolano attraverso la (3.45), ottenendo:  √  1  2 √ ln x + 2x + 1 + arctan 2x + 1 + C1 2 √  1 2 √ J2 (x) = ln x − 2x + 1 − arctan 2x − 1 + C2 2

J1 (x) =

Finalmente:

# " √  √  √ 1 x2 + 2x + 1 + 2 arctan √ 2x + 1 − 2 arctan 2x − 1 I (x) = √ ln x2 − 2x + 1 4 2 (3.65) Z dx 12. I (x) = x4 +x 2 +1 . Qui è:

Quindi:

  p (x) = 1, q (x) = x4 + x2 + 1 = x2 + x + 1 x2 − x + 1 (Ax + B) (x2 − x + 1) + (Cx + D) (x2 + x + 1) p (x) = q (x) q (x)

3.12 Integrazione delle funzioni razionali

126

da cui il sistema: A+0+C +0=0 −A + B + C + D = 0 A−B+C +D =0 0+B+0+D =1 compatible e determinato (A, B, C, D) = Quindi l’integrale: I (x) = essendo: J1 (x) =

Z



1 1 1 1 , ,− , 2 2 2 2



1 [J1 (x) − J2 (x)] , 2

√ √ Z x− 2 x+ 2 √ √ dx, J2 (x) = dx x2 + 2x + 1 x2 − 2x + 1

che si calcolano attraverso la (3.45), ottenendo:    1 1 2x + 1 2 √ J1 (x) = ln x + x + 1 + √ arctan + K1 2 3 3    1 2x − 1 1 2 √ + K2 J2 (x) = ln x − x + 1 − √ arctan 2 3 3

Finalmente: " r  # 2x + 1 1 2x − 1 1 x2 + x + 1 I (x) = + arctan √ + √ arctan √ +K ln 2 x2 − x + 1 3 3 3 ***

3.12.5 1) 2) 3) 4)

Calcolare i seguenti integrali Z

x+1 dx x2 −5x+6

Z

(x+2)2 dx x3 −1

Z

Z

x2 +1 dx x3 −x2 −x+1

x3 +2x2 +1 dx x5 −x4 +2x3 −2x2 +x−1

5) 6) 7) 8)

Z

x+1 dx x2 −3x+2

9)

10)

Z

x2 dx (x+2)(x−1)2 x2 +2 dx (x−1)3

11)

x2 −1 dx (x−2)(x2 +1)

12)

Z

Z

Z

Z

Z

Z

x2 +1 dx x3 −4x2 +5x−2 2x2 +1 dx x3 −2x2 +x−2 x+1 dx x3 −1 x2 −2x dx (2x−1)(x2 +1)

3.12 Integrazione delle funzioni razionali

3.12.6

Soluzioni

1. I (x) =

Z

x+1 dx. x2 −5x+6

Qui è:

p (x) = 1, q (x) = x2 − 5x + 6 = (x − 3) (x − 2) Quindi: A B p (x) = + q (x) x−3 x−2 da cui (A, B) = (4, −3) L’integrale:

2. I (x) =

Z

I (x) = 4 ln |x − 3| − 3 ln |x − 2| + C x2 +1 dx. x3 −x2 −x+1

Qui è:

p (x) = x2 + 1, q (x) = x3 − x2 − x + 1 = (x − 1)2 (x + 1) Le radici di q (x) = 0 e le relative moltepicità sono: ξ1 = 1, (ν1 = 2) ξ2 = −1, (ν2 = 1) La (3.54) porge: 2

ν

i p (x) XX Ai,j = q (x) (x − ξi )j i=1 j=1  2  X Ai,2 Ai,1 Ai,νi + = + ... + x − ξi (x − ξi )2 (x − ξi )νi i=1 A1,2 A1,1 A2,1 + = 2 + x − ξ1 (x − ξ1 ) x − ξ2

Per non appesantire la notazione, ridefiniamo: def

(A1,1 , A1,2 , A2,1 ) = (A, B, C) per cui: C p (x) A B = + 2 + q (x) x − 1 (x − 1) x+1

127

3.12 Integrazione delle funzioni razionali

128

che conduce al sistema A+0+C =1 0 + B − 2C = 0 −A + B + C = 1, compatibile e determinato: (A, B, C) =



1 1 , 1, 2 2

L’integrale: I (x) = ln 3. I (x) =

Quindi:

Z

(x+2)2 dx. x3 −1

p |x2 − 1| −



1 +K x−1

Qui è:

 p (x) = x2 + 4x + 4, q (x) = (x − 1) x2 + x + 1

da cui

p (x) A Bx + C = + 2 q (x) x−1 x +x+1 A+B+0=1 A−B+C =4 A+0−C =4

La soluzione è (A, B, C) = (3, −2, −1) Perciò: p (x) 3 2x + 1 = − 2 q (x) x−1 x +x+1 d (x2 + x + 1) 3 − dx 2 = x−1 x +x+1 Quindi:  I (x) = 3 ln |x − 1| − ln x2 + x + 1 + K ! |x − 1|3 = ln +K x2 + x + 1

3.12 Integrazione delle funzioni razionali 4. I (x) =

Z

x3 +2x2 +1 dx. x5 −x4 +2x3 −2x2 +x−1

129

Qui è:

 2 p (x) = x3 +2x2 +1, q (x) = x5 −x4 +2x3 −2x2 +x−1 = x2 − 1 x2 + 1 Quindi:

A Bx + C Dx + E p (x) = + 2 + q (x) x−1 x +1 (x2 + 1)2 da cui il sistema: A+B+0+0+0=0 0−B+C +0+0=1 2A + B − C + D + 0 = 2 0−B−C −D+E =0 A+0−C +0−E =1 compatibile e determinato (A, B, C, D + E) = (1, −1, 0, 1, 0) Ciò implica: 1 x x p (x) = − 2 + q (x) x − 1 x + 1 (x2 + 1)2 Quindi l’integrale: I (x) = ln |x − 1| − 5. I (x) =

Z

x+1 dx. x2 −3x+2

 1 1 ln x2 + 1 − +K 2 2 (x2 + 1)

Qui è:

p (x) = x + 1, q (x) = x2 − 3x + 2 = (x − 2) (x − 1) Quindi: p (x) A B = + q (x) x−2 x−1 Affinché ciò sia vero, deve essere: A = 3, B = −2 Ciò implica: I (x) = 3 ln |x − 2| − 2 ln |x − 1| + K

3.12 Integrazione delle funzioni razionali 6. I (x) =

Z

x2 dx. (x+2)(x−1)2

Qui è:

p (x) = x2 , q (x) = (x + 2) (x − 1)2

Quindi: A B1 B2 p (x) = + + q (x) x + 2 x − 1 (x − 1)2

Affinché ciò sia vero, deve essere:

A + B1 + 0 = 1 −2A + B1 + B2 = 0 A − 2B1 + 2B2 = 0 L’unica soluzione di tale sistema è 5 1 4 A= , B= , C= 9 9 3 Ciò implica: I (x) = 7. I (x) =

Z

x2 +2 dx. (x−1)3

4 5 1 ln |x + 2| + ln |x − 1| − +C 9 9 3 (x − 1)

Qui è: p (x) = x2 + 2, q (x) = (x − 1)3

Quindi: A B p (x) C = + 2 + q (x) x − 1 (x − 1) (x − 1)3

Affinché ciò sia vero, deve essere:

A+0+0=1 −2A + B + 0 = 0 A−B+C =2 L’unica soluzione di tale sistema è A = 1, B = 2, C = 3 Ciò implica: I (x) = ln |x − 1| −

3 2 − +K x − 1 2 (x − 1)2

130

3.12 Integrazione delle funzioni razionali Z

8. I (x) =

x2 −1 dx. (x−2)(x2 +1)

131

Qui è:

 p (x) = x2 − 1, q (x) = (x − 2) x2 + 1

Quindi:

p (x) A Bx + C = + 2 q (x) x−2 x +1

Affinché ciò sia vero, deve essere:

A+B+0=1 0 − 2B + C = 0 A + 0 − 2C = −1 L’unica soluzione di tale sistema è (A, B, C) =



3 2 4 , , 5 5 5



Ciò implica:

9. I (x) =

Z

 4 3 1 I (x) = − ln |x − 2| + ln x2 + 1 + arctanx + C 5 5 5 x2 +1 dx. x3 −4x2 +5x−2

Qui è:

p (x) = x2 + 1, q (x) = x3 − 4x2 + 5x − 2 = (x − 1)2 (x − 2) Quindi: A B C p (x) = + + q (x) x − 2 x − 1 (x − 1)2

Affinché ciò sia vero, deve essere:

A+B+0=1 −2A − 3B + C = 0 A + 2B − 2C = 1 L’unica soluzione di tale sistema è (A, B, C) = (5, −4, −2) Ciò implica: I (x) = 5 ln |x − 2| +

2 − 4 ln |x − 1| + K x−1

3.12 Integrazione delle funzioni razionali 10. I (x) =

Z

2x2 +1 dx. x3 −2x2 +x−2

132

Qui è:

 p (x) = 2x2 − 1, q (x) = x3 − 2x2 + x − 2 = (x − 2) x2 + 1

Quindi:

A Bx + C p (x) = + 2 q (x) x−2 x +1 da cui A+B+0=2 0 − 2B + C = 0 A + 0 − 2C = 1 La soluzione è (A, B, C) = Perciò:



9 1 2 , , 5 5 5



p (x) 9 1 x+2 = + · 2 q (x) 5 (x − 2) 5 x + 1

L’integrale: I (x) =

1 9 ln |x − 2| + J (x) , 5 5

essendo: x+2 dx x2 + 1 Z Z dx x dx + 2 = 2 2 x +1 x +1 1 2 = ln x + 1 + 2 arctan x + C1 , 2

J (x) =

donde: I (x) = 11. I (x) =

Z

Z

x+1 dx. x3 −1

 2 1 9 ln |x − 2| + ln x2 + 1 + arctan x + C 5 10 5 Qui è:

 p (x) = x + 1, q (x) = x3 − 1 = (x − 1) x2 + x + 1

3.12 Integrazione delle funzioni razionali

133

Quindi: A Bx + C p (x) = + 2 q (x) x−1 x +x+1 da cui A+B+0=0 A−B+C =0 A+0−C =1 La soluzione è (A, B, C) = (1, −1, 0)

Perciò:

p (x) 1 x = − 2 q (x) x−1 x +x+1

L’integrale: I (x) = ln |x − 1| − J (x) , essendo: J (x) =

Z

x2

x dx +x+1

 1 1 = ln x2 + x + 1 − √ arctan 2 3



1 |x − 1| + √ arctan I (x) = ln √ x2 + x + 1 3





+ K1 ,



+K

2x + 1 √ 3

Si conclude che:

12. I (x) =

Quindi:

Z

x2 −2x dx. (2x−1)(x2 +1)

2x + 1 √ 3

Qui è:

 p (x) = x2 − 2x, q (x) = (2x − 1) x2 + 1 p (x) A Bx + C = + 2 q (x) 2x − 1 x +1

da cui A + 2B + 0 = 1 0 − B + 2C = −2 A+0−C =0

3.12 Integrazione delle funzioni razionali La soluzione è (A, B, C) = Perciò:



134

3 4 3 − , ,− 5 5 5



p (x) 3 1 4x − 3 =− + · 2 q (x) 5 (2x − 1) 5 x + 1

L’integrale: I (x) = −

1 3 ln |2x − 1| + J (x) , 10 5

essendo: 4x − 3 dx x2 + 1  = 2 ln x2 + 1 − 3 arctan x + C1 ,

J (x) =

Si conclude che: I (x) = −

Z

 3 2 3 ln |2x − 1| + ln x2 + 1 − arctan x + C 10 5 5 ***

3.12.7

Calcolare i seguenti integrali 1) 2) 3) 4)

3.12.8

Z

Z

Z

Z

5x2 +11x−2 dx (x+5)(x2 +9)

5)

x−1 dx 9x2 −18x+7

6)

x3 +x+1 dx x4 +x2

7)

dx (x2 −1)2

8)

Z

Z

Z

Z

Z

x2 −2x+3 dx (x−1)2 (x2 +1)

9)

x−3 dx (x−2)(x2 +x+1)2

10)

dx x(x2 +1)

11)

x3 +2x2 −2x dx x4 +4

12)

2x2 +12 dx x4 +12x2 +16

Z

dx x3 +x

Z

x3 +x−1 dx (x2 +1)2

Z

2x3 dx (x2 +1)2

Soluzioni

1. I (x) =

Quindi:

Z

5x2 +11x−2 dx. (x+5)(x2 +9)

Qui è:

 p (x) = 5x2 + 11x − 2, q (x) = (x + 5) x2 + 9 A Bx + C p (x) = + 2 q (x) x+5 x +9

3.12 Integrazione delle funzioni razionali

135

da cui A+B+0=5 0 + 5B + C = 11 9A + 0 + 5C = −2 La soluzione è (A, B, C) = (2, 3, −4)

Perciò:

p (x) 2 3x − 4 = + 2 q (x) x+5 x +9

L’integrale: I (x) = 2 ln |x + 5| + J (x) , essendo: 3x − 4 dx x2 + 9 x  4 3 + K1 , = ln x2 + 9 − arctan 2 3 3

J (x) =

Si conclude che:

Z

x  4 3 2 I (x) = 2 ln |x + 5| + ln x + 9 − arctan +K 2 3 3

R x−1 dx. Anziché procedere per decomposizione in frazioni 2. I (x) = 9x2 −18x+7 semplici, è preferibile procedere nel modo seguente Z 1 d (9x2 − 18x + 7) I (x) = 18 9x2 − 18x + 7 1 2 ln 9x − 18x + 7 + K = 18 3. I (x) =

Quindi:

R

x3 +x+1 dx. x4 +x2

Qui è:

 p (x) = x3 + x + 1, q (x) = x4 + x2 = x2 x2 + 1 A B Cx + D p (x) = + 2+ 2 q (x) x x x +1

3.12 Integrazione delle funzioni razionali

136

da cui A+0+C +0=1 0+B+0+D =0 A+0+0+0=1 0+B+0+0=1 La soluzione è (A, B, C, D) = (1, 1, 0, −1)

Perciò:

p (x) 1 1 1 = + 2− 2 q (x) x x x +1

L’integrale: I (x) = ln |x| − 4. I (x) =

R

dx . (x2 −1)2

1 − arctan x + K, x

Qui è:

2 p (x) = 1, q (x) = x2 − 1 = (x − 1)2 (x + 1)2

Quindi:

A B D C p (x) = + + 2 + q (x) x − 1 (x − 1) x + 1 (x + 1)2 da cui A+0+C +0=0 A+B−C +D =0 −A + 2B − C − 2D = 0 −A + B + C + D = 1 La soluzione è (A, B, C, D) = Perciò:



1 1 1 1 − , , , 4 4 4 4



1 1 1 p (x) 1 =− + + 2 + q (x) 4 (x − 1) 4 (x − 1) 4 (x + 1) 4 (x + 1)2 L’integrale:

  x + 1 1 2x − ln +K I (x) = 4 x − 1 x2 − 1

3.12 Integrazione delle funzioni razionali 5. I (x) =

R

x2 −2x+3 dx. (x−1)2 (x2 +1)

137

Qui è:

 p (x) = x2 − 2x + 3, q (x) = (x − 1)2 x2 + 1

Quindi:

A B Cx + D p (x) = + 2 + q (x) x − 1 (x − 1) (x2 + 1)

da cui

A+0+C +0=0 −A + B − 2C + D = 1 A + 0 + C − 2D = −2 −A + B + 0 + D = 3 La soluzione è (A, B, C, D) = (−1, 1, 1, 1) Perciò:

L’integrale:

1 1 x+1 p (x) =− + 2 + q (x) x − 1 (x − 1) (x2 + 1) I (x) = − ln |x − 1| +

essendo:

Quindi:

Quindi:

Z

x+1 dx x2 + 1 Z Z x dx = dx + 2 2 x +1 x +1  1 = ln x2 + 1 + arctan x 2

J (x) =

6. I (x) =

1 + J (x) , 1−x

R

√ x2 + 1 1 + ln + arctan x + K I (x) = 1−x |x − 1| x−3 dx. (x−2)(x2 +x+1)2

Qui è:

2 p (x) = x − 3, q (x) = (x − 2) x2 + x + 1 A Bx + C Dx + E p (x) = + 2 + 2 q (x) x − 2 x + x + 1 (x + x + 1)2

3.12 Integrazione delle funzioni razionali

138

da cui A+B+0+0+0=0 2A − B + C + 0 + 0 = 0 3A − B − C + D + 0 = 0 2A − 2B − C − 2D + E = 1 A + 0 − 2C + 0 − 2E = −2 La soluzione è   1 1 3 1 10 (A, B, C, D, E) = − , , , , 49 49 49 7 7 Perciò: 1 1 1 1 x+3 x + 10 p (x) =− + + 2 2 q (x) 49 x − 2 49 x + x + 1 7 (x + x + 1)2 L’integrale: I (x) = −

1 1 1 ln |x − 2| + J1 (x) + J2 (x) , 49 49 7

essendo: Z

 x+3 1 J1 (x) = dx = ln x2 + x + 1 + 2 x +x+1 2 Z 19x + 8 x + 10 + dx = J2 (x) = 2 3 (x2 + x + 1) (x2 + x + 1) Quindi: I (x) =

1 882

+K 7. I (x) =

Quindi:

R

h

42(19x+8) x2 +x+1

dx . x(x2 +1)

  2x + 1 5 √ arctan √ + K1 3 3   2x + 1 38 √ arctan √ + K2 3 3 3

  i √ 2x+1 2 √ + 562 3 arctan − 18 ln |x − 2| + 9 ln (x + x + 1) + 3

Qui è:  p (x) = 1, q (x) = x x2 + 1 A Bx + C p (x) = + 2 q (x) x x +1

3.12 Integrazione delle funzioni razionali

139

da cui A+B+0=0 0+0+C =0 A+0+0=1 La soluzione è (A, B, C) = (1, −1, 0)

Perciò:

p (x) 1 x = − 2 q (x) x x +1

L’integrale: I (x) = ln |x| − 8. I (x) =

R

essendo:

x3 +2x2 −2x dx. x4 +4

Abbiamo: Z Z 2x2 − 2x x3 dx + dx I (x) = x4 + 4 x4 + 4  1 = ln x4 + 4 + J (x) , 4 J (x) =

Abbiamo:

da cui:

 1 ln x2 + 1 + K 2

Z

2x2 − 2x dx x4 + 4

Ax + B Cx + D 2x2 − 2x = 2 + 2 4 x +4 x − 2x + 2 x + 2x + 2 A+0+C +0=0 2A + B − 2C + D = 1 2A + 2B + 2C − 2D = −1 0 + 2B + 0 + 2D = 0

La soluzione di tale sistema è: 1 1 1 1 A = ,B = − ,C = − ,D = 4 4 4 4 Quindi: J (x) =

1 [G1 (x) − G2 (x)] , 4

(3.66)

3.12 Integrazione delle funzioni razionali essendo: G1,2 =

Z

x2

140

x−1 dx ± 2x + 2

Calcolando i due integrali: 1 2 ln x − 2x + 2 + C1 2  1 G2 (x) = ln x2 + 2x + 2 − 2 arctan (x + 1) + C2 2 G1 (x) =

Finalmente:

 1 |x2 − 2x + 2| 1 4 I (x) = ln x + 4 + ln 2 + arctan (x + 1) + C 4 4 x + 2x + 2 9. I (x) =

R

2x2 +12 dx x4 +12x2 +16

=2

R

x2 +6 dx. x4 +12x2 +16

Qui è:

 √  √  p (x) = x2 + 6, q (x) = x4 + 12x2 + 16 = x2 + 6 + 2 5 x2 + 6 − 2 5 Quindi:

da cui il sistema:

Cx + D Ax + B p (x) √ + √ = 2 2 q (x) x +6+2 5 x +6−2 5

A+0+C +0=0 0+B+0+D =1   √  √  6−2 5 A+0+ 6+2 5 C +0=0   √  √  A+ 6−2 5 B+0+ 6+2 5 D =6 La soluzione è: (A, B, C, D) = donde: I (x) =

Z



1 1 0, , 0, 2 2

dx √ + 6 + 2 5 + x2

Z



,

dx √ 6 − 2 5 + x2

3.12 Integrazione delle funzioni razionali Calcoliamo (α > 0) Z

dx 1 = 2 α+x α

Z

1 =√ α

1+ Z

141

dx  2 √x α

d



1+

√x α



1 = √ arctan α



√x α



2

x √ α



+ const

Quindi: I (x) = q 10. I (x) =

R



1







x x 1     √  arctan q √  +q √  arctan q √  2 3+ 5 2 3+ 5 2 3− 5 2 3− 5

dx . x3 +x

Qui è:

p (x) = 1, q (x) = x3 + x

Quindi: p (x) A Bx + C = + 2 q (x) x x +1 da cui il sistema: A+B+0=0 0+0+C =0 A+0+0=1 La soluzione è: (A, B, C) = (1, −1, 0) , donde: Z

Z dx x I (x) = − dx 2 x x +1 |x| = ln √ +C x2 + 1

3.12 Integrazione delle funzioni razionali 11. I (x) =

R

2x3 dx. (x2 +1)2

142

Qui è: 2 p (x) = 2x3 , q (x) = x2 + 1

Quindi:

Ax + B Cx + D p (x) = 2 + q (x) x +1 (x2 + 1)2 da cui il sistema: A+0+0+0=2 0+B+0+0=0 A+0+C +0=0 0+B+0+D =0 La soluzione è: (A, B, C, D) = (2, 0, −2, 0) , donde: Z

12. I (x) =

R

Z 2x 2x dx I (x) = dx − 2 2 x +1 (x + 1)2 Z Z d (x2 + 1) d (x2 + 1) = − x2 + 1 (x2 + 1)2  1 = ln x2 + 1 + 2 +C x +1 x3 +x−1 dx. (x2 +1)2

Procedendo come nel n. 11: A+0+0+0=1 0+B+0+0=0 A+0+C +0=1 0 + B + 0 + D = −1,

la cui soluzione è: (A, B, C, D) = (1, 0, 0, −1) , donde: I (x) = ln



x2

+1−

Z

(x2

dx + 1)2

3.12 Integrazione delle funzioni razionali

143

L’integrale a secondo membro è dato dalla (3.63), che riscriviamo:   Z dx x 1 + arctan x + C1 = 2 x2 + 1 (x2 + 1)2 per cui: I (x) = ln

3.12.9



1 x2 + 1 − 2



(3.67)

 x + arctan x + C x2 + 1

Calcolare i seguenti integrali 1) 2) 3) 4)

3.12.10

Z

x4 +8x3 −x2 +2x+1 dx (x2 +x)(x3 +1)

5)

Z

x3 +x2 −5x+15 dx (x2 +5)(x2 +2x+3)

7)

Z

dx e2x −ex sin x dx cos x(1+cos2 x)

8)

Z

6)

Z

x3 +x2 +x+3 dx (x2 +1)(x2 +3)

Z

x2 −3x−1 dx x3 +x2 −2x

Z

xdx (x−2)2

Z

dx (x+1)2 (x2 −1)2

9)

Z

10)

Z

dx (x4 −1)2 dx (x+1)2 (x2 +1)

Soluzioni

1. I (x) =

Z

x4 +8x3 −x2 +2x+1 dx. (x2 +x)(x3 +1)

Qui è:

p (x) = x4 +8x3 −x2 +2x+1, q (x) = x2 + x Quindi:



  x3 + 1 = x (x + 1)2 x2 − x + 1

Dx + E p (x) A B C = + + 2 + 2 q (x) x x + 1 (x + 1) x −x+1 da cui A+B+0+D+0=1 A + 0 + C + 2D + E = 8 0 + 0 − C + D + 2E = −1 A+B+C +0+E =2 A+0+0+0+0=1 La soluzione è Perciò:

(A, B, C, D, E) = (1, −2, 3, 2, 0) 2x p (x) 1 2 3 = − + 2 + 2 q (x) x x + 1 (x + 1) x −x+1

3.12 Integrazione delle funzioni razionali

144

L’integrale: 3 + I (x) = ln |x| − 2 ln |x + 1| − x+1

Z

x2

2x dx −x+1

Calcoliamo a parte l’integrale a secondo membro: Z Z 2x − 1 + 1 2x dx = dx 2 x −x+1 x2 − x + 1 Z 2 1 = ln x − x + 1 + dx 2 x −x+1 Z 4 1 h i dx = ln x2 − x + 1 + 3 1 + √2 x − 1  2 3    2x − 1 2 2 √ + C1 , = ln x − x + 1 + √ arctan 3 3 donde: |x3 − x2 + x| 2 3 I (x) = ln + √ arctan − 2 x+1 3 (x + 1) 2. I (x) =

Quindi:

R

x3 +x2 −5x+15 dx. (x2 +5)(x2 +2x+3)



2x − 1 √ 3



+C

Qui è:

  p (x) = x3 + x2 − 5x + 15, q (x) = x2 + 5 x2 + 2x + 3 Ax + B Cx + D p (x) = 2 + 2 q (x) x +5 (x + 2x + 3)

da cui A+0+C +0=1 2A + B + 0 + D = 1 3A + 2B + 5C + 0 = −5 0 + 3B + 0 + 5D = 15 La soluzione è (A, B, C, D) = (0, −5, 1, 6) Da ciò segue: Z I (x) = −5

dx + 2 x +5

Z

x2

x+6 dx + 2x + 3

3.12 Integrazione delle funzioni razionali

145

Calcoliamo a parte i due integrali: Z

dx 1 =√ 2 x +5 5

Z

d



1+

√x 5



1 = √ arctan 5



√x 5



2

x √ 5



+C

Il secondo integrale si calcola con la (3.45):   Z √ x + 1 x+6 5 +C dx = ln x2 + 2x + 3 + √ arctan √ x2 + 2x + 3 2 2 Finalmente:     √ 5 1 x+1 x 2 I (x) = ln x + 2x + 3 + √ arctan √ − √ arctan √ +C 2 2 5 5 R 3. I (x) = e2xdx . Eseguiamo il cambio di variabile: −ex y = ex =⇒ dx =

per cui: I (y) =

Z

y2

dy , y

dy (y − 1)

Procediamo per decomposizione in frazioni semplici: 1 A B C = + + y 2 (y − 1) y y2 y − 1 Il principio di identità dei polinomi conduce al sistema di Cramer: A+0+C =0 −A + B + 0 = 0 0 − B + 0 = 1, con soluzione: (A, B, C) = (−1, −1, 1) Quindi:

y − 1 1 +C I (y) = + ln y y

Ripristinando la variabile x:

I (x) = e−x + ln 1 − e−x + C

3.12 Integrazione delle funzioni razionali 4. I (x) =

R

sin x dx. cos x(1+cos2 x)

Eseguiamo il cambio di variabile:

y = cos x =⇒ dy = − sin xdx,

donde:

Z I (y) = −

dy y (1 + y 2 )

Procediamo per decomposizione in frazioni semplici: 1 A By + C = + 2 y 2 (1 + y 2 ) y y +1 Il principio di identità dei polinomi conduce al sistema di Cramer: A+B+0=0 0+0+C =0 A + 0 + 0 = 1, con soluzione: (A, B, C) = (1, −1, 0) Quindi: |y| −I (y) = ln p +C 1 + y2

Ripristinando la variabile x:

I (x) = ln 5. I (x) =

Quindi:

R

x3 +x2 +x+3 dx. (x2 +1)(x2 +3)



1 + cos2 x +C |cos x|

Qui è:

  p (x) = x3 + x2 + x + 3, q (x) = x2 + 1 x2 + 3 Ax + B Cx + D p (x) = 2 + 2 q (x) x +1 x +3

da cui A+0+C +0=1 0+B+0+D =1 3A + 0 + C + 0 = 1 0 + 3B + 0 + D = 3

146

3.12 Integrazione delle funzioni razionali

147

La soluzione è (A, B, C, D) = (0, 1, 1, 0) Da ciò segue: I (x) = arctan x + 6. I (x) =

R

xdx . (x−2)2

Qui è:

 1 ln x2 + 3 + C 2

p (x) = x, q (x) = (x − 2)2

Quindi: A B p (x) = + q (x) x − 2 (x − 2)2

da cui

A=1 B − 2A = 0 La soluzione è (A, B) = (1, 2) Da ciò segue: I (x) = ln |x − 2| − 7. I (x) =

R

x2 −3x−1 dx. x3 +x2 −2x

Qui è:

2 +C x−2

p (x) = x2 − 3x − 1, q (x) = x3 + x2 − 2x = x (x + 2) (x − 1) Quindi:

da cui

A B C p (x) = + + q (x) x x+2 x−1 A+B+C =1 A − B + 2C = −3 2A + 0 + 0 = 1

La soluzione è (A, B, C) = Da ciò segue: I (x) = ln

q



 1 3 , , −1 2 2

x (x + 2)3 |x − 1|

+C

3.12 Integrazione delle funzioni razionali 8. I (x) =

Quindi:

R

dx . (x+1)2 (x2 −1)2

148

Qui è:

2 p (x) = 1, q (x) = (x + 1)2 x2 − 1 = (x + 1)4 (x − 1)2

p (x) A B C D E F = + 2 + 3 + 4 + 4 + q (x) x + 1 (x + 1) (x + 1) (x + 1) (x + 1) (x − 1)2 da cui (A + E) x5 + (A + B + 3E + F ) x4 + (−2A + C + 2E + 4F ) x3 + + (−2A − 2B − C + D − 2E + 6F ) x2 + + (A − C − 2D − 3E + 4F ) x+ + (A + B + C + D − E + F ) =1 Per il principio di identità dei polinomi: A+0+0+0+E+0=0 A + B + 0 + 0 + 3E + F = 0 −2A + 0 + C + 0 + 2E + 4F = 0 −2A − 2B − C + D − 2E + 6F = 0 A + 0 − C − 2D − 3E + 4F = 0 A+B+C +D−E+F =1 La soluzione è (A, B, C, D, E, F ) =



1 3 1 1 1 1 , , , ,− , 8 16 4 4 8 16



Da ciò segue: Z Z Z 1 dx 3 dx dx 1 I (x) = + 2 + 8 x + 1 16 (x + 1) 4 (x + 1)3 Z Z Z dx dx dx 1 1 1 + + 4 − 4 (x + 1) 8 x − 1 16 (x − 1)2 1 x + 1 3x3 + 6x2 + x − 4 − +C = ln 8 x − 1 12 (x + 1)3 (x − 1)

3.12 Integrazione delle funzioni razionali 9. I (x) =

R

dx . (x4 −1)2

149

Qui è:

2 p (x) = 1, q (x) = (x − 1)2 (x + 1)2 x2 + 1

Quindi:

p (x) A B D Gx + H C Ex + F = + + + 2 + 2 + q (x) x − 1 (x − 1) x + 1 (x + 1) x2 + 1 (x2 + 1)2 da cui A+0+C +0+E+0+0+0=0 A+B−C +D+0+F +0+0=0 A + 2B + C − 2D − E + 0 + G + 0 = 0 A + 3B − C + 3D + 0 − F + 0 + H = 0 −A + 4B − C − 4D − E + 0 − 2G + 0 = 0 −A + 3B + C + 3D + 0 − F + 0 − 2H = 0 −A + 2B − C − 2D + E + 0 + G + 0 = 0 −A + 2B + C + D + 0 + F + 0 + H = 1 La soluzione è (A, B, C, D, E, F, G, H) =



1 1 −3 1 3 , , , 0, , 0, 16 16 16 4 4



Da ciò segue: Z Z Z dx 1 dx dx 3 3 + I (x) = − 2 + 16 x − 1 16 (x − 1) 16 x + 1 Z Z Z 1 1 dx dx dx 1 + + 2 + 2 16 (x + 1) 4 x + 1 4 (x2 + 1)2 Z 3 x + 1 1 1 1 1 1 1 dx + const = ln − + + arctan x − 2 16 x−1 4 16 x − 1 16 x + 1 4 (x + 1)2

L’integrale

R

dx (x2 +1)2

è dato dalla (3.63) donde:

  1 + x 1 4x + 6 arctan x − 3 ln + const I (x) = 16 1 − x x4 − 1

3.12 Integrazione delle funzioni razionali 10. I (x) =

R

dx . (x+1)2 (x2 +1)

Qui è:

 p (x) = 1, q (x) = (x + 1)2 x2 + 1

Quindi:

A B Cx + D p (x) = + 2 + q (x) x + 1 (x + 1) x2 + 1 Per il principio di identità dei polinomi: A+0+C +0=0 A + B + 2C + D = 0 A + 0 + C + 2D = 0 A + B + 0 + D = 1, la cui soluzione è: (A, B, C, D) =



 1 1 1 , ,− ,0 2 2 2

Quindi: 1 1 p (x) x = + , 2 − 2 q (x) 2 (x + 1) 2 (x + 1) 2 (x + 1) da cui l’integrale: I (x) =

3.12.11

 1 1 1 ln |x + 1| − + ln x2 + 1 + C 2 2 (x + 1) 4

Funzioni razionali improprie

Qui il grado del numeratore è ≥ di quello del denominatore. Esempio Z 3 x +8 dx x−2 Eseguendo la divisione dei polinomi, otteniamo: x3 + 8 16 = x2 + 2x + 4 + , x−2 x−2

donde: Z

x3 + 8 x3 dx = + x2 + 4x + 16 ln |x − 2| + C x−2 3

150

3.12 Integrazione delle funzioni razionali

3.12.12

151

Calcolare i seguenti integrali 1) 2) 3) 4)

3.12.13

Z

x6 −x5 +x4 −x2 +x dx x4 −1

5) 6)

Z

x4 −2x3 +3x2 −x+3 dx x3 −2x2 +3x x2 +3x−4 dx x2 −2x−8

7)

x2 −5x+9 dx x2 −5x+6

8)

Z

Z

Z

5x3 +2 dx x3 −5x2 +4x

Z

x4 −6x3 +12x2 +6 dx x3 −6x2 +12x−8

Z

Z

9)

x3 −1 dx 4x3 −x

Z

x4 dx (1−x)3

x4 dx x4 −1

Soluzioni

1. I (x) =

R

x6 −x5 +x4 −x2 +x dx. x4 −1

Eseguendo la divisione tra polinomi:

1 x6 − x5 + x4 − x2 + x = x2 − x + 1 + 4 , 4 x −1 x −1

donde:

1 1 I (x) = x3 − x2 + x + I1 (x) , 3 2 Z dx I1 (x) = 4 x −1

essendo:

Per calcolare I1 (x) occorre applicare il metodo dei coefficienti indeterminati: 1 A B Cx + D = + + 2 , 4 x −1 x−1 x+1 x +1 da cui il sistema:

A+B+C +0=0 A−B+0+D =0 A+B−C +0=0 A−B+0−D =1 La cui unica soluzione è: (A, B, C, D) =



1 1 1 , − , 0, − 4 4 2



,

per cui: Z Z Z dx 1 dx 1 dx 1 − − I1 (x) = 4 x − 1 4 x + 1 2 x2 + 1 1 x − 1 1 = ln − arctan x + C 4 x + 1 2

3.12 Integrazione delle funzioni razionali Finalmente:

2. I (x) =

R

1 3 1 2 1 x − 1 1 I (x) = x − x + x + ln − arctan x + C 3 2 4 x + 1 2

x4 −2x3 +3x2 −x+3 dx. x3 −2x2 +3x

Risulta:

x4 − 2x3 + 3x2 − x + 3 3−x =x+ 3 , 3 2 x − 2x + 3x x − 2x2 + 3x

per cui:

1 I (x) = x2 + I1 (x) , 2

essendo: I1 (x) =

Z

3−x dx x3 − 2x2 + 3x

Applichiamo il metodo dei coefficienti indeterminati: x3

A Bx + C 3−x = + 2 , 2 − 2x + 3x x x − 2x + 3

ottenendo il sistema: A+B+0=0 −2A + 0 + C = −1 3A + 0 + 0 = 3, che ammette l’unica soluzione: (A, B, C) = (1, −1, 1) Quindi: Z

Z

x−1 dx − 2x + 3 Z 1 d (x2 − 2x + 3) = ln |x| − 2 x2 − 2x + 3  1 = ln |x| − ln x2 − 2x + 3 + C 2

I1 (x) =

dx − x

x2

Si conclude che:  1 1 I (x) = x2 + ln |x| − ln x2 − 2x + 3 + C 2 2

152

3.12 Integrazione delle funzioni razionali 3. I (x) =

R

x2 +3x−4 dx. x2 −2x−8

153

Risulta: 5x + 4 x2 + 3x − 4 =1+ 2 2 x − 2x − 8 x − 2x − 8

Perciè: I (x) = x + I1 (x) , essendo: I1 (x) =

Z

x2

5x + 4 dx − 2x − 8

Tale integrale si calcola con la (3.45): 3 x − 4 5 2 +C I1 (x) = ln x − 2x − 8 + ln 2 2 x + 2 = ln (x + 2) (x − 4)4 + C,

donde: 4. I (x) =

R

I (x) = x + ln (x + 2) (x − 4)4 + C

x2 −5x+9 dx. x2 −5x+6

Abbiamo: Z 2 x − 5x + 6 + 3 dx I (x) = x2 − 5x + 6 Z Z dx = dx + 3 2 x − 5x + 6

L’integrale a secondo membro si calcola con la (3.45): Z x − 3 dx + C, = ln x2 − 5x + 6 x − 2 per cui:

x − 3 +C I (x) = x + 3 ln x − 2 R R R 5x3 +2 2 −20x dx + 2+25x dx. Calcoliamo a parte il 5. I (x) = x3 −5x 2 +4x dx = 5 x3 −5x2 +4x secondo integrale: Z 25x2 − 20x + 2 def J (x) = dx x3 − 5x2 + 4x Metodo dei coefficienti indeterminati:

A B C 25x2 − 20x + 2 = + + , x3 − 5x2 + 4x x x−4 x−1

3.12 Integrazione delle funzioni razionali

154

da cui il sistema lineare: A + B + C = 25 5A + B + 4C = 20 4A + 0 + 0 = 2, che ammette l’unica soluzione: (A, B, C) =



1 161 7 , ,− 2 6 3



,

perciò: I (x) = 5x +

161 7 1 ln |x| + ln |x − 4| − ln |x − 1| + C 2 6 3

R R R x3 −1 1 dx + 6. I (x) = 4x 3 −x dx = 4 a parte il secondo integrale:

1 x−1 4 4x3 −x

def

J (x) =

dx =

Z

1 4

R

dx + 41

R

x−4 dx. 4x3 −x

Calcoliamo

x−4 dx 4x3 − x

Metodo dei coefficienti indeterminati: Z Z Z 4 7 9 J (x) = dx − dx − dx x 2 (2x − 1) 2 (2x + 1) 9 7 = 4 ln x − ln (2x − 1) − ln (2x + 1) + C 4 4 Quindi: 1 7 9 I (x) = x + ln x − ln (2x − 1) − ln (2x + 1) + C 4 16 16 7. I (x) =

per cui:

R

x4 −6x3 +12x2 +6 dx. x3 −6x2 +12x−8

Abbiamo:

x4 − 6x3 + 12x2 + 6 6 + 8x =x+ 3 , 3 2 x − 6x + 12x − 8 x − 6x2 + 12x − 8 Z 4x + 3 1 2 I (x) = x + 2 dx 3 2 x − 6x2 + 12x − 8

Applicando il metodo dei coefficienti indeterminati x3

4x + 3 4 11 = 3 + 2 − 6x + 12x − 8 (x − 2) (x − 2)2

(3.68)

3.12 Integrazione delle funzioni razionali Quindi

Z

155

4x + 3 5 − 8x +C dx = x3 − 6x2 + 12x − 8 2 (x − 2)2

Sostituendo nella (3.68): I (x) = 8. I (x) =

R

x4 dx. x4 −1

x4 − 4x3 + 4x2 − 16x + 10 +C 2 (x − 2)2

Abbiamo: x4 1 =1+ 4 , 4 x −1 x −1

donde: I (x) = x +

Z

dx −1

(3.69)

x4

Calcoliamo l’integrale a secondo membro della (3.69) con il metodo dei coefficienti indeterminati: A B Cx + D 1 = + + , x4 − 1 x−1 x+1 x2 + 1 da cui il sistema lineare: A+B+C +0=0 A−B+0+D =0 A+B−C +0=0 A − B + 0 − D = 1, la cui unica soluzione è: (A, B, C, D) = Quindi: x4 Perciò:

Z



1 1 1 , − , 0, − 4 4 2



1 1 1 1 = − − 2 −1 4 (x − 1) 4 (x + 1) 2 (x + 1) 1 x − 1 1 dx − arctan x + C = ln x4 − 1 4 x + 1 2

Sostituendo nella (3.69):

1 x − 1 1 I (x) = x + ln − arctan x + C 4 x + 1 2

3.12 Integrazione delle funzioni razionali 9. I (x) =

R

x4 dx. (1−x)3

156

Abbiamo: −8x + 6x2 + 3 x4 = −x − 3 + , (1 − x)3 (1 − x)3

da cui l’integrale:  Z  6x2 − 8x + 3 −x − 3 − I (x) = dx (x − 1)3 Z 6x2 − 8x + 3 1 2 dx = − x − 3x − 2 (x − 1)3

(3.70)

Calcoliamo l’integrale a secondo membro della (3.70) con il metodo dei coefficienti indeterminati: 6x2 − 8x + 3 B A C + = , 3 2 + x − 1 (x − 1) (x − 1) (x − 1)3 da qui il sistema lineare: A+0+0=6 −2A + B + 0 = −8 A − B + C = 3, che ammette l’unica soluzione: (A, B, C) = (6, 4, 1) Quindi Z 1 4 1 6x2 − 8x + 3 − dx = 6 ln |x − 1| − +C 3 2 + x − 1 2 (x − 1) (x − 1) (x − 1)3 Sostituendo nella (3.70) 8x − 7 1 +C I (x) = − x2 − 3x − 6 ln |x − 1| + 2 2 (x − 1)2

3.13 In (x) =

3.13

Z

dx , (x2 −1)n

In (x) =

Z

Z

Jn (x) =

dx

n x2 −1

(

)

dx (x2 +1)n

, Jn (x) =

157

Z

dx

n x2 +1

(

)

Iniziamo con il primo eseguendo il cambio di variabile (anziché applicare il metodo dei coefficienti indeterminati): t=

x−1 x+1

(3.71)

Dalla (3.71) otteniamo: x= donde:

1+t 2 dt, , dx = 1−t (1 − t)2

In (t) =

1 22n−1

Z

(1 − t)2(n−1) dt tn

(3.72)

La (3.72) può essere utilizzata solo per piccoli valori di n. Ad esempio, per n = 1: Z 1 1 dt = ln |t| + C I1 (t) = 2 t 2 Ripristinando la variabile x: 1 x − 1 I1 (x) = ln +C 2 x + 1

Osservazione. Il risultato precedente è equivalente a: Z dx = − arctanh x + C 2 x −1 Per n = 2:

Z 1 (1 − t)2 dt I2 (x) = 8 t2 Z Z  Z 1 dt dt = + dt −2 8 t2 t   1 1 = − − 2 ln |t| + t + C1 8 t Ripristinando la variabile x:

3.13 In (x) =

Z

dx , (x2 −1)n

Jn (x) =

dx (x2 +1)n

158

 x − 1 x − 1 x+1 − 2 ln + − +C x−1 x + 1 x + 1 x 1 x − 1 +C = − ln 2 (1 − x2 ) 4 x + 1

1 I2 (x) = 8

Per n = 3:

Z



Z 1 (1 − t)4 I3 (x) = dt 32 t3  Z Z Z Z Z dt dt dt 1 −4 +6 − 4 dt + tdt = 32 t3 t2 t   1 4 1 2 1 = − 2 + + 6 ln |t| − 4t + t + C 32 2t t 2 Ripristinando la variabile x: # " 2 x − 1 x+1 1 (x − 1) 1 1 (x + 1)2 x − 1 −4 +4 +C I3 (x) = + 6 ln + − 32 2 (x − 1)2 x−1 x + 1 x + 1 2 (x + 1)2 x (3x2 − 5) 3 x − 1 = + ln +C 16 x + 1 8 (x2 − 1)2

I rimanenti integrali (n > 3) sono riportati in Appendice A. Passiamo a Jn (x); per n = 1 è un integrale fondamentale: J1 (x) = arctan x + C Per n = 2, abbiamo già calcolato (vedi eq.3.59): J2 (x) =

x 1 arctan x + +C 2 2 2 (x + 1)

È possibile giungere al medesimo risultato attraverso un’integrazione per parti. Per ogni n: Z

x2 + 1 − x2 dx (x2 + 1)n Z Z x2 + 1 x2 = dx − dx, (x2 + 1)n (x2 + 1)n

Jn (x) =

cioè:

3.13 In (x) =

Z

dx , (x2 −1)n

Jn (x) =

Z

dx (x2 +1)n

Jn (x) = Jn−1 (x) − Jn (x) ,

essendo:

def

Jn (x) = Per n = 2:

Z

x2 dx (x2 + 1)n

159

(3.73)

(3.74)

J2 (x) = J1 (x) − J2 (x)

Qui è:

Z

x2 dx (x2 + 1)2 Per calcolare tale integrale, procediamo per parti osservando che:   1 2xdx d = − , x2 + 1 (x2 + 1)2 donde: J2 (x) =

  Z 1 1 J2 (x) = − xd 2 x2 + 1   Z dx 1 1 − x 2 =− 2 x +1 x2 + 1   1 x =− − J1 (x) 2 x2 + 1 Quindi: x 1 J2 (x) = J1 (x) + 2 2 2 (x + 1) Ma J1 (x) =

R

dx x2 +1

= arctan x+const, per cui: J2 (x) =

x 1 arctan x + +C 2 2 (x2 + 1)

Passiamo a n = 3; per la (3.73):

Qui è:

J3 (x) = J2 (x) − J3 (x)

J3 (x) =

Z

x2 dx (x2 + 1)3

(3.75)

3.13 In (x) =

Z

dx , (x2 −1)n

Jn (x) =

Z

dx (x2 +1)n

160

Anche in questo caso procediamo per parti osservando che:   1 4xdx d , 2 = − 2 (x + 1) (x2 + 1)3

donde:

  Z 1 1 xd J3 (x) = − 4 (x2 + 1)2 x 1 1 =− + J2 (x) 2 4 (x2 + 1) 4 Quindi: 3 x 1 J3 (x) = J2 (x) + 4 4 (x2 + 1)2 L’integrale J2 (x) è dato dalla (3.75), per cui: 3 arctan x + 8 3 = arctan x + 8

J3 (x) =

3 x 1 x +C + 2 2 8 x + 1 4 (x + 1)2 x (3x2 + 5) +C 8 (x2 + 1)2

(3.76)

Iterando il procedimento, si trova: Jn (x) =

2n − 3 x Jn−1 (x) + +C 2 (n − 1) 2 (n − 1) (x2 + 1)n−1

(3.77)

Ad esempio per n = 4:

x 5 J4 (x) = J3 (x) + (3.78) 6 6 (x2 + 1)3 5 x 5 x 5 x arctan x + + = + +C 2 16 16 x2 + 1 24 (x2 + 1) 6 (x2 + 1)3 Per n = 5: x 7 (3.79) J5 (x) = J4 (x) + 2 8 8 (x + 1)4 35 x 35 x x 35 7 x arctan x + + = + + +C 2 3 2 128 128 x + 1 192 (x2 + 1) 48 (x2 + 1) 8 (x2 + 1)4 Una tabella di valori di Jn (x) è riportata in Appendice A.

3.14 Integrali di funzioni irrazionali

3.14 3.14.1

161

Integrali di funzioni irrazionali Integrali del “tipo 1”

Hanno un espressione generale:   p1   p2  pn # Z "  ax + b q2 ax + b qn ax + b q1 , , ..., R x, dx, cx + d cx + d cx + d essendo R una funzione razionale, mentre pi , qi ∈ N − {0}. Gli integrali di questo tipo si riconducono ad integrali di funzioni razionali attraverso la sostituzione: ax + b = tk cx + d Qui k è il m.c.m. di q1 , q2 , ...qn . Ad esempio, calcoliamo l’integrale: Z dx √ I (x) = √ 2x − 1 − 4 2x − 1

La (3.80) è:

2x − 1 = t4 ,

donde:

dx = 2t3 dt Quindi l’integrale: Z t2 I (t) = 2 dt t−1 Z  t+1+ =2

1 t−1



dt

= (t + 1)2 + 2 ln |t − 1| + C

Ripristinando la variabile x: I (x) = 1 +

√ 4

√ 2 2x − 1 + 2 ln 4 2x − 1 − 1 + C

(3.80)

3.14 Integrali di funzioni irrazionali

3.14.2

Calcolare i seguenti integrali 1) 2) 3) 4)

3.14.3

162

Z

Z

Z

Z

3 √x dx x−1

5)

xdx √ 3 ax+b

6)



dx √

x+1+

(x+1)3

7) 8)

√ dx√ x+ 3 x

Z

Z

Z

Z

√ x−1 √ 3 x+1 dx √ x+1+2 √ dx (x+1)2 − x+1 √ x dx x+2 dx √ (2−x) 1−x

Z

q x x−1 dx x+1 Z q 3 x+1 dx 10) x−1 Z 11) x2 √x+3 dx 2x+3 q Z 2− 3 2x+1 x−1 q dx 12) 2x+1 9)

x+4

x−1

Soluzioni

1. I (x) =

R

3 √x dx. x−1

Qui è q1 = 2, n = 1, perciò: k = 2. La (3.80) porge: x − 1 = t2

Ciò implica: dx = 2tdt Quindi l’integrale in funzione di t: Z 3 t2 + 1 dt I (t) = 2

2 6 = t7 + t5 + 2t3 + 2t + K 7 5

Ripristinando la variabile x: q q q √ 2 6 7 5 (x − 1) + (x − 1) + 2 (x − 1)3 + 2 x − 1 + C1 I (x) = 7 5   √ 1 3 3 2 (x − 1) + (x − 1) + x + C =2 x−1 7 5 2. I (x) =

R

xdx √ . 3 ax+b

Qui è q1 = 3, n = 1, perciò: k = 3. La (3.80) porge: ax + b = t3

Ciò implica:  1 3 t −b a 3t2 dx = dt a

x=

3.14 Integrali di funzioni irrazionali

163

Quindi l’integrale in funzione di t: Z  3t 1 3 I (t) = dt t −b a a Z  3 = 2 t t3 − b dt a   3 1 5 b 2 t − t + const = 2 a 5 2 Ripristinando la variabile x:   q q 3 13 3 5 2 I (x) = 2 (ax + b) − b (ax + b) + const a 5 3. I (x) =

R



dx √

x+1+

(x+1)3

=

R

i h 3 1 R (x + 1) 2 , (x + 1) 2 . Qui è q1 = q2 = 2,

perciò: k = 2. La (3.80) porge:

x + 1 = t2 Ciò implica: dx = 2tdt Quindi l’integrale in funzione di t: Z

2tdt t + t3 Z dt =2 1 + t2 = 2 arctan t + const I (t) =

Ripristinando la variabile x: √ I (x) = 2 arctan x + 1 + const R

4. I (x) = = La (3.80) porge: √ dx√ x+ 3 x

R

 1 1 R x 2 , x 3 dx. Qui è q1 = 2, q2 = 3, perciò: k = 6. x = t6

Ciò implica: dx = 6t5 dt

3.14 Integrali di funzioni irrazionali

164

Quindi l’integrale in funzione di t: Z

6t5 dt t3 + t2 Z t3 dt =6 1+t   Z 1 2 t −t+1− =6 t+1 3 2 = 2t − 3t + 6t − ln |t + 1| + const I (t) =

Ripristinando la variabile x: √ √ √ √ I (x) = 2 x − 3 3 x + 6 6 x − 6 ln 6 x + 1 + const

R  1 1 R √x−1 dx = R x 2 , x 3 dx. Qui è q1 = 2, q2 = 3, perciò: k = 6. 5. I (x) = √ 3 x+1 La (3.80) porge: x = t6 Ciò implica: dx = 6t5 dt Quindi l’integrale in funzione di t: Z

t3 − 1 5 6t dt I (t) = t2 + 1 Z 5 3 t (t − 1) =6 dt t2 + 1  Z  t−1 6 4 3 2 t −t −t +t +t−1− 2 =6 t +1   Z 1 7 1 5 1 4 1 3 1 2 t−1 =6 t − t − t + t + t −t− 7 5 4 3 2 t2 + 1   1 7 1 5 1 4 1 3 1 2 1 2 =6 t − t − t + t + t − t − ln t + 1 + arctan t + const 7 5 4 3 2 2

Ripristinando la variabile x:

√ 6√ 3√ 6 √ 6 3 x5 − x2 + 2 x I (x) = x 6 x − 2√ 7√ 5 √ √ +3 3 x − 6 6 x − 3 ln 3 x + 1 + 6 arctan 6 x + const

3.14 Integrali di funzioni irrazionali 6. I (x) =

R



x+1+2 √ dx (x+1)2 − x+1

=

R

165

i h 1 1 R (x + 1)2 , (x + 1) 2 , (x + 1) 2 dx. Qui è: p1 = 2, q1 = 1 p2 = 1, q1 = 2 p3 = 1, q3 = 2,

donde: k=2 Il cambio di variabile è: x + 1 = t2 Ciò implica: dx = 2tdt L’integrale:

Z 2 t + 2t I (t) = 2 dt t4 − t

L’integrando è una funzione razionale propria:

t+2 1 t+1 = − 2 , 2 (t − 1) (t + t + 1) t−1 t +t+1 per cui: Z

 t+1 I (t) = 2 dt t2 + t + 1   Z t+1 dt = 2 ln |t − 1| − t2 + t + 1 dt − t−1

Z

(3.81)

L’integrale al secondo termine del secondo membro della (3.81) si calcola attraverso la (3.45) ottenendo:   Z  t+1 1 1 2t + 1 2 √ +C dt = ln t + t + 1 + √ arctan t2 + t + 1 2 3 3

Sostituendo nella (3.81):

 2 I (t) = 2 ln |t − 1| − ln t + t + 1 − √ arctan 3 2



2t + 1 √ 3



+C

Ripristinando la variabile x: 2 √   √ x+1−1 2 2 x+1+1 √ √ +C − √ arctan I (x) = ln x+2+ x+1 3 3

3.14 Integrali di funzioni irrazionali 7. I (x) =

R



x dx x+2

=

R

166

  1 R x, x 2 dx. Qui è k = 2. La (3.80) porge: x = t2

Ciò implica: dx = 2tdt Quindi l’integrale in funzione di t: Z

t2 dt I (t) = t2 + 2 Z  Z dt =2 dt − 2 2 t +2     t √ Z d √2   = 6 t − 2  2  1 + √t2   √ t = 2 t − 2 arctan √ 2 Ripristinando la variabile x: r   √ √ x I (x) = 2 x − 2 arctan + const 2 i R R h 1 8. I (x) = (2−x)dx√1−x = R x, (1 − x) 2 dx. Qui è q1 = 2, perciò: k = 2. La (3.80) porge: 1 − x = t2 Ciò implica: dx = −2tdt Quindi l’integrale in funzione di t: Z I (t) = −2

dt 1 + t2 = −2 arctan t + const Ripristinando la variabile x: I (x) = −2 arctan



 1 − x + const

3.14 Integrali di funzioni irrazionali 9. I (x) =

R q x−1 R h x x+1 dx = R x,

1−x 1+x

167

 pq1 i 1

dx. Qui è:

p1 = 1, q1 = 2,

donde: k=2 La (3.80) porge:

x−1 = t2 x+1

Risolvendo rispetto a x: x= da cui il differenziale: dx =

1 + t2 1 − t2

4t dt (1 − t2 )

Quindi l’integrale in funzione di t: Z 2 2 t (t + 1) I (t) = 4 dt (1 − t2 )3 Applicando il metodo dei coefficienti indeterminati: 1 t + 1 t + 3t2 I (t) = ln + +C 2 t − 1 (t2 − 1)2

Ripristinando la variabile x: √ √ 1 x − 1 + x + 1 √ + I (x) = ln √ 2 x − 1 − x + 1 √ √ (x + 1)2 3 (x − 1) x + 1 + (x + 1) x − 1 q +C + 4 3 (x + 1) 10. I (x) =

Rq 3 x+1

dx = x−1

R

R

h

x+1 x−1

 pq1 i 1

. Qui è:

p1 = 1, q1 = 3,

donde: k=3

3.14 Integrali di funzioni irrazionali La (3.80) porge:

Risolvendo rispetto a x:

x+1 = t3 x−1 x=

da cui il differenziale:

168

t3 − 1 t3 − 1

6t2 dx = − 3 dt (t − 1)

Quindi l’integrale in funzione di t:

Z I (t) = −6

t3 dt (t3 − 1)2

Sviluppiamo l’integrando in frazioni parziali: t+3 t+1 t3 1 1 − + = + 2 2 9 (t − 1) 9 (t2 + t + 1) 3 (t2 + t + 1)2 (t3 − 1) 9 (t − 1) Quindi: Z Z Z t+3 t+1 1 1 1 1 t3 1 − dt− dt = ln |t − 1|− dt 2 2 9 9t−1 9 t +t+1 3 (t2 + t + 1)2 (t3 − 1) Risulta: Z

   1 5 1 + 2t t+3 2 √ + C1 dt = ln t + t + 1 + √ arctan t2 + t + 1 2 3 3   Z t+1 2 1 + 2t t−1 √ + C2 + √ arctan dt = 3 (t2 + t + 1) 3 3 3 (t2 + t + 1)2

donde:    2  √ 1 + 2t 6t t +t+1 1 √ + 3 ln +C I (t) = + 2 3 arctan 3 t −1 3 (t − 1)2 Ripristinando la variabile x: q q  3 (x + 1)2 + 3 (x − 1)2 − 2√ 3 x2 − 1 1 q + I (x) = ln q √ 3 2 2 3 3 3 2 (x + 1) + x − 1 − (x − 1) ! √ √  3 p √ x−1+23x+1 3 p − 2 3 arctan − 3 (x2 − 1) (x − 1) + C 3 3 (x − 1)

3.14 Integrali di funzioni irrazionali 11. I (x) =

R

x+3 √ dx. x2 2x+3

169

Qui è: p1 = 1, q1 = 2,

donde: k=2 La (3.80) porge: 2x + 3 = t2 Risolvendo rispetto a x: x= da cui il differenziale:

 1 2 t −3 2

dx = tdt Quindi l’integrale in funzione di t: Z I (t) = 2

t2 + 3 dt (t2 − 3)2

Sviluppiamo l’integrando in frazioni parziali: 6 t2 + 3 1 + = 2 2 t − 3 (t2 − 3)2 (t2 − 3) L’integrale in funzione di t I (t) = −

t2

2t +C −3

Ripristinando la variabile x: I (x) = − 12. I (x) =

R

q 2− 3 2x+1 x−1 q dx x+4 2x+1 x−1

=

R

h R x,

cui il cambio di variabile è:



2x + 3 +C x

 2x+1 1/3 x−1

,

2x + 1 = t6 x−1 Risolvendo rispetto a x: x=

t6 + 1 t6 − 2

 2x+1 1/4 x−1

i

dx. Qui è k = 6, per

3.14 Integrali di funzioni irrazionali

170

Differenziando rispetto a t: dx = −18

t5 dt (t6 − 2)

L’integrale diventa: Z I (t) = 18

t7 − 2t5 dt, (t6 − 2) (4t9 + t6 − 8t3 + 1)

che è l’integrale di una funzione razionale propria. Riducendo:  Z 5  Z 8t8 + 2t5 − 8t4 + t 2 t −t I (t) = 18 dt + dt 3 t6 − 2 3 (4t9 + t6 − 8t3 + 1) Il primo integrale a secondo membro è: " ! # √ √ Z 5 2 + √ 3 3 3 2 2 4 √ √ √ 1 + 1 t −t 4t 4t + 2t 3 3 √ √ dt = 2 2 3 arctan + 2 ln + 4 ln t6 − 2 3 2 t6 − 2 24 3 4t − 2 + C1

Il secondo integrale non si esprime in forma chiusa. Utilizzando il software Mathematica: Z 8 8t + 2t5 − 8t4 + t 1 X ln (t − ρ) − 8ρ3 ln (t − ρ) + 2ρ4 ln (t − ρ) + 8ρ7 ln (t − ρ) dt = , 4t9 + t6 − 8t3 + 1 6 ρ 6ρ7 + ρ4 − 4ρ dove la somma è estesa a tutte le radici reali ρ del polinomio 4t9 +t6 −8t3 +1.

3.14.4

Integrali del “tipo 2”

Hanno un espressione generale: In (x) =

Z



pn (x) dx, ax2 + bx + c

(3.82)

essendo pn (x) un assegnato polinomio di grado n: pn (x) =

n X

ak x k

k=0

Qui è n ≥ 2, poichè per n = 0, 1 l’integrale si calcola con i metodi visti nelle sezioni precedenti.

3.14 Integrali di funzioni irrazionali

171

Proposizione. L’integrale (3.82) è: In (x) = qn−1 (x)



ax2 + bx + c + λ

Z



ax2

dx , + bx + c

(3.83)

essendo λ ∈ R, mentre qn−1 (x) è un polinomio di grado n − 1 a coefficienti indeterminati: n−1 X qn−1 (x) = bk xk k=0

Dimostrazione. Omessa

I coefficienti indeterminati e il numero reale λ si ottengono derivando primo e secondo membro della (3.83): n X

ak x k

√ k=0 ax2 +

bx + c

=



+√

ax2

ax2

n−2 X

n−1 X 2ax + b + bx + c (k + 1) bk+1 x + √ bk xk + 2 2 ax + bx + c k=0 k=0 k

λ , + bx + c

da cui: n−2 n n−1 X X X k k 2 2 ak x = 2 ax + bx + c (k + 1) bk+1 x + (2ax + b) bk xk + 2λ (3.84) k=0

k=0

k=0

Per il principio di identità dei polinomi la n-pla (bn−1 , bn−2 , ..., b0 , λ) è la soluzione di un sistema di Cramer. Esempio Z 4 x + 4x2 I4 (x) = √ x2 + 4 La (3.83) è:

I4 (x) = q3 (x) Derivando:



x2

+4+λ

Z



dx x2 + 4

√ x4 + 4x2 λ x (b3 x3 + b2 x2 + b1 x + b0 ) √ √ x2 + 4 + = 3b3 x2 + 2b2 x + b1 +√ x2 + 4 x2 + 4 x2 + 4

Segue il sistema:

3.14 Integrali di funzioni irrazionali

172

4b3 + 0 + 0 + 0 + 0 = 1 0 + 3b2 + 0 + 0 + 0 = 0 12b3 + 0 + 2b1 + 0 + 0 = 4 0 + 8b2 + 0 + b0 + 0 + 0 = 0 0 + 0 + 4b1 + 0 + λ = 0, la cui unica soluzione è: (b3 , b2 , b1 , b0 , λ) = donde:



 1 1 , 0, , −2 , 4 2

Z x3 + 2x √ 2 dx I4 (x) = x +4−2 √ 4 x2 + 4   3 √ x + 2x √ 2 2 = x + 4 − 2 ln x + x + 4 + C 4

3.14.5

Calcolare i seguenti integrali 1) 4)

1. I (x) = Z

R

Z

Z

2 √ x dx x2 −x+1



4x dx x2 −4x+2

2 √ x dx. x2 −x+1

2)

Z

5 √x dx 1−x2

3)

Z

6

√x dx 1+x2

La (3.83) è:

√ x2 √ dx = (b1 x + b0 ) x2 − x + 1 + λ x2 − x + 1

Z



dx x2 − x + 1

(3.85)

Derivando primo e secondo membro e riordinando i termini: 2x2 = 4b1 x2 + (2b0 − 3b1 ) x + (−b0 + 2b1 + 2λ) Per il principio di identità dei polinomi: 2b1 + 0 + 0 = 1 −3b1 + 2b0 + 0 = 0 2b1 − b0 + λ = 0

(3.86)

3.14 Integrali di funzioni irrazionali

173

La soluzione del sistema (3.86) è: (b1 , b0 , λ) =



1 3 1 , ,− 2 4 8



Sostituendo nella (3.85): I (x) =

1 1 (2x + 3) − J (x) , 4 8

essendo: J (x) =

Z



dx x2 − x + 1

(3.87)

L’integrale (3.87) si calcola attraverso la (3.46), e si ottiene: √ 2x − 1 + 2 x2 − x + 1 + C1 √ J (x) = ln 3 √ = ln 2x − 1 + 2 x2 − x + 1 + C2 ,

2. I (x) =

R



3 nella costante di integrazione. In definitiva: √ 1 1 I (x) = (2x + 3) − ln 2x − 1 + 2 x2 − x + 1 + C 4 8

avendo incorporato ln

5

√x dx. 1−x2

Z

La (3.83) è:

√ x5 √ dx = q4 (x) 1 − x2 + λ 1 − x2

Z



dx 1 − x2

Applicando direttamente la (3.84): 5

2x = 2 1 − x

2

3 X k=0

(k + 1) bk+1 x

k+1

− 2x

4 X

bk xk + 2λ

k=0

Si ottiene: 1 4 8 b4 = − , b3 = 0, b2 = − , b1 = 0, b0 = − , λ = 0 5 15 15 da cui: I (x) = −

√ 1 3x4 + 4x2 + 8 1 − x2 + C 15

3.14 Integrali di funzioni irrazionali 3. I (x) =

R

6

√x dx. 1+x2

Z

174

La (3.83) è:

√ x6 √ dx = q6 (x) 1 + x2 + λ 1 + x2

Z



dx 1 + x2

Applicando direttamente la (3.84): 4 5 X X x6 = x2 + 1 (k + 1) bk+1 xk + x bk xk + λ k=0

k=0

Per il principio di identità dei polinomi: 6b5 + 0 + 0 + 0 + 0 + 0 + 0 = 1 0 + 5b4 + 0 + 0 + 0 + 0 + 0 = 0 5b5 + 0 + 4b3 + 0 + 0 + 0 + 0 = 0 0 + 4b4 + 0 + 3b2 + 0 + 0 + 0 = 0 0 + 0 + 3b3 + 0 + 2b1 + 0 + 0 = 0 0 + 0 + 0 + 2b2 + 0 + b0 + 0 = 0 0 + 0 + 0 + 0 + b1 + 0 + λ = 0 Tale sistema ammette l’unica soluzione:   5 5 5 1 , 0, − , 0, , 0, − (b5 , b4 , b3 , b2 , b1 , b0 , λ) = , 6 24 16 16 donde:

5 5 1 x − x3 + x5 16 24 6 √ = ln x + x2 + 1 + C1 , si ottiene:

q5 (x) = Ricordando che I (x) = 4. I (x) =

R



R

√ dx x2 +1

√ √ 1 5 8x5 − 10x3 + 15x x2 + 1 − ln x + x2 + 1 + C 48 16

4x dx. x2 −4x+2

Si calcola con la (3.46):

√ √ I (x) = 4 x2 − 4x + 2 + 8 ln x − 2 + x2 − 4x + 2 + const

3.14 Integrali di funzioni irrazionali

3.14.6

175

Integrali del “tipo 3”

Hanno un espressione generale: Z Jn (x) =

dx √ (x − x0 ) ax2 + bx + c

(3.88)

n

Si osservi che per n = 0, 1 l’integrale (3.88) è del tipo (3.46)-(3.47) rispettivamente, donde assumiamo n ∈ N − {0, 1}. Eseguiamo il cambio di variabile: 1 =t x − x0

L’integrale diventa: Z Jn (t) = − p

essendo:

tn−1 αt2 + βt + γ

(3.89)

(3.90)

dt = −In−1 (t) ,

(3.91)

α = ax20 + bx0 + c β = 2ax0 + b γ=a e In−1 (t) =

Z

tn−1 p dt αt2 + βt + γ

Abbiamo quindi ricondotto l’integrale al tipo 2, per cui è n ≥ 3. Quindi: In−1 (t) = qn−2 (t) n−1

2t

2

p

αt2

+ βt + γ + λ

= 2 αt + βt + γ

n−3 X

Z

dt p

k

(k + 1) bk+1 t + (2αt + β)

k=0

3.14.7

(3.92)

αt2 + βt + γ n−2 X k=0

Calcolare i seguenti integrali 1)

Z

dx √ (x+1)3 x2 +2x

2)

Z

√dx x5 x2 −1

3)

Z

x2 +x+1 √ dx x x2 −x+1

bk tk + 2λ

3.14 Integrali di funzioni irrazionali

3.14.8 1.

R

176

Soluzioni dx √ . (x+1)3 x2 +2x

Qui è: α = −1, β = 0, γ = 1

Quindi:

Calcoliamo I2 (t):

Z t2 dt J3 (t) = −I2 (t) = − √ 1 − t2 √

I2 (t) = q1 (t) 1 − t2 + λ

Z

(3.93)

dt √ 1 − t2

Per la seconda delle (3.92):

Ciò implica:

 2t2 = 2 1 − t2 b1 + (−2t) (b0 + b1 t) + 2λ −2b1 + 0 + 0 = 1 0 − b0 + 0 = 0 b1 + 0 + λ = 0,

la cui unica soluzione è: (b1 , b0 , λ) = Osservando che

R

√ dt 1−t2



1 1 − , 0, 2 2



= arcsin t si ottiene:

I2 (t) =

 1 √ −t 1 − t2 + arcsin t + C2 2

Per la (3.93): 1 J3 (x) = 2 2.

R

√dx . x5 x2 −1

Quindi:

"√

# 1 x2 + 2x +C − arcsin x+1 (x + 1)2

Qui è: α = 0, β = 0, γ = 1 Z t4 dt J5 (t) = −I4 (t) = − √ 1 − t2

(3.94)

3.14 Integrali di funzioni irrazionali

177

Calcoliamo I4 (t): √

I4 (t) = q3 (t) 1 −

t2



Z

dt √ 1 − t2

Per la seconda delle (3.92):    2t4 = 2 1 − t2 b1 + 2b2 t + 3b3 t2 + (−2t) b0 + b1 t + b2 t2 + b3 t3 + 2λ

Ciò implica:

−4b3 + 0 + 0 + 0 + 0 = 1 0 − 3b2 + 0 + 0 + 0 = 0 3b3 + 0 − 2b1 + 0 + 0 = 0 0 + 2b2 + 0 − b0 + 0 = 0 0 + 0 + b1 + 0 + λ = 0, la cui unica soluzione è: (b3 , b2 , b1 , b0 , λ) = Osservando che

Per la (3.94):

R

R

3 3 1 − , 0, − , 0, 4 8 8



= arcsin t+const, si ottiene: i √ 1h 3 2 − 3t + 8t I4 (t) = 1 − t + arcsin t + C1 8 √ dt 1−t2

1 J5 (x) = 8 3.



x2 +x+1 √ dx. x x2 −x+1



2 + 3x2 √ 2 1 x − 1 − arcsin 4 x x



+C

Spezziamo l’integrale: Z x2 + x + 1 √ F (x) = dx x x2 − x + 1 3 X = Fk (x) , k=1

essendo: Z

xdx −x+1 Z dx √ F2 (x) = 2 x −x+1 Z dx √ F3 (x) = x x2 − x + 1 F1 (x) =



x2

(3.95)

3.14 Integrali di funzioni irrazionali

178

Calcoliamo separatamente i tre integrali (3.95). Per la (3.46): √

√ 1 x2 − x + 1 + ln 2x − 1 + 2 x2 − x + 1 + C1 2 √ 2 F2 (x) = ln 2x − 1 + 2 x − x + 1 + C2 F1 (x) =

F3 (x) si calcola con la sostituzione x = t−1 ottenendo Z dt F (t) = − √ , 2 t −t+1

che a parte il segno, è pari a F2 (x), quindi: √ 2 F3 (t) = − ln 2t − 1 + 2 t − t + 1 + C3 ,

essendo t = x−1 . Ripristinando la variabile x: √ 2 F3 (x) = ln |x| − ln 2 − x + 2 x − x + 1 + C3 Quindi:

√ 3 x2 − x + 1 + ln 2 − x + 2 x2 − x + 1 + 2 √ 2 + ln |x| − ln 2 − x + 2 x − x + 1 + const

F (x) =

3.14.9



Integrali del “tipo 4”

Hanno un espressione generale: Im,n,p (x) = essendo m, n, p ∈ Q.

Z

xm (a + bxn )p dx,

(3.96)

Proposizione (Teorema di Cebyscev) 19. L’integrale (3.96) è esprimibile attraverso una combinazione finita di funzioni elementari se e solo se è verificata una delle condizioni: 1. p ∈ Z 2.

m+1 n

∈Z

3.14 Integrali di funzioni irrazionali 3.

m+1 n

179

 +p ∈Z

Nel caso 1 il cambio di variabile è: x = tq ,

(3.97)

essendo q il m.c.m. dei denominatori di m e n. Nel caso 2 il cambio di variabile è: a + bxn = tk ,

(3.98)

essendo k il denominatore di p. Nel caso 3 il cambio di variabile è: ax−n + b = tk

(3.99)

Dimostrazione. Omessa. Esempio: I− 1 , 1 , 1

2 4 3

Qui è:

√ Z p 3 1+ 4x √ dx (x) = x

1 1 1 m = − ,n = ,p = ,a = b = 1 2 4 3 Siamo nel caso 2: 3 1 1 + x 4 = t3 =⇒ dx = 12t2 t3 − 1 dt

Ciò implica:

Z

 12 t6 − t3 dt = t7 − 3t4 + const 7

q

q 7 √ √ 4 3 4 1 + x − 3 1 + 4 x + const

I− 1 , 1 , 1 (t) = 12 2 4 3

Ripristinando la variabile x: I− 1 , 1 , 1 2 4 3

3.14.10 1) 4) 7)

Z

Z

Z

12 (x) = 7

Calcolare i seguenti integrali

3

2 −3/2

x (1 + 2x )

dx

√dx x 3 1+x5



3

x (1 +

√ 3

2

x) dx

Z

xm dx √ 4 1+x4

Z

per m = −1, 4 3) x4 √dx 2) Fm (x) = 1+x2 Z Z dx dx 6) √ √ 5) x2 (2+x √ 3 3 )5/3 4 3 x 1+ x3 Z Z √ √ p 3 √ 4 √ 4 1+ x 3 3 √ dx x 1 + x2 dx 9) 8) x

3.14 Integrali di funzioni irrazionali

3.14.11 1.

R

180

Soluzioni −3/2

x3 (1 + 2x2 )

dx. Qui è m = 3, n = 2, p = − 32 : Z −3/2 I3,2,− 3 (x) = x3 1 + 2x2 dx, 2

quindi:

tdt 1 a + bxn = tk ⇐⇒ 1 + 2x2 = t2 =⇒ dx = √ √ 2 t2 − 1 L’integrale diventa: I3,2,− 3

2

 Z  1 1 1 − 2 dt (t) = 4 t 2 1t +1 = + const 4 t

Ripristinando la variabile x: 1 + x2 + const I3,2,− 3 (x) = √ 2 2 1 + 2x2 2. F−1 (x) =

R

√dx . x 4 1+x4

Qui è

m+1 n

= 0, per cui il cambio di variabile: 1 + x4 = t4 ,

per cui: F−1 (t) =

Z

t2 dt (t4 − 1)

Procedendo per decomposizione in frazioni semplici: 1 t + 1 1 F−1 (t) = ln + arctan t + C 4 t − 1 2

Ripristinando la variabile x: √ √ 1 4 x−4 + 1 + 1 1 4 x−4 + 1 + C F−1 (t) = ln √ arctan + 4 −4 4 x + 1 − 1 2

Procedendo in maniera simile per m = 4: q   √ 4 −4 + 1)3 4 −4 (x 4 √ 1  x + 1 − 1 4 +C + 2 arctan x−4 + 1 + q ln √ F4 (x) = 4 x−4 + 1 + 1 16 3 4 (x−4 + 1) − 1

3.14 Integrali di funzioni irrazionali 3.

R

√dx . x4 1+x2

181

Qui è m = −4, n = 2, p = − 21 I−4,2,− 1 (x) = 2

Z

dx √ , x4 1 + x2

quindi: ax−n + b = tk ⇐⇒ x−2 + 1 = t2 =⇒ dx = −

tdt (t2

− 1)3/2

L’integrale diventa: I1,4,− 1

4

Z (t) = −

 t2 − 1 dt

1 = − t3 + t + const 3

Ripristinando la variabile x: I1,4,− 1 (x) = − 4

4.

R

√dx . x 3 1+x5

q

(1 + x2 )3

+

2 3x √ 1 + x2 = 3x2

Qui è m = −1, n = 5, p = − 13 I−1,5,− 1 (x) = 3

Z



1 + x2 + const x  2x2 − 1 + const

dx , x 1 + x5 √ 3

quindi: a + bxn = tk ⇐⇒ 1 + x5 = t3 =⇒ dx = L’integrale diventa: I−1,5,− 1

3

3 (t) = 5

Z

3 t2 dt 5 (t3 − 1)4/5

tdt −1

t3

Abbiamo quindi ricondotto l’integrale 4 all’integrale di una funzione razionale propria. Riduciamo l’integrando in frazioni semplici: t A Bt + C = + t3 − 1 t − 1 t2 + t + 1

3.14 Integrali di funzioni irrazionali

182

Si ottiene: A+B+0=0 A−B+C =1 A + 0 − C = 0, la cui unica soluzione è: (A, B, C) = (0, 1, 0) Quindi: I−1,5,− 1

3

3 (t) = 5



1 1 ln |t − 1| − 3 3

Z

t−1 t2 + t + 1



+ const

Calcoliamo l’integrale a secondo membro con la (3.45):   Z  √ 2t + 1 1 t−1 2 √ + const = ln t + t + 1 − 3 arctan t2 + t + 1 2 3

5.

Ripristinando la variabile x:  √ 3 5−1 √ 1 + x 1 + 3 arctan r I−1,5,− 1 (x) =  ln 3 5  q √ 3 (1 + x5 )2 + 3 1 + x5 + 1 R

dx . x2 (2+x3 )5/3

 ! 2 1 + x5 + 1  +C √  3 √ 3

Qui è m = −2, n = 3, p = − 53 Z dx I−2,3,− 5 (x) = , 3 x2 (2 + x3 )5/3

quindi: ax−n +b = tk ⇐⇒ 2x−3 +1 = t3 =⇒

x=

21/3 (t3

− 1)

1/3

, dx = −21/3

t2 dt (t3 − 1)4/3

L’integrale diventa: I−2,3,− 5 3

# Z " 3 5/3 2/3 2 1 (t3 − 1) t (t − 1) · 5/3 · 21/3 dt (t) = − 22/3 2 t5 (t2 − 1)4/3 Z 1 t3 − 1 =− dt 4 t3   1 1 −2 =− t+ t +C 4 2

!

3.14 Integrali di funzioni irrazionali

183

Ripristinando la variabile x: I−2,3,− 5

3

6.

R



x3

dx √ 3

1+

√ 4

x3

quindi:

" # 1/3 1 x2 1 (2 + x3 ) + +C (x) = − 4 x 2 (2 + x3 )2/3 4 + 3x3 =− +C 8x (2 + x3 )2/3

. Qui è m = − 32 , n = 43 , p = − 13 Z dx , I− 3 , 3 ,− 1 (x) = √ p √ 3 4 2 4 3 x3 1 + x3

ax−n + b = tk ⇐⇒ x−3/4 + 1 = t3 =⇒ dx = −4

t2 dt (t3 − 1)7/3

L’integrale diventa: I−2,3,− 5

3

Z (t) = −4 tdt

= −2t2 + C

Ripristinando la variabile x: r √ 3

I−2,3,− 5 (x) = −2 3

7.

R√

x (1 +

√ 3

4

2 x3 + 1 √ +C x

2

x) dx. Qui è m = 12 , n = 31 , p = 2 Z √ 2 √ x 1 + 3 x dx, I 1 , 1 ,2 (x) = 2 3

quindi:

x = t6 =⇒ dx = 6t5 dt L’integrale diventa: Z  I 1 , 1 ,2 (t) = 6 t12 + 2t10 + t8 dt 2 3   2 11 1 9 1 13 t + t + t + const =6 13 11 9 Ripristinando la variabile x:   1 13/6 2 11/6 1 3/2 I 1 , 1 ,2 (x) = 6 + const x + x + x 2 3 13 11 9

3.14 Integrali di funzioni irrazionali 8.

184

p √ R√ 4 3 3 x 1 + x2 dx. Qui è m = 13 , n = 32 , p = 41 q Z √ √ 4 3 3 x 1 + x2 dx, I 1 , 2 , 1 (x) = 3 3 4

Abbiamo:

m+1 = 1, n

donde il cambio di variabile è n

k

a+bx = t ⇐⇒ 1+x

2/3

L’integrale diventa:



3/2 1/2  4 3 = t =⇒ x = t − 1 , dx = 6t t − 1 dt 4

4

Z  I 1 , 1 ,2 (t) = 6 t4 t4 − 1 dt 2 3   1 9 1 5 t − t + const =6 9 5 Ripristinando la variabile x: " r # 9 1 r 5 √ √ 14  3 3 4 I 1 , 1 ,2 (x) = 6 1 + x2 − 1 + x2 +C 2 3 9 5 r 3 5 5√ √ x2 − 4 3 4 2 +C 1+ x =6 45 √ 3 R √ 1+ 4 x √ dx. Qui è m = − 12 , n = 41 , p = 31 9. x √ Z p 3 1+ 4x √ dx, I− 1 , 1 , 1 (x) = 2 4 3 x Abbiamo:

m+1 = 2, n

donde il cambio di variabile è n

k

a + bx = t ⇐⇒ 1 + x L’integrale diventa:

1/4



4 3  3 2 = t =⇒ x = t − 1 , dx = 12t t − 1 dt 3

3

Z  I 1 , 1 ,2 (t) = 12 t3 t3 − 1 dt 2 3   1 7 1 4 t − t +C = 12 7 4

3.15 Esercizi riepilogativi sugli integrali di funzioni irrazionali

185

Ripristinando la variabile x:   q 7 1 q 4 √ √ 13 3 4 4 1+ x − +C I 1 , 1 ,2 (x) = 12 1+ x 2 3 7 4 q  √ 7 √ 33 = 1+ 4x 44x−3 +C 7

3.15

Esercizi riepilogativi sugli integrali di funzioni irrazionali

3.15.1 1) 5) 9)

Z



x2 +4x+4+x2 dx x+1 √ x dx 1+x

Z

Z

13) 17)

Calcolare i seguenti integrali

Z

Z

√ dx x2 −x+1 dx (x+1)1/2 +(x+1)1/4 √ 1− 3 x √ dx √ x+ 4 x

2) 6)

Z

Z

10) 14)

18)

Z

Z

Z

dx √ x−2 3 x+4

3)

dx √ √ x(1+ x)

7)

√ dx x x2 +x−1 √dx x2 4−x2 √ 3 √ 1− x+1 √ dx x+1+ 3 x+1

Z

Z

11)

x

Z

−3

4 1/2

(1 + x )

dx √ 3+ x+2 √ dx

2

Z p6+x−x √ 15) 1 + xdx Z dx 19) √x2 −3x+2

dx 4)

Z

dx √ 3 x+x3 Z √ √3x+2 dx 8) 1− 1+ 3x+2 Z √ 4x−x2 12) dx x3

16)

20)

Z

Z

√ x2 1 − xdx √

*** Z 3 +x 21) x2 + 4x + 13dx 22) −x2 − x + 1dx 23) √−xx4 +3x 2 −2 dx Z Z Z √ √ √ 4 x x+2 24) x+√ −x2 − 14x + 17dx dx 26) dx 25) √x+1+x 3 x+2 Z Z Z p x 6x−5 √ dx 28) 27) dx 29) x√x2dx+x+1 x−2 x2 −12x+52 Z



3.15.2

Z



xdx −x2 +x+2

Soluzioni

R √x2 +4x+4+x2 1. I (x) = dx. Si riduce facilmente all’integrale di una funzione x+1 razionale impropria: Z (x + 2) + x2 I (x) = dx x+1  Z  2 dx x+ = x+1 1 = x2 + 2 ln |x + 1| + const 2  R R √ R x, x1/3 dx, cioè è del tipo 1 con (p1 , q1 ) = (1, 3), 2. I (x) = x−2dx 3 x+4 = donde: x = t3

3.15 Esercizi riepilogativi sugli integrali di funzioni irrazionali

186

L’integrale diventa: Z t2 dt I (t) = 3 3 t − 2t + 4 Z t2 dt =3 (t + 2) (t2 − 2t + 2) Riduciamo in frazioni semplici: A Bt + C t2 = + 2 2 (t + 2) (t − 2t + 2) t + 2 t − 2t + 2 cioè: (A + B) t2 + (−2A + 2B + C) t + 2A + 2C = t2 Per il principio di identità dei polinomi: A+B+0=1 −2A + 2B + C = 0 2A + 0 + 2C = 0

(3.100)

Il sistema (3.100) è di Cramer, e risolvendo:   2 3 2 (A, B, C) = , ,− 5 5 5 Quindi:

  Z dt 1 2 + J (t) , I (t) = 3 5 t+2 5

essendo: J (t) = che si calcola con la (3.45): J (t) = Quindi:

Z

t2

3t − 2 dt, − 2t + 2

 3 ln t2 − 2t + 2 + arctan (t − 1) + const 2

  1 3 2 2 ln |t + 2| + ln t − 2t + 2 + arctan (t − 1) + const I (t) = 3 5 10 5 

Ripristinando la variabile x:    3 √ √  √ √ 3 3 3 3 3 2 I (x) = x − 2 x + 2 + arctan x − 1 +const 2 ln x + 2 + ln 5 2

3.15 Esercizi riepilogativi sugli integrali di funzioni irrazionali 3.

R

1/2

x−3 (1 + x4 )

dx. È del tipo 4 con m = −3, n = 4, p = Z 1/2 I−3,4, 1 (x) = x−3 1 + x4 dx,

187

1 2

2

Abbiamo:

m+1 + p = 0, n

donde il cambio di variabile è ax

−n

−4

k

2

+b = t ⇐⇒ x +1 = t =⇒



L’integrale diventa:

Z 1 t2 (t) = − dt 2 t2 − 1   Z dt 1 dt =− t+ 2 t2 − 1   1 1 t + 1 =− t + ln +C 2 2 t − 1

I3,4, 1

2

4.

 1/4 −5/4 1 2 x= t −1 , dx = − t t − 1 dt 2 2

Ripristinando la variabile x: ! √ √ x4 + 1 1 x4 + 1 + x2 1 − ln √ I−3,4, 1 (x) = − +C 2 2 x2 2 x4 + 1 − x2 R

dx √ 3 x+x3

=

R

−1/3

x−1/3 (1 + x2 )

dx. È del tipo 4 con m = − 31 , n = 2, p = − 31

I− 1 ,2,− 1 (x) = 3

3

Abbiamo:

Z

 x−1/3 1 + x2 dx,

m+1 + p = 0, n

donde il cambio di variabile è ax

−n

k

−2

3

+b = t ⇐⇒ x +1 = t =⇒

L’integrale diventa: I− 1 ,2,− 1 3

3



 −3/2 −1/2 3 2 3 dt x= t −1 , dx = − t t − 1 2

3 (t) = − 2

3

Z

t3

t dt −1

3.15 Esercizi riepilogativi sugli integrali di funzioni irrazionali

188

Riduciamo in frazioni semplici: t A Bt + C = + t3 − 1 t − 1 t2 + t + 1 A (t2 + t + 1) + (Bt + C) (t − 1) = t3 − 1 (A + B) t2 + (A − B + C) t + A − C = t3 − 1 Per il principio di identità dei polinomi: (3.101)

A+B+0=0 A−B+C =1 A+0−C =0

Il sistema (3.101) è di Cramer, quindi risolviamolo con l’omonima regola: 1 1 0 ∆ = 1 −1 1 = 3 1 0 −1 0 1 0 ∆A = 1 −1 1 = 1 0 0 −1 1 1 0 ∆B = 1 −1 1 = −1 1 0 0 1 1 0 ∆C = 1 −1 1 = 1 1 0 0

Quindi:

∆A 1 = ∆ 3 ∆B 1 B= =− ∆ 3 1 ∆C = C= ∆ 3 A=

L’integrale diventa: I− 1 ,2,− 1 3

3

1 (t) = − 2



ln |t − 1| + C1 −

Z

t−1 2 t +t+1



3.15 Esercizi riepilogativi sugli integrali di funzioni irrazionali

189

L’integrale a secondo membro si calcola con la (3.45), ottenendo:   Z  √ t−1 2t + 1 1 2 √ + C2 = ln t + t + 1 − 3 arctan t2 + t + 1 2 3 Sostituendo nella precedente: I− 1 ,2,− 1 3

3

√    1 1 2t + 1 3 2 √ (t) = − ln |t − 1| + ln t + t + 1 − + const arctan 2 4 2 3

Ripristinando la variabile x:  1 q p 1 √ 3 3 2 −2 −2 −2 I− 1 ,2,− 1 (x) = − ln x + 1 − 1 + ln (x + 1) + (x + 1) + 1 3 3 2 4 ! √ √ 2 3 x−2 + 1 + 1 3 √ +C arctan − 2 3 5. I (x) =

R



x dx. 1+x

Il cambio di variabile è √ x = t =⇒ dx = 2tdt

donde: Z t2 dt I (t) = 2 2 t +1 Z 2 t +1−1 =2 dt t2 + 1  Z Z dt =2 dt − t2 + 1 = 2 (t − arctan t) + const Ripristinando la variabile x: I (x) = 2 6. I (x) =

R



dx √ . x(1+ x)



√  x − arctan x + const

Il cambio di variabile è √

x = t =⇒ dx = 2tdt

donde: Z I (t) = 2

dt t+1 = 2 ln |t + 1| + const Ripristinando la variabile x:

√ I (x) = 2 ln x + 1 + const

3.15 Esercizi riepilogativi sugli integrali di funzioni irrazionali 7. I (x) =

R

dx √ . 3+ x+2

Eseguiamo il cambio di variabile: √

x + 2 = t =⇒ dx = 2tdt

donde: Z tdt I (t) = 2 t+3 = 2 (t − 3 ln |t + 3|) + const Ripristinando la variabile x: √ i h√ x + 2 + 3 + const I (x) = 2 x + 2 − 3 ln 8. I (x) =

R

√ 1−√3x+2 dx. 1+ 3x+2



Eseguiamo il cambio di variabile:

3x + 2 = t =⇒



  1 2 2 x= t − 2 , dx = tdt 3 3

donde: Z 2 t − t2 dt I (t) = 3 1+t  Z  2 2 = dt + C1 −t + 2 − 3 1+t   1 2 2 − t + 2t − 2 ln |1 + t| + C1 = 3 2 Ripristinando la variabile x:  i √ 4 h√ I (x) = −x + 3x + 2 − ln 1 + 3x + 2 + C 3 9. I (x) =

10. I (x) =

R

√ dx . x2 −x+1

R

√ dx . x x2 +x−1

Si calcola direttamente con la (3.46):

√ I (x) = ln 2x − 1 + 2 x2 − x + 1 + const Eseguiamo il cambio di variabile: t=

1 , x

190

3.15 Esercizi riepilogativi sugli integrali di funzioni irrazionali ottenendo:

Z I (t) = − √

con la (3.46):

−t2

dt , +t+1

1 − 2t I (t) = arcsin √ + const 5

Ripristinando la variabile x: I (x) = arcsin 11. I (x) =

12. I (x) =

R

√ dx . 6+x−x2

R



x−2 √ + const x 5

Si calcola direttamente con la (3.46): 1 − 2x + const 5 2x − 1 = arcsin + const 5

I (x) = − arcsin

4x−x2 dx. x3

Può essere scritto come: Z √ I (x) = x−5/2 4 − xdx Z = x−m (a + bxn )p dx

Risulta:

m+1 + p = −1, n

per cui il cambio di variabile è: 4x−1 − 1 = t2 , da cui: x=

t2

4 8tdt , dx = − 2 +1 (t + 1)

L’integrale diventa: I (t) =

Z

5/2 4−5/2 t2 + 1

(−8) · t 2 dt (t2 + 1) (t2 + 1) Z −5/2 = −16 · 4 t2 dt 2t

1/2

=

16 −5/2 3 ·4 t + const 3

191

3.15 Esercizi riepilogativi sugli integrali di funzioni irrazionali

192

Ripristinando la variabile x: I (x) = −

q (4x − x2 )3 6x3

+ const

i R R h 1/2 1/4 13. I (x) = (x+1)1/2dx = R (x + 1) , (x + 1) , quindi è del tipo 1 +(x+1)1/4 con q1 = 2, q2 = 4, donde è k = 4. Il cambio di variabile è: (x + 1)4 = t L’integrale diventa: Z 2 t dt I (t) = 4 t+1  Z  1 dt t−1+ =4 t+1   1 2 =t t − t + ln |t + 1| + const 2 Rirpristinando la variabile x:   1 1/4 1/2 1/4 (x + 1) − (x + 1) + ln (x + 1) + 1 + const I (x) = 4 2

14. I (x) =

R

√dx . x2 4−x2

Eseguiamo il cambio di variabile: x=

2 t

L’integrale diventa: Z tdt 1 √ I (t) = − 4 t2 − 1 1√ 2 =− t − 1 + const 4 Ripristinando la variabile x: I (x) = −



4 − x2 + const 4x

3.15 Esercizi riepilogativi sugli integrali di funzioni irrazionali

193

Rp √ 15. I (x) = 1 + xdx. È del tipo 4 con m = 0, n = 1/2, p = 1/2, per cui il cambio di variabile è: 1 + x1/2 = t2 , differenziando:

 dx = 4t t2 − 1 dt

L’integrale diventa:

Z  I (t) = 4 t2 t2 − 1 dt   1 5 1 3 =4 t − t + const 5 3 Ripristinando la variabile x:     1 1 1/2 5/2 1/2 3/2 I (x) = 4 + const 1+x − 1+x 5 3 q  √ 3 √ 4 1 + x 3 x − 2 + const = 15

16. I (x) =

R

√ x2 1 − xdx. È del tipo 4 con m = 2, n = 1, p = 1/2, per cui: √ 1 − x = t2

L’integrale diventa: Z  t6 − 2t4 + t2 dt I (t) = −2   1 7 2 5 1 3 t − t + t + const = −2 7 5 3 Ripristinando la variabile x: q  2 (1 − x)3 15x2 + 12x + 8 + const I (x) = − 105 17. I (x) =

R

√ 1− 3 x √ dx √ x+ 4 x

=

R

 R x1/3 , x1/2 , x1/4 , quindi il cambio di variabile è: x = t12

3.15 Esercizi riepilogativi sugli integrali di funzioni irrazionali

194

L’integrale diventa: Z 8 t (1 − t4 ) dt I (t) = 12 t3 + 1  Z  t−1 2 3 5 6 9 1−t −t +t +t −t + 2 = 12 dt t −t+1   1 3 1 4 1 6 1 6 1 7 1 10 = 12 t − t − t + t + t + t − t + J (t) , 3 4 6 6 7 10 essendo: J (t) =

Z

t−1 t2 − t + 1

 1 1 = ln t2 − t + 1 − √ arctan 2 3



2t − 1 √ 3



+ C1

Sostituendo nella precedente e ripristinando la variabile x: √ √ √ √ √ 12 12 x7 I (x) = 12 12 x − 4 4 x − 3 3 x + 2 x + 7 √ 12  12 √ √ x−1 6√ 2 6 6 12 √ x5 + 6 ln x − x + 1 − √ arctan − +C 5 3 3 i R 1− √ R h 3 1/2 1/3 x+1 √ √ 18. I (x) = dx, quindi il cambio dx = R (x + 1) , (x + 1) x+1+ 3 x+1 di variabile è: x + 1 = t6 L’integrale diventa: Z I (t) = 6

3

4

t −t



dt = 6



1 4 1 5 t − t 4 5



+ const

Rirpristinando la variabile x: q q 33 66 2 I (x) = (x + 1) − (x + 1)5 + const 2 5 R dx . Si calcola direttamente con la (3.46), ottenendo: 19. I (x) = √x2 −3x+2 √ 2 I (x) = ln 2x − 3 + 2 x − 3x + 2 20. I (x) =

R



xdx . −x2 +x+2

I (x) =

Si calcola direttamente con la (3.46), ottenendo: 1 1 − 2x √ 2 arcsin − −x + x + 2 + const 2 3

3.15 Esercizi riepilogativi sugli integrali di funzioni irrazionali 21. I (x) = do:

R√

195

x2 + 4x + 13dx. Si calcola direttamente con la (3.48), ottenen-

√ 9 x + 2√ 2 x + 4x + 13 + ln x + 2 + x2 + 4x + 13 + const 2 2 R√ 22. I (x) = −x2 − x + 1dx. Si calcola direttamente con la (3.48), ottenendo: 5 2x + 1 √ 2 2x + 1 −x − x + 1 + arcsin √ I (x) = + const 4 8 5 R 3 +x 23. I (x) = √−xx4 +3x 2 −2 dx. Può essere scritto come: Z  x2 + 1 1 √ I (x) = d x2 , 2 −x4 + 3x2 − 2 I (x) =

per cui è conveniente eseguire il cambio di variabile: y = x2 , donde:

1 I (y) = 2

Z

y+1 p dy, −y 2 + 3y − 2

che si calcola con la (3.46), ottenendo: I (y) =

5 1p 2 −y + 3y − 2 + const arcsin (2y − 3) − 4 2

Ripristinando la variabile x:

 1√ 5 −x4 + 3x2 − 2 + const arcsin 2x2 − 3 − 4 2 i R x+ √ R h 4 1/3 1/4 x+2 √ 24. I (x) = dx = R x, (x + 2) , (x + 2) dx, per cui il cambio 3 x+2 di variabile è: x + 2 = t12 I (x) =

Differenziando rispetto a t: dx = 12t11 dt L’integrale diventa: Z  t19 + t10 − 2t7 dt I (t) = 12   20 t11 1 8 t + − t + const = 12 20 11 4

3.15 Esercizi riepilogativi sugli integrali di funzioni irrazionali

196

Ripristinando la variabile x: 2/3

I (x) = 12 (x + 2) 25. I (x) =

R





x dx. x+1+x



 1 1 1/11 + const (x − 1) + (x + 2) 20 11

Indicando con f (x) la funzione integranda: √ x x+1−1 f (x) = √ ·√ x+1+1 x+1−1  √ √ x x+1−1 = x r x+1 1 −√ , = x x √

donde: Z r

Z x+1 dx I (x) = dx − √ x x √ = J (x) − 2 x + const, essendo: J (x) =

Z r

x+1 dx x

Per il calcolo di J (x) eseguiamo il cambio di variabile: r x+1 = t, x da cui: x= Quindi:

t2

2tdt 1 , dx = − 2 −1 (t − 1)2

Z J (t) = −2

t2 dt (t2 − 1)2

Riduciamo l’integrando in frazioni semplici: t2 t2 = (t2 − 1)2 (t − 1)2 (t2 + 1)   1 1 1 1 1 + + − = 4 (t − 1)2 t − 1 (t + 1)2 t + 1

(3.102)

3.15 Esercizi riepilogativi sugli integrali di funzioni irrazionali Integrando: 1 J (t) = 2

  t + 1 2t + ln + const t − 1 t2 − 1

Ripristinando la variabile x e sostituendo nella (3.102): r √ √ √ 1 x + 1 + x x+1 − 2 x + ln √ I (x) = x √ + const x 2 x + 1 − x

26. I (x) =

R√

−x2 − 14x + 17dx. Si calcola direttamente con la (3.48):

I (x) =

27. I (x) =

R



x + 7√ 2 x+7 −x − 14x + 17 + 33 arcsin √ + const 2 66

6x−5 dx. x2 −12x+52

Si calcola direttamente con la (3.46):

√ √ x − 6 + x2 − 12x + 52 + const I (x) = 6 x2 − 12x + 52 + 31 ln 4

28. I (x) =

Rp

x dx. x−2

Eseguiamo il cambio di variabile: x = t2 x−2

donde: x= L’integrale diventa:

4t 2t2 , dx = − 2 dt 2 t −1 t −1

Z I (t) = −4

t2 dt (t2 − 1)2

Sviluppiamo l’integrando in frazioni semplici:   1 t2 1 1 1 1 + = + − 4 t − 1 (t − 1)2 (t + 1)2 t + 1 (t2 − 1)2 Da cui l’integrale: t + 1 2t + const I (t) = 2 + ln t −1 t − 1

Ripristinando la variabile x: I (x) =

p

√ √ x + x − 2 + const √ x (x − 2) + ln √ x − x − 2

197

3.16 Integrali di funzioni trigonometriche 29. I (x) =

R

√ dx . x x2 +x+1

198

Si calcola con la (3.47). Il cambio di variabile è: ξ=

1 , x

da cui: Z dξ I (ξ) = − p ξ2 + ξ + 1  p √  = − ln 2ξ + 1 + 2 ξ 2 + ξ + 1 − ln 3 + const

√ Ripristinando la variabile x e incorporando ln 3 nella costante di integrazione: x + const √ I (x) = ln x + 2 + 2 x2 + x + 1

3.16

3.16.1

Integrali di funzioni trigonometriche Integrali del “tipo 1”

Hanno la seguente espressione: In1 ,n2 (x) =

Z

(sin x)n1 (cos x)n2 dx,

(3.103)

essendo n1 , n2 ∈ Z. Consideriamo ni entrambi positivi. Lo schema di calcolo per In1 ,n2 (x) è legato alla partità di n1 , n2 . Più precisamente, il caso più immediato è quello in cui ni è dispari. Senza perdita di generalità, supponiamo che n1 sia dispari: ∃k ∈ N : n1 = 2k + 1,

donde:

Z

(sin x)2k (cos x)n2 sin xdx Z k 1 − cos2 x (cos x)n2 d (cos x) =−

In1 (k),n2 (x) =

Eseguiamo il cambio di variabile:

y = cos x Quindi:

(3.104)

3.16 Integrali di funzioni trigonometriche

Z In1 (k),n2 (y) = −

1 − y2

che si risolve facilmente. Ad esempio:

199

k

y n2 dy,

Z

sin3 x cos4 xdx Z  1 − cos2 x cos4 xd (cos x) =−

I (x) =

Il cambio di variabile (3.104):

Z I (y) = −

 1 − y 2 y 4 dy

1 1 = − y 5 + y 7 + const 5 7

Ripristinando la variabile x: 1 1 I (x) = − cos5 x + cos7 x + const 5 7 Nel caso n2 = 2k + 1, il cambio di variabile è y = sin x: In1 ,n2 (k) (x) = = Cambiando la variabile:

Z

Z

(sin x)n1 (cos x)2k cos xdx (sin x)n1 1 − sin2 x

In1 ,n2 (k) (y) = che si risolve facilmente.

Z

y n1 1 − y 2

k

k

d (sin x)

dy,

*** Se n1 e n2 sono entrambi pari, si cerca di trasformare l’integrando utilizzando le formule trigonometriche: 1 (1 − cos 2x) 2 1 cos2 x = (1 + cos 2x) 2 1 sin x cos x = sin 2x 2 sin2 x =

3.16 Integrali di funzioni trigonometriche

200

Ad esempio: I (x) = = = = = = =

Z

Z

cos2 3x sin4 3xdx (cos 3x sin 3x)2 sin2 3xdx

Z

sin2 6x 1 − cos 6x · dx 4 2 Z  1 sin2 6x − sin2 6x cos 6x dx 8  Z  1 1 − cos 12x 2 − sin 6x cos 6x dx 2 2   1 x 1 1 3 − sin 12x − sin 6x + C 2 2 24 18  1 36x − 3 sin 12x − 4 sin3 6x + C 576 ***

Consideriamo ora il caso in cui n1 , n2 ≤ 0 : (n1 , n2 ) 6= (0, 0). L’integrale (3.103) si scrive: Z dx In1 ,n2 (x) = (3.105) (sin x)|n1 | (cos x)|n2 | Ricordiamo le formule: 1 1 =1+ 2 tan2 x sin x 1 = 1 + tan2 x, 2 cos x

(3.106)

da cui: 1

(sin x)|n1 | = 1+ dx (cos x)|n2 |

1 tan2 x

= 1 + tan2 x

Sostituendo in (3.105): In1 ,n2 (x) =

Z 

 |n21 |

1 1+ tan2 x

 |n21 |

 |n22|−2

d (tan x)

1 + tan2 x

 |n22|−2

d (tan x)

3.16 Integrali di funzioni trigonometriche

201

Eseguiamo il cambio di variabile: y = tan x Quindi: In1 ,n2 (y) =

|n1 |+|n2 | −1

Z

(y 2 + 1) 2 y |n1 |

dy

(3.107)

Osservazione. La (3.107) è valida anche se n1 , n2 sono numeri razionali. Alcuni esempi: I (x) = Qui è:

Z

dx cos4 x

|n1 | = 0, |n2 | = 4,

donde la (3.107):

I (y) = =

Z

 y 2 + 1 dy

y3 +y+C 3

Ripristinando la variabile x: I (x) =

1 tan3 x + tan x + C 3

Consideriamo ora: I (x) = Conviene riscrivere I (x) nella forma: 1 I (x) = 8 Poniamo:

Z

Z

dx sin3 x dx cos3

sin3 x2

ξ=

x 2

x 2

Ciò implica: 1 I (ξ) = 4

Z

dξ sin ξ cos3 ξ 3

3.16 Integrali di funzioni trigonometriche

202

Qui è: |n1 | = |n2 | = 3,

per cui:

Z 2 1 (y 2 + 1) I (y) = dy 4 y3  Z Z 2y 2 + 1 1 dy ydy + = 4 y3   1 y2 1 = + 2 ln |y| − 2 + C 4 2 2y Ripristinando la variabile x:

3.16.1.1

 x i 1h 2x x I (x) = + 4 ln tan − cot2 tan +C 8 2 2 2 Z Z dx , (cosdxx)n (sin x)n

Questi integrali possono essere calcolati attraverso un procedimento ricorsivo (qui è n ≥ 2). Iniziamo con il primo integrale: Z

dx (sin x)n Z sin2 x + cos2 x dx = (sin x)n = Fn−2 (x) + Hn (x) ,

Fn (x) =

essendo: Hn (x) = Osserviamo che

Quindi:

Z

cos2 x dx (sin x)n

  cos x d 1 n−1 = − (n − 1) dx (sin x) (sin x)n

3.16 Integrali di funzioni trigonometriche

Hn (x) =

Z

cos x

1 =− n−1

cos x dx (sin x)n  Z cos xd

203

1 (sin x)n−1



Eseguendo un’integrazione per parti nell’ultimo integrale: Hn (x) = − Finalmente:

cos x 1 1 Fn−2 (x) n−1 − n − 1 (sin x) n−1

  1 cos x Fn (x) = (n − 2) Fn−2 (x) − n−1 (sin x)n−1

(3.108)

Attraverso la formula ricorrente (3.108) è possibile determinare Fn (x) per assegnati valori di n (ved. Appendice). Passiamo all’integrale contenente il coseno. Poniamo: Z

dx (cos x)n Z sin2 x + cos2 x dx = (cos x)n = Gn−2 (x) + Kn (x) ,

Gn (x) =

essendo: Kn (x) = Osserviamo che

Quindi:

Z

sin2 x dx (cos x)n

  sin x d 1 n−1 = (n − 1) dx (cos x) (cos x)n

Z

sin x dx sin x (cos x)n   Z 1 1 sin xd = n−1 (cos x)n−1

Kn (x) =

Eseguendo un’integrazione per parti nell’ultimo integrale:

3.16 Integrali di funzioni trigonometriche

Kn (x) = Finalmente

204

sin x 1 1 Gn−2 (x) n−1 − n − 1 (cos x) n−1

  1 sin x Gn (x) = (n − 2) Gn−2 (x) + n−1 (cos x)n−1

(3.109)

Attraverso la formula ricorrente (3.109) è possibile determinare Gn (x) per assegnati valori di n (ved. Appendice). Osservazione. Le (3.108)-(3.109) sono inapplicabili per n = 1. A tale valore corrispondono due integrali notevoli:  x  F1 (x) = ln tan +C  x π2  + G1 (x) = ln tan +C 2 4

3.16.2

Integrali del “tipo 2”

Per definizione: In (x) =

Z

(tan x)n dx,

(3.110)

essendo n ∈ N. Per n = 0, 1 il calcolo è immediato:

Per n ≥ 2:

I0 (x) = x + C I1 (x) = − ln |cos x| + C  1 = ln 1 + tan2 x + C 2 In (x) =

Z

(tan x)n−2 tan2 xdx

(3.111)

Sostituendo la seconda delle (3.106) nella (3.111):   Z 1 n−2 − 1 dx In (x) = (tan x) cos2 x Cioè:

In (x) =

1 (tan x)n−1 − In−2 (x) n−1

(3.112)

3.16 Integrali di funzioni trigonometriche

205

Procedendo in maniera analoga per l’integrale: Z Jn (x) = (cot x)n dx, si giunge:

1 (cot x)n−1 − Jn−2 (x) (3.113) 1−n In Appendice 5 sono esplicitati gli integrali In (x), Jn (x) per alcuni valori di n. Jn (x) =

3.16.3 1) 5) 9)

Z

cos xdx cos5 x dx sin3 x

Z

17) 21)

2)

3

Z

13)

Esercizi

2

6) 4

sin x cos xdx

Z

Z

10)

Z

cos2 x dx sin6 x

14)

sin( ) dx sin x cos x

18)

Z

cos3 x dx sin4 x

Z

x+ π4

3.16.4

22)

Z

Z

Z

Z

5

sin xdx 4

sin xdx 6

cos 3xdx

3) 7)

Z

Z

11)

dx sin2 x cos4 x

15)

dx sin5 x

19) 2

2

x sin x dx

23)

Z

Z

Z

Z

2

3

sin x cos xdx 4

cos xdx

4) 8)

R

cos3 xdx =

R

cos2 x cos xdx =

= sin x − 31 sin3 x + C R 2. I (x) = sin5 xdx. Abbiamo:

R

Z

dx sin4 x

12)

dx sin5 x cos3 x

16)

tan2 5xdx

20)

5

√ 3

sin x cos xdx

24)

Soluzioni

1. I (x) =

Z

 1 − sin2 x d (sin x)

Z

I (x) = sin4 x sin xdx Z 2 1 − cos2 x d (cos x) =−

Il cambio di variabile y = cos x implica: Z 2 1 − y 2 dy I (y) = −   2 3 1 5 =− y− y + y +C 3 5

Z

sin3 x2 cos5 x2 dx sin2 x cos2 xdx dx cos6 x

Z

sin

Z



Z

dx cos3

x 2

x 2

dx cos5 4x dx sin x cos3 x

3.16 Integrali di funzioni trigonometriche Ripristinando la variabile x: I (x) = 3. I (x) =

R

2 1 cos3 x − cos x − cos5 x + C 3 5

sin2 x cos3 dx. Abbiamo: Z I (x) = sin2 x cos2 x cos xdx Z  = sin2 x 1 − sin2 x d (sin x)

Il cambio di variabile y = sin x implica: Z  I (y) = y 2 1 − y 2 dy 1 1 = y3 − y5 + C 3 5

Ripristinando la variabile x: I (x) = 4. I (x) =

R

1 3 1 sin x − sin5 x + C 3 5

sin3 x2 cos5 x2 dx. Poniamo: y=

x 2

Quindi: Z I (ξ) = 2 sin3 y cos5 ydy Z  = 2 sin3 y 1 − sin2 y d (sin y)

Il cambio di variabile ξ = sin y implica: Z 2 I (ξ) = ξ 3 1 − ξ 2 dξ   1 8 1 6 1 4 ξ − ξ + ξ +C =2 8 3 4

Ripristinando la variabile x: x x  x i 1 h I (x) = 3 sin8 − 8 sin6 + 6 sin4 +C 12 2 2 2

206

3.16 Integrali di funzioni trigonometriche 5. I (x) =

R

cos5 x dx. sin3 x

207

Abbiamo: Z

cos4 x d (sin x) sin3 x 2 Z 1 − sin2 x d (sin x) = sin3 x

I (x) =

Ponendo y = sin x: 2

Z

(1 − y 2 ) dy I (y) = y3  Z  1 2 y − + 3 dy = y y 1 1 = y 2 − 2 ln |y| − 2 + C 2 2y Ripristinando la variabile x: I (x) = 6. I (x) =

R

1 2 1 +C sin x − 2 ln |sin x| − 2 2 sin2 x

sin4 xdx. Abbiamo: sin4 x =

Sostituendo: I (x) = essendo: J (x) =

Z

1 (1 − cos 2x)2 4

1 [x − sin 2x + J (x)] , 4

1 cos 2xdx = 2 2

Z

cos2 2xd (2x)

Dalla seconda delle (3.5): J (x) = Quindi: I (x) =

1 (4x + sin 4x) + const 8

1 (12x − 8 sin 2x + sin 4x) + const 32

3.16 Integrali di funzioni trigonometriche 7. I (x) =

R

cos4 xdx. Abbiamo: cos4 x =

Sostituendo:

essendo: J (x) =

Z

cos2 2xdx =

Quindi: I (x) = R

1 (1 + cos 2x)2 4

1 [x + sin 2x + J (x)] , 4

I (x) =

8. I (x) =

208

1 (4x + sin 4x) + const 8

1 (12x + 8 sin 2x + sin 4x) + C 32

sin2 x cos2 xdx. Abbiamo: sin2 x cos2 x =

donde:

1 I (x) = 8

Z

1 2 sin 2x, 4

sin2 2xd (2x)

Dalla prima delle (3.5): I (x) = 9. I (x) =

R

Z

1 (4x − sin 4x) + C 32

sin2 x cos4 xdx. Risulta: Z

sin2 2x 1 · (1 + cos 2x) dx sin x cos xdx = 4 2 Z  Z 1 2 2 = sin 2xdx + sin 2x cos 2xdx 8 2

4

Poniamo: I1 (x) =

Z

2

sin 2xdx, I2 (x) =

Z

sin2 2x cos 2xdx

Calcoliamo I1 (x): 1 I1 (ξ) = 2

Z

sin2 ξdξ,

con ξ = 2x

3.16 Integrali di funzioni trigonometriche

209

Quindi:   Z 1 1 1 I1 (ξ) = ξ − sin 2ξ + C1 (1 − cos 2ξ) = 4 4 2 1 1 I1 (x) = x − sin 4x + C1 2 8 Calcoliamo I2 (x): 1 I2 (x) = 2

Z

da cui: R

1 3 sin 2x + C2 6

 1 12x − 3 sin 4x + 4 sin3 2x 192

I (x) = 10. I (x) =

sin2 2xd (sin 2x) =

cos6 3xdx. Poniamo y = 3x: Z 1 cos6 ydy I (y) = 3

Sviluppiamo l’integrando: 1 (1 + cos 2y)3 8  1 cos3 2y + 3 cos2 2y + 3 cos 2y + 1 = 8

cos6 y =

Quindi:

  1 3 ′ I (y) = y + sin 2y + 3F1 (y) + F2 (y) + C , 24 2

essendo: F1 (y) = =

Z

cos2 (2y) dy

y 1 + sin 4y + C1 Z2 8

cos3 (2y) dy   1 3 1 sin 2y − sin 2y + C2 = 2 3

F2 (y) =

Semplificando e ripristinando la variabile x: I (x) =

 1  48 sin 6x + 180x + 9 sin 12x − 4 sin3 6x + C 576

3.16 Integrali di funzioni trigonometriche

210

R 11. I (x) = sindx4 x . Si calcola attraverso una formula di ricorrenza (ved. Appendice). Alternativamente, procediamo nel seguente modo: Z 1 I (x) = − d (cot x) sin2 x Dalla prima delle (3.106): 1 = 1 + cot2 x, 2 sin x donde: Z I (y) = −

 1 + y 2 dy

1 = −y − y 3 + C, 3

essendo y = cot x. I (x) = − cot x − 12. I (x) =

R

dx . cos6 x

1 cot3 x + C 3

Abbiamo: I (x) =

Z

1 d (tan x) cos4 x

Dalla seconda delle (3.106): 2 1 = 1 + tan2 x , 4 cos x

donde:

I (y) =

Z

1 + y2

2

dy

1 2 = y 5 + y 3 + y + C, 5 3 essendo y = tan x. I (x) =

2 1 tan5 x + tan3 x + tan x + C 5 3

3.16 Integrali di funzioni trigonometriche 13. I (x) =

R

cos2 x dx. sin6 x

211

Abbiamo: Z cos2 x I (x) = − d (cot x) sin4 x Z  cos x 2 1 d (cot x) =− sin x sin2 x

Tenendo conto della prima delle (3.106): Z  y 2 + y 4 dy I (y) = −

1 1 = − y 3 − y 5 + C, 3 5

essendo y = cot x. 1 1 I (x) = − cot3 x − cot5 x + C 3 5 14. I (x) =

15. I (x) =

R

dx . sin2 x cos4 x

R

dx . sin5 x cos3 x

R 16. I (x) = variabile:

Applicando la (3.107): 2

Z

(y 2 + 1) dy I (y) = y2  Z  1 2 y + 2 + 2 dy = y 1 1 = y 3 − + 2y + C 3 y 1 1 I (x) = tan3 x − + 2 tan x + C 3 tan x Applicando la (3.107): Z

3

(y 2 + 1) dy I (y) = y5  Z  3 3 1 y + + 3 + 5 dy = y y y 3 1 1 = y + 3 ln |y| − 2 − 4 + C 2 2y 4y 1 3 1 I (x) = tan2 x + 3 ln |tan x| − − +C 2 2 tan2 x 4 tan4 x sin

dx cos3

x 2

x 2

. Prima di applicare la (3.107) eseguiamo il cambio di ξ=

x , 2

3.16 Integrali di funzioni trigonometriche per cui:

Z I (ξ) = 2

212

dξ sin ξ cos3 ξ

La (3.107) si scrive: Z

(y 2 + 1) dy y  x  x = 2 ln tan +C + tan2 2 2

I (y) =

R sin(x+ π ) 17. I (x) = sin x cos4x dx. Sviluppando il numeratore con le formule di addizione degli archi: √ Z 2 sin x + cos x I (x) = dx 2 sin x cos x √ Z  Z 2 dx dx + = 2 cos x sin x Gli integrali a secondo membro dell’equazione precedente, compongono una coppia di integrali notevoli dati dalla (2.4) che qui riscriviamo: Z 1 dx = ln − cot x + C1 sin x sin x Z 1 dx = ln + tan x + C1 cos x cos x

Quindi:

18. I (x) =

R

√   1 1 2 I (x) = + tan x + ln − cot x + C ln 2 cos x sin x dx . sin5 x

Abbiamo:

Z I (x) = −

1 + cot2 x

3/2

d (cot x)

Con l’usuale cambio di variabile y = cot x: Z q I (y) = − (1 + y 2 )3 dy

3.16 Integrali di funzioni trigonometriche

213

Tale integrale può essere risolto ponendo y = cosh t: Z I (t) = − sin4 tdt Utilizzando la nota relazione: sin2 t =

1 (cosh 2t − 1) 2

si ha: Z 1 (cosh 2t − 1)2 I (t) = − 4 Z  1 2 =− cosh (2t) dt − sinh (2t) + t + C 4 Calcoliamo a parte l’integrale a secondo membro: Z  Z 1 2 (cosh (4t) dt + t)1 cosh (2t) dt = 2 t 1 = sinh (4t) + + C1 8 2 Quindi: I (t) = −

1 1 3 sinh (4t) + sinh (2t) − t + C, 32 4 8

con t = arccosh(cot x). R 19. I (x) = tan2 5xdx. Poniamo:

ξ = 5x,

per cui: Z 1 tan2 ξdξ I (ξ) = 5  Z  1 1 − 1 dξ = 5 cos2 ξ 1 = tan 5x − x + C 5 R 20. I (x) = cosdx 5 4x . Dopo aver eseguito il cambio di variabile ξ = 4x conviene applicare la formula ricorsiva esplicitata in Appendice 4, ottenendo:      sin 4x 1 3 π  sin 4x ln tan 2x + + +C I (x) = + 16 2 4 cos2 4x cos4 4x

3.16 Integrali di funzioni trigonometriche 21. I (x) = parti:

R

cos3 x dx sin4 x

=

R

cos x cos2 x sin 4 x dx =

I (x) = − 22. I (x) =

R

R

214

 cos2 xd − 13 sin13 x ; integrando per

cos2 x 2 1 + +C 3 3 sin x 3 sin x

x sin2 x2 dx. Abbiamo: 1 I (x) = 2

Z

sin2 x2 d x2

Eseguendo il cambio di variabile y = x2 : Z 1 sin2 ydy, I (y) = 2



che è un integrale notevole (eq. 3.5): I (y) =

1 (2y − sin 2y) + C 8

Ripristinando la variabile x:  1 2x2 − sin 2x2 + C 8 R R √ 2√ 23. I (x) = sin5 x 3 cos xdx = − (1 − cos2 x) 3 cos xd (cos x). Eseguendo il cambio di variabile y = cos x: Z 2 I (y) = − y 1/3 1 − y 2 dy I (x) =

3 3 3 = y 10/3 − y 16/3 − y 4/3 + C 5 16 4

Quindi: I (x) = −

3√ 3√ 3√ 3 3 3 cos4 x + cos10 x − cos16 x + C 4 5 16

24. Qui è: I (x) =

Z

1

d (tan x) (sin x) (cos x)−1/2 1/4 Z  1 1 d (tan x) 1+ = 2 1/4 tan x (1 + tan2 x) Z = (tan x)−1/2 d (tan x) √ = 2 tan x + C 1/2

3.16 Integrali di funzioni trigonometriche

3.16.5

215

Integrali del “tipo 3”

Sono integrali del tipo: Z

(3.114)

fm (x) fn (x) dx,

essendo fm (x) una funzione sin, cos fm (x) = sin mx, cos mx Si trasforma l’integrando della (3.114) in una somma procedendo poi per decomposizione. A tale scopo si utilizzano le note formule trigonometriche: 1 [sin (m + n) x + sin (m − n) x] 2 1 sin mx sin nx = [cos (m − n) x − cos (m + n) x] 2 1 cos mx cos nx = [cos (m − n) x + cos (m + n) x] 2

(3.115)

sin mx cos nx =

Ad esempio:

I (x) =

Z

sin 4x cos 12xdx Z  Z 1 sin 16xdx − sin 8xdx = 2 1 = (2 cos 8x − cos 16x) + C 32

3.16.6 1)

Esercizi

Z

Z

sin 3x cos 5xdx 2) sin 10x sin 15xdx Z Z   x 2 4) sin 3 cos 3 x dx 5) cos (ax + b) cos (ax − b) dx Z Z 2 7) cos x cos 3xdx 8) sin x sin 2x sin 3xdx

3.16.7

Soluzioni

1. I (x) = =

1 16

R

sin 3x cos 5xdx =

(4 cos 2x − cos 8x) + C

1 2

R

sin 8xdx −

R

sin 2xdx



3) 6)

Z

Z

cos

x 2



cos

x 3



dx

sin (ωt) sin (ωt + φ) dt

3.16 Integrali di funzioni trigonometriche 2. I (x) = 1 50

R

sin 10x sin 15xdx =

1 2

R

cos 5xdx −

216 R

cos 25xdx



(5 sin 5x − sin 25x) + C   R 3. I (x) = cos x2 cos x3 dx = 3 sin 16 x + 35 sin 65 x + C    R 4. I (x) = sin x3 cos 32 x dx = 12 3 cos x3 − cos x + C R 5. I (x) = cos (ax + b) cos (ax − b) dx. Poniamo: α = ax + b, β = ax − b, donde l’integrando: =

cos α cos β =

Quindi: I (x) =

1 [cos (α − β) + cos (α + β)] 2 1 = [cos (2b) + cos (2ax)] 2

1 [2ax cos (2b) + sin (2ax)] + C 4a

R 6. I (t) = sin (ωt) sin (ωt + φ) dt. Poniamo: α = ωt, β = ωt + φ, donde l’integrando: sin α sin β =

1 [cos (α − β) − cos (α + β)] 2 1 = [cos φ − cos (2ωt + φ)] 2

Quindi:   1 1 I (t) = t cos φ − sin (2ωt + φ) + C 2 2ω 1 = [2ωt cos φ − sin (2ωt + φ)] + C 4ω 7. I (x) =

R

cos x cos2 3xdx Z I (x) = cos x cos 3x cos 3xdx Z 1 = (cos 2x + cos 4x) cos 3xdx 2 Z  Z 1 cos 2x cos 3xdx + cos 4x cos 3xdx = 2     1 1 1 1 sin x + sin 5x + C1 + sin x + sin 7x + C2 = 2 5 2 7 1 = (70 sin x + 7 sin 5x + 5 sin 7x) + C 140

3.16 Integrali di funzioni trigonometriche 8. I (x) =

3.16.8

R

217

sin x sin 2x sin 3xdx Z 1 I (x) = (cos x − cos 3x) sin 3xdx 2 Z  Z 1 cos x sin 3xdx − cos 3x sin 3xdx = 2  Z  Z 1 1 1 = (sin 4x + sin 2x) dx − sin 6xdx 2 2 2 1 = (2 cos 6x − 3 cos 4x − 6 cos 2x) + C 48

Integrali del “tipo 4”

Sono integrali del tipo: Z

(3.116)

R (sin x, cos x) dx,

essendo R una funzione razionale. Il cambio di variabile è: x x → t = tan , 2

(3.117)

da cui: R (sin x, cos x) → R e il differenziale: dx =



2t 1 − t2 , 1 + t2 1 + t2

2dt 1 + t2

Ad esempio: I (x) = diventa: I (t) = Ripristinando x:

Z

Z

dx , sin x + cos x + 1

dt = ln |1 + t| + C 1+t

x I (x) = ln 1 + tan + C 2 ***



,

3.16 Integrali di funzioni trigonometriche

218

Se R è una funzione pari: R (− sin x, − cos x) ≡ R (sin x, cos x) ,

il cambio di variabile è

(3.118)

x → t = tan x,

donde:

R (sin x, cos x) → R e il differenziale: dx =



t 1 √ ,√ 2 1+t 1 + t2



,

dt 1 + t2

Ad esempio: I (x) = diventa: I (t) = cioè:

Z

Z

dx , 1 + cos2 x

√   t 2 dt +C = arctan √ 2 + t2 2 2 √

2 I (x) = arctan 2

3.16.9 1) 4) 7)

dx 3+5 cos x

Z

3 sin x+2 cos x dx 2 sin x+3 cos x

Z

13) 16)

tan x √ 2



+C

Esercizi

Z

10)



sin x dx 1−sin x

2) 5) 8)

Z

dx sin x+cos x

Z

1+tan x dx 1−tan x

Z

Z

dx 3 sin2 x+5 cos2 x

11) 14)

Z

sin x dx (1−cos x)3 cos x dx sin2 x−6 sin x+5

17)

Z

dx 8−4 sin x+7 cos x

3) 6) 9)

Z

cos x dx 1+cos x

Z

dx 1+3 cos2 x

Z

Z

dx sin2 x+3 sin x cos x−cos2 x

12)

15)

Z

sin 2x dx 1+sin2 x dx (2−sin x)(3−sin x)

18)

Z

dx cos x−2 sin x+3

Z

dx sin2 x−5 sin x cos x

Z

1−sin x+cos x dx 1+sin x−cos x

Z

cos 2x dx cos4 x+sin4 x

3.16 Integrali di funzioni trigonometriche

3.16.10

219

Soluzioni

1. I (x) =

R

dx . 3+5 cos x

Qui è: R (cos x) =

1 , 3 + 5 cos x

donde il cambio di variabile è (3.117). L’integrale diventa: Z dt 1 2 + t I (t) = = ln +C 4 − t2 4 2 − t

Cioè:

2.

R

dx . sin x+cos x

1 2 + tan I (x) = ln 4 2 − tan



x 2  x 2

+C

Z x dt x → t = tan =⇒ I (t) = − 2 2 t − 2t − 1

Applicando direttamente la (3.45) troviamo: t + 1 + √2 1 √ +C I (t) = √ ln 2 2 t − 1 − 2

Cioè:

3. I (x) =

R

cos x dx. 1+cos x

√ x 2 + 1 + tan 1 2 √ I (x) = √ ln +C 2 2 tan x2 − 1 − 2 Z  1− I (x) =

 1 dx 1 + cos x = x − J (x) + const,

essendo: J (x) =

Z

dx 1 + cos x

Eseguiamo il cambio di variabile: x x → t = tan , 2

3.16 Integrali di funzioni trigonometriche

220

donde: J (t) =

Z

=

Z

1 dt 2 1−t2 1 + t2 1 + 1+t2 dt = t + const,

quindi: J (x) = tan

x + const 2

Cosicché l’integrale vale: I (x) = x − tan 4. I (x) =

R

sin x dx 1−sin x

=

R

sin x−1+1 dx 1−sin x

x + const 2

= −x + J (x), essendo:

J (x) =

Z

dx 1 − sin x

Eseguiamo il cambio di variabile: x x → t = tan , 2 donde: Z

1 2dt 2t 1 − 1+t2 1 + t2 Z dt 2 =2 + const, 2 = − t−1 (t − 1) J (t) =

quindi: J (x) = −

2 x 2

tan − 1

+ const

Cosicché l’integrale vale: I (x) = −x − 5. I (x) =

R

dx . 8−4 sin x+7 cos x

2 x 2

tan − 1

+ const

Eseguiamo il cambio di variabile: x → tan

x 2

3.16 Integrali di funzioni trigonometriche Quindi:

Z I (t) = 2

221

dt t2 − 8t + 15

L’integrale suddetto si risolve con la (3.45): t − 5 + C, I (t) = ln t − 3

da cui:

6. I (x) =

R

dx . cos x−2 sin x+3

tan x2 − 5 +C I (x) = ln tan x2 − 3

Eseguiamo il cambio di variabile: x → tan

Quindi: I (t) =

Z

t2

x 2

dt − 2t + 2

L’integrale suddetto si risolve con la (3.45): I (t) = arctan (t − 1) + C, da cui:

 x I (x) = arctan tan − 1 + C 2 

R x+2 cos x dx. Osserviamo che il numeratore si può esprimere 7. I (x) = 32 sin sin x+3 cos x come combinazione lineare del denominatore e della sua derivata prima: 3 sin x + 2 cos x = A (2 sin x + 3 cos x) + B

d (2 sin x + 3 cos x) dx

Quindi: I (x) = Ax + B ln |2 sin x + 3 cos x| + C, essendo C l’usuale costante di integrazione. Dalla (3.119) si ottiene il sistema: 2A − 3B = 3 3A + 2B = 2,

(3.119)

3.16 Integrali di funzioni trigonometriche

222

quindi i coefficienti indeterminati A e B: A=

12 5 , B=− 13 13

e l’integrale: I (x) = 8. I (x) =

R

1+tan x dx 1−tan x

=

R

5 12 x− ln |2 sin x + 3 cos x| + C 13 13

sin x+cos x dx cos x−sin x

=−

R

d(cos x−sin x) cos x−sin x

= − ln |cos x − sin x| + C R . Osserviamo che: 9. I (x) = 1+3dx cos2 x Z dx I (x) = 1 + 3 cos2 x Z dx 1 = 1 + 3 cos2 x cos2 x Ma: 1 = 1 + tan2 x cos2 x dx = d (tan x) , cos2 x donde eseguendo il cambio di variabile x → y = tan x: Z dy I (x) = 4 + y2  Z d y2 1 =  2 1+ y 2 2   1 tan x = arctan +C 2 2 10. I (x) =

R

dx . 3 sin2 x+5 cos2 x

Osserviamo che: Z dx I (x) = 2 2 Z 3 sin x + 5 cos x dx 1 = 2 3 tan x + 5 cos2 x Z 1 d (tan x) , = 5 + 3 tan2 x

3.16 Integrali di funzioni trigonometriche donde eseguendo il cambio di variabile x → y = tan x: Z dy I (x) = 5 + 3y 2 q  3 Z d y 5 1 =√ q 2 3 15 y 1+ 5 ! r 3 1 = √ arctan tan x + C 5 15 11. I (x) =

R

dx . sin2 x+3 sin x cos x−cos2 x

I (x) =

Z

Risulta:

cos2

dx x (tan x + 3 tan x − 1) 2

Eseguendo il cambio di variabile x → y = tan x: Z dy I (y) = y 2 + 3y − 1 2y + 3 − √13 1 √ + C, = √ ln 13 2y + 3 + 13 cioè:

12. I (x) =

R

2 tan x + 3 − √13 1 √ +C I (x) = √ ln 13 2 tan x + 3 + 13

dx . sin2 x−5 sin x cos x

Risulta:

I (x) =

Z

dx sin x (1 − 5 cot x) 2

Eseguendo il cambio di variabile x → y = cot x: Z dy I (y) = 5y − 1 1 = ln |5y − 1| + C, 5 cioè:

1 5 − tan x +C I (x) = ln 5 tan x

223

3.16 Integrali di funzioni trigonometriche 13. I (x) =

R

sin x dx. (1−cos x)3

224

Risulta: Z I (x) = −

d (cos x) (1 − cos x)3

Eseguendo il cambio di variabile x → y = cos x: Z d (1 − y) I (y) = (1 − y)3 1 +C =− 2 (1 − y)2 Cioè: I (x) = − 14. I (x) =

R

sin 2x dx. 1+sin2 x

1 +C 2 (1 − cos x)2

Risulta: Z sin x cos x I (x) = 2 2 dx 1 + sin x Z sin x d (sin x) =2 1 + sin2 x

Eseguendo il cambio di variabile x → y = sin x: Z ydy I (y) = 2 1 + y2 Z d (1 + y 2 ) = 1 + y2  = ln 1 + y 2 + C

Cioè: 15.

R

cos 2x dx. cos4 x+sin4 x

Risulta:

 I (x) = ln 1 + sin2 x + C Z

cos2 x − sin2 x I (x) = dx cos4 x + sin4 x Z 1 − tan2 x d (tan x) = 1 + tan2 x

Eseguendo il cambio di variabile x → y = tan x: Z 1 − y2 I (y) = dy 1 + y4 = I1 (y) − I2 (y) ,

(3.120)

3.16 Integrali di funzioni trigonometriche

225

essendo: Z

dy 1 + y4 Z 2 y dy I2 (y) = 1 + y4 I1 (y) =

L’integrale I1 (y) è stato già calcolato precedentemente (eq. 3.65): # " y 2 + √2y + 1   √  √  1 √ I1 (y) = √ −2 arctan 1 − 2y + 2 arctan 1 + 2y + ln 2 4 2 y − 2y + 1 +const

Per il secondo integrale procediamo per decomposizione in frazioni semplici: y2 Cy + D Ay + B √ √ + = 4 1+y y 2 + 2y + 1 y 2 − 2y + 1 Risolvendo il conseguente sistema di Cramer:   1 1 (A, B, C, D) = − √ , 0, √ , 0 , 2 2 2 2 donde:

1 1 I2 (y) = − √ J1 (y) + √ J2 (y) , 2 2 2 2

essendo: Z

ydy √ J1 (y) = y 2 + 2y + 1 Z ydy √ J2 (y) = , 2 y − 2y + 1 che si calcolano con la (3.45):  √ 1 2 √ 2y + 1 + const J1 (y) = ln y + 2y + 1 − arctan 2   √ 1 2 √ J1 (y) = ln y − 2y + 1 + arctan 2y − 1 + const 2

Quindi:

y 2 + √2y + 1   √ i √  1 1 h √ I2 (y) = − √ ln + √ arctan 1 + 2y − arctan 1 − 2y 4 2 y 2 − 2y + 1 2 2 +const

3.16 Integrali di funzioni trigonometriche

226

Sostituendo nella (3.120): y 2 + √2y + 1 1 √ I (y) = √ ln + const 2 2 2 y − 2y + 1

Cioè:

16.

R

tan2 x + √2 tan x + 1 1 √ I (x) = √ ln + const 2 2 2 tan x − 2 tan x + 1 √ 2 + sin 2x 1 = √ ln √ + const 2 2 2 − sin 2x

cos x dx sin2 x−6 sin x+5

y = sin x:

=

R

d(sin x) . sin2 x−6 sin x+5

Z

dy y 2 − 6y + 5 1 y − 5 +C = ln 4 y − 1

I (y) =

Da cui:

Eseguendo il cambio di variabile x →

1 sin x − 5 +C I (x) = ln 4 sin x − 1

R dx . Riduciamo l’integrando in frazioni semplici (rispet17. I (x) = (2−sin x)(3−sin x) to a sin x): 1 A B = + (2 − sin x) (3 − sin x) 2 − sin x 3 − sin x (3A + 2B) + (−A − B) sin x = (2 − sin x) (3 − sin x) Deve essere: A+B =0 3A + 2B = 1 Risolvendo: (A, B) = (1, −1) Quindi:

1 1 1 = − (2 − sin x) (3 − sin x) 2 − sin x 3 − sin x

3.16 Integrali di funzioni trigonometriche

227

L’integrale: I (x) = I1 (x) − I2 (x) , essendo: Z

dx 2 − sin x Z dx I2 (x) = 3 − sin x I1 (x) =

Risolviamo I1 (x): x x → t = tan =⇒ I1 (t) = 2 Perciò:

Z

dt 2 = √ arctan 2 t −t+1 3

2 I1 (x) = √ arctan 3



2 tan x2 − 1 √ 3





2t − 1 √ 3



+ const

+ const

Risolviamo I2 (x):   Z x dt 1 3t − 1 √ x → t = tan =⇒ I2 (t) = 2 + const = √ arctan 2 3t2 − 2t + 3 2 2 2 Perciò:

1 I1 (x) = √ arctan 2





3 tan x2 − 1 √ 2 2



1 + √ arctan 2

+ const

L’integrale I (x): 2 I (x) = √ arctan 3



2 tan x2 − 1 √ 3



3 tan x2 − 1 √ 2 2



+C

R x+cos x dx. Prima di eseguire il cambio di variabile x → t = 18. I (x) = 1−sin 1+sin x−cos x x tan 2 , semplifichiamo l’integrando:  Z  2 −1 + dx I (x) = 1 + sin x − cos x = −x + 2J (x) , essendo: J (x) =

Z

dx 1 + sin x − cos x

3.16 Integrali di funzioni trigonometriche

228

Cambiando la variabile: Z

dt t (t + 1)  Z  1 1 − dt = t t+1 t +C = ln t + 1

J (t) =

da cui I (x):

tan x2 +C I (x) = −x + 2 ln tan x + 1 2

3.16.11 1) 4) 7)

Z

Z

Z

10)

13) 16)

Esercizi riepilogativi sugli integrali trigonometrici 2)

3

4

cos 2x sin 2xdx cosn nx dx √

(n > 1)

5)

Z

Z

tan3 3x sec4 3xdx

11)

Z

sin7 xdx

Z

14)

4

cot 3x csc 3xdx

1. I (x) =

R

4

2

sin 3x cos 3xdx

Z

8)

5

sin 3x cos 3xdx

Z

1 − cos xdx

3

(1 + cos 3x)3/2 dx

Z

Z

tan2 x sec3 xdx 3

5

cot x csc xdx

3) 6) 9)

Z

sin2 3x cos5 3xdx

Z

√ dx 1−sin 2x

Z

12)

15)

Z

sin 3x sin 2xdx

Z

tan3 2x sec3 2xdx sin4 2xdx

cos4 2x sin3 2x. Abbiamo: Z  1 cos4 2x 1 − cos2 2x d (cos 2x) I (x) = − 2

Poniamo y = cos 2x:

Z  1 I (y) = y 4 y 2 − 1 dy 2   1 1 7 1 5 y − y +C = 2 7 5 Cioè: I (x) =

1 1 cos7 2x − cos5 2x + C 14 10

3.16 Integrali di funzioni trigonometriche 2. I (x) =

R

sin3 3x cos5 3xdx. Abbiamo: Z  1 − sin2 3x cos5 3x sin 3xdx I (x) =

Poniamo y = cos 3x:

Z  1 y 5 1 − y 2 dy I (y) = − 3   1 1 6 1 8 =− y − y +C 3 6 8 Cioè: I (x) = 3. I (x) =

R

1 1 cos8 3x − cos6 3x + C 24 18

sin2 3x cos5 3xdx. Abbiamo: Z 2 1 I (x) = sin2 3x 1 − sin2 3x d (sin 3x) 3

Poniamo y = sin 3x:

Z 2 1 y 2 1 − y 2 dy I (y) = 3   1 1 3 2 5 1 7 = y − y + y +C 3 3 5 7 Cioè:

4. In (x) =

1 I (x) = 3 R



 1 3 2 5 1 7 sin 3x − sin 3x + sin 3x + C 3 5 7

cosn nx dx. Z  x n−1 x cos In (x) = n cos dx n n Z  n−1   x 2 2 x 1 − sin d sin =n n n

Eseguendo il cambio di variabile x → y = sin nx : Z  n−1 1 − y 2 2 dy In (y) = n

229

3.16 Integrali di funzioni trigonometriche

230

Ad esempio, per n = 3: Z  1 − y 2 dy I3 (y) = 3   1 3 = 3 y − y + C, 3 per cui: I3 (x) = 3 sin 5. I (x) =

R

x x − sin3 + C 3 3

sin4 3x cos2 3xdx. Abbiamo: Z I (x) = (sin 3x cos 3x)2 sin2 3xdx

Osservando che: 1 sin 6x 2 1 sin2 3x = (1 − cos 6x) , 2

sin 3x cos 3x =

si ha: Z 1 I (x) = sin2 6x (1 − cos 6x) dx 8 1 = [I1 (x) − I2 (x)] , 8 essendo:   1 1 I1 (x) = (1 − cos 12x) dx = x− sin 12x + C1 2 12 Z Z 1 1 2 I2 (x) = sin 6x cos 6xdx = sin3 6x + C2 , sin2 6xd (sin 6x) = 6 18 Z

1 sin 6xdx = 2 2

per cui: I (x) = 6. I (x) =

R

Z

1 1 x − sin 12x − sin3 6x + C 16 144 192

sin 3x sin 2xdx . Per le (3.115): sin 3x sin 2x =

donde: I (x) =

1 (cos x − cos 5x) , 2

1 1 sin x − sin 5x + C 2 10

3.16 Integrali di funzioni trigonometriche 7. I (x) =

R√

1 − cos xdx. Osservando che:

x cos x = 1 − 2 sin2 , 2

segue:

(3.121)

x 1 − cos x = 2 sin2 , 2

donde:

8. I (x) =

231

R

√ Z √ x x I (x) = 2 sin dx = −2 2 cos + C 2 2 (1 + cos 3x)3/2 dx. Dalla (3.121): y cos y = 2 cos2 − 1 2

Se poniamo y = 3x: 2

cos 3x = 2 cos donde: (1 + cos 3x)

3/2



 3 x − 1, 2

√ = 2 2 cos3



3 x 2



L’integrale diventa:     √ Z 3 3 2 x cos x dx I (x) = 2 2 cos 2 2      Z  3 4√ 3 2 1 − sin 2 = x d sin x 3 2 2     3 4√ 3 2 2 sin = x 3 − sin x +C 9 2 2 R R dx dx q . Poniamo: = 9. I (x) = √1−sin 2x 1−cos( π2 −2x) π y = − 2x 2 donde: Z

dy √ 1 − cos y Z 1 dy =− √ y 2 2 sin 2 1 1 = − √ ln y − cot y + C sin 2 2

1 I (y) = − 2

3.16 Integrali di funzioni trigonometriche

232

Ripristinando la variabile x: √  π 1 2  − cot I (x) = − ln −x +C sin π4 − x 2 4

10. I (x) =

R

tan3 3x sec4 3xdx. Poniamo 3x = ξ: Z 1 dξ I (ξ) = tan3 ξ 4 3 cos ξ Z 1 1 tan3 ξ 2 d (tan ξ) = 3 cos ξ

(3.122)

Ma 1 sin2 ξ + cos2 ξ = cos2 ξ cos2 ξ 2 = tan ξ + 1 Quindi la (3.122) diventa: 1 I (ξ) = 3

Z

 tan3 ξ tan2 ξ + 1 d (tan ξ)

Esegiamo il cambio di variabile ξ → y = tan ξ: Z  1 y 3 y 2 + 1 dy I (y) = 3   1 1 6 1 4 = y + y + const 3 6 4

Ritornando alla variabile x:   1 1 1 6 4 I (x) = tan 3x + tan 3x + C 6 3 2 R R R 11. I (x) = tan2 x sec3 xdx = cosdx5 x − cosdx3 x = G5 (x)−G2 (x), essendo Gn (x) espresso dalla (3.109). Abbiamo:   sin x 1 G1 (x) + + const G3 (x) = 2 cos2 x    x π  1 sin x = + ln tan + const + 2 2 4 cos2 x   1 sin x 3G3 (x) + + const, G5 (x) = 4 cos4 x donde:

1 sin x I (x) = − G3 (x) + +C 4 4 cos4 x

3.16 Integrali di funzioni trigonometriche 12. I (x) =

R

233

tan3 2x sec3 2xdx. Poniamo ξ = 2x: Z 1 dξ I (ξ) = tan3 ξ 3 2 cos ξ Z 1 1 − cos2 ξ =− d (cos ξ) 2 cos6 ξ

Eseguiamo il cambio di variabile y = cos ξ: Z  Z 1 −4 −6 y dy − y dy I (y) = 2   1 1 −3 1 −5 = +C − y + y 2 3 5 Ritornando alla variabile x: 1 I (x) = 2 13. I (x) =

R



1 1 − 5 5 cos 2x 3 cos3 2x

cot 3x csc4 3xdx = − 13

R

cot 3x d (cot 3x). sin2 3x



+C

Osservando che:

1 = 1 + cot2 3x, sin2 3x

si ha:

14. I (x) =

R

Z  1 I (x) = − cot 3x 1 + cot2 3x d (cot 3x) 3  1 = − cot2 3x 2 + cot2 3x + C 12

cot3 x csc5 xdx. Sviluppiamo l’integrando:

cos2 x dx sin8 x d (sin x) d (sin x) − , = sin8 x sin6 x

cot3 x csc5 xdx =

donde: Z d (sin x) d (sin x) − I (x) = 8 sin x sin6 x 1 1 + +C =− 7 7 sin x 5 sin5 x Z

3.17 Integrazione delle funzioni iperboliche 15. I (x) =

234

R

sin4 2xdx. Abbiamo (ξ = 2x): Z 1 I (ξ) = sin4 ξdξ 2   Z 1 1 2 = (2ξ − sin 2ξ) sin ξd 2 4   1 1 1 1 1 2 2 = 2ξ sin ξ − sin ξ sin 2ξ + ξ cos 2ξ − cos 2ξ − sin 2ξ + ξ − sin 4ξ + C 8 2 2 2 8

Ripristinando la variabile x:   1 1 1 2 2 4x sin 2x − sin 2x sin 4x + 2x cos 4x − sin 4x + x − sin 8x + C I (x) = 8 2 8 3 3 1 cos 2x sin 2x + x + C = − sin3 2x cos 2x − 8 16 8 R R 3 16. I (x) = sin7 xdx = − (1 − cos2 x) d (cos x). Eseguiamo il cambio di variabile x → y = cos x: 1 3 I (y) = y 7 − y 5 + y 3 − y + C 7 5 Ripristinando la variabile x: I (x) =

3.17

3 1 cos7 x − cos5 x + cos3 x − cos x + C 7 5

Integrazione delle funzioni iperboliche

Il punto di partenza è la relazione fondamentale: cosh2 x − sinh2 x = 1

(3.123)

cosh 2x = 2 sinh2 x + 1,

(3.124)

La prima relazione utile si ottiene dalla formula di duplicazione:

risolvendo rispetto a sinh2 x: 1 (cosh 2x − 1) 2 La (3.125) è utilizzabile quando l’integrando contiene sinh2 x. Tenendo conto della (3.123), la (3.125) diventa: sinh2 x =

cosh2 x =

1 (cosh 2x + 1) 2

(3.125)

(3.126)

3.17 Integrazione delle funzioni iperboliche

235

Infine dalla: sinh 2x = 2 sinh x cosh x, si ottiene: 1 sinh 2x, 2 utilizzabile quando l’integrando contiene sinh x cosh x. Esempio 1 Calcoliamo:

(3.127)

sinh x cosh x =

I (x) = = = = =

Z

cosh2 xdx Z 1 (cosh 2x + 1) dx 2  Z  Z 1 1 cos 2xd (2x) + dx 2 2   1 1 sinh 2x + x + C 2 2 1 1 x + sinh 2x + C 2 4

Esempio 2 Calcoliamo: I (x) = = = =

3.17.1

Z

Z

cosh3 xdx cosh2 xd (sinh x)

Z

 sinh2 x + 1 d (sinh x)

1 sinh3 x + sinh x + C 3

Esercizi 1) 4) 7)

Z

sinh xdx

Z

tanh3 xdx

Z

3

2

2) 2

sinh x cosh xdx

5)

Z

Z

4

cosh xdx dx sinh x cosh2 x

3) 6)

Z

Z

sinh3 x cosh xdx dx sinh2 x cosh2 x

3.17 Integrazione delle funzioni iperboliche 1. I (x) =

R

sinh3 xdx =

R

sinh2 xd (cosh x) =

R

= 31 cosh3 x − cosh x + C R R 2. I (x) = cosh4 xdx = 14 (cosh 2x + 1)2 dx =   = 41 18 sinh (4x) + x2 + 21 sinh (2x) + c + C

236  cosh2 x − 1 d (cosh x) 1 4

R

 R cosh2 2xdx + 2 cosh 2x + x

sinh (4x) + 14 sinh (2x) + 43 x + C. R R 3. I (x) = sinh3 x cosh xdx = sinh3 xd (sinh x) = 14 sinh4 x + C R R R 4. I (x) = sinh2 x cosh2 xdx = (sinh x cosh x)2 dx = 14 sinh2 2xdx R Calcoliamo a parte sinh2 2xdx: Z Z 1 2 sinh2 tdt sinh 2xdx = t=2x 2 Z 1 (cosh 2t − 1) dt = 4 1 1 = sinh 2t − t + C1 8 4 1 1 sinh (4x) − x + C =⇒ I (x) = 32 8 R R 1 = sinh 5. I (x) = sinh xdx d (tanh x) x cosh2 x =

1 32

Abbiamo:

1 = sinh x

s

cosh2 x − sinh2 x = sinh2 x

quindi: I (x) =

Z r

r

1 − 1, tanh2 x

1 − 1d (tanh x) tanh2 x

Poniamo y = tanh x, per cui:

Z p 1 − y2 I (y) = dy y Z = y m (1 − y n )p dy ∈ N, quindi la sostituzione è con m = −1, n = 2, p = 1/2. Risulta: m+1 n 2 2 1−y =t Z t2 dt I (t) = t2 − 1  Z  t − 1 1 +C 1+ 2 = dt = t + ln t −1 t + 1

3.17 Integrazione delle funzioni iperboliche

237

Ripristinando la variabile y: I (y) = Ma y = tanh x

p

1−

I (x) = 6. I (x) =

R

dx sinh2 x cosh2 x

Abbiamo:

y2

p 2 + ln |y| − ln 1 + 1 − y

 x  1 + ln tanh +C cosh x 2

1 1 = − 1, 2 sinh x tanh2 x

quindi: I (x) =

Z 

 1 − 1 d (tanh x) tanh2 x

Poniamo y = tanh x, per cui: Z

1 − y2 dy y2 Z Z dy = − dy y2 1 =− −y+C y

I (y) =

Ripristinando la variabile x: I (x) = − coth x − tanh x + C 7. I (x) =

R

tanh3 xdx. È del tipo In (x) =

Z

tanhn xdx

e va calcolato tramite una relazione di ricorrenza. I0 (x) = x + C I1 (x) = ln |cosh x| + C Per ogni n

(3.128)

3.18 Sostituzioni trigonometriche ed iperboliche

3.18

238

Sostituzioni trigonometriche ed iperboliche

Sia I (x) =

Z

 √  R x, ax2 + bx + c dx,

essendo R una funzione razionale. Per calcolare I (x) è conveniente ridurre il trinomio ax2 + bx + c alla forma a (x + k)2 + l, e con un opportuno cambio di variabile si ha: Z

  p sost. trig. R y, α2 − y 2 dy =⇒ y = α sin t Z  p  sost. iper. I (y) = R y, α2 + y 2 dy =⇒ y = α sinh t Z  p  sost. iper. I (y) = R y, y 2 − α2 dy =⇒ y = α cosh t I (y) =

(3.129)

Nel secondo e terzo integrale è possibile eseguire sostituzioni trigonometriche anzichè iperboliche: Z

  p sost. trig. 2 2 I (y) = R y, α + y dy =⇒ y = α tan t Z  p  sost. trig. I (y) = R y, y 2 − α2 dy =⇒ y = α sec t

(3.130)

Capitolo 4 Integrali definiti 4.1

Somme integrali

Si chiede di calcolare i seguenti integrali considerandoli come il limite delle corrispondenti somme integrali. 1)

Zb

x2 dx

0

4)

Z1

3)

dx

a

x2 dx

−2

4.1.1

2)

Zb

Z10 5) 2x dx

ZT

(v0 + gt) dt, con v0 , g =const

0

6) f (x) =

0

Zx

sin tdt

0

Soluzioni

1. Eseguiamo una equipartizione Dn di [0, b]: b xk = k , n

k ∈ N = {0, 1, ..., n − 1}

L’ampiezza è δn =

b n

La somma integrale: σDn =

n−1 X k=0

Prendiamo ξk = xk : σDn

f (ξk ) (xk+1 − xk )

 3 X n−1 b k2 = n k=0

4.1 Somme integrali

240

Ma:

n−1 X

n (2n − 1) (n − 1) , 6

k2 =

k=0

donde: σDn =

b3 (2n − 1) (n − 1) , 6 n2

da cui l’integrale: Zb

x2 dx = lim σDn = lim σDn = n→+∞

δn →0

0

b3 3

2. Eseguiamo unequipartizione Dn di [a, b]: xk = a +

k (b − a) , k ∈ N = {0, 1, 2, ..., n − 1} n

L’ampiezza di Dn è: δn =

b−a n

Assumendo ξk = xk+1 σDn =

n−1 X k=0

f (xk+1 ) (xk+1 − xk ) n

=

b − aX f (xk ) n k=1

= b − a,

quindi: Zb a

dx = lim σδn = lim σn = b − a n→+∞

δn →0

3. Quest’integrale ha una semplice interpretazione fisica, se poniamo: y (T ) =

ZT

(v0 + gt) dt

0

Qui y (T ) è al tempo T la quota di un punto materiale in caduta libera in un campo gravitazionale (g è l’accelerazione di gravità) con velocità iniziale

4.1 Somme integrali

241

v0 . L’asse y è orientato verso il basso e si trascura la resistenza dell’aria. L’integrando è la velocità istantanea: v (t) = v0 + gt Eseguiamo un’equipartizione Dn di [0, T ]: T tk = k , con k ∈ N = {0, 1, 2, ..., n − 1} n L’ampiezza è δn = max (tk+1 − tk ) = k∈N

T n

La somma integrale è σDn =

n−1 X k=0

v (τk ) (tk+1 − tk )

Assumendo τk = tk+1 : σDn

 n  T TX v0 + g k = n k=1 n = T v0 +

n + 1 gT n 2

Quindi l’integrale: y (T ) = lim σDn δn →0

= lim σDn n→+∞

1 = v0 T + gT 2 2 4. Eseguiamo una equipartizione Dn di [−2, 1]: xk = −2 +

3 k, con k = 0, 1, ..., n − 1 n

di ampiezza: δn =

3 n

4.1 Somme integrali

242

Assumendo ξk = xk+1 : n

3X 2 x n k=1 k 2 n  3X 3 k−2 = n k=1 n σDn =

= da cui:

Z1

3 2n2 − 3n + 3 2 n

x2 dx = lim σDn = 3 n→+∞

−2

5. Eseguiamo una equipartizione Dn di [0, 10]: xk =

10 k, con k = 0, 1, ..., n − 1 n

di ampiezza: δn =

10 n

Assumendo ξk = xk+1 : n

σDn

da cui:

10 X k 10 2 n = n k=1 n  10+n 52 n −1 + (1024)1/n = n (−1 + 10241/n )

Z10 2x dx = lim σDn = n→+∞

10230 ln (1024)

0

6. Eseguiamo l’equipartizione: x tk = k , con k ∈ N = {0, 1, ..., n − 1} n L’ampiezza dell’intervallo [tk , tk+1 ] è: δk,n = tk+1 − tk =

x , n

4.2 Teorema fondamentale del calcolo integrale.

243

donde l’ampiezza della partizione: x n

δn = max (δk ) = k∈N

Assumendo τk = tk+1 e ponendo g (t) = sin t: σDn = =

n−1 X

g (tk+1 )

k=0 n X

x n

 x x sin k n k=1 n

Per il calcolo della sommatoria utilizziamo una nota relazione trigonometrica:     n X cos y2 − cos n + 21 y  , sin (ky) = y 2 sin 2 k=1

donde:

σDn =

 x cos

y 2



Passiamo al limite per n → +∞:  x cos lim σDn = lim n→∞

n→∞

=

   − cos n + 21 y  2n sin y2 x 2n



 − cos 1 +  x 2n sin 2n

x (1 − cos x)  x 2 lim n sin 2n

1 2n

  x

n→+∞

Calcoliamo a parte il limite a denominatore:  x sin mx x 2 = , lim n sin =−1 lim+ n→+∞ 2n m=n m→0 m 2 Quindi f (x) = lim σDn = 1 − cos x n→∞

4.2

Teorema fondamentale del calcolo integrale.

1. Calcolare

dI dI , , da db

essendo: I=

Zb a

dx ln (sin x)

4.2 Teorema fondamentale del calcolo integrale. sia F (x) una qualunque primitiva di [ln (sin x)]−1 : Z dx F (x) = , ln (sin x) per cui: I = F (b) − F (a) Quindi le derivate: dI 1 = −F ′ (a) = − da ln (sin a) 1 dI = F ′ (b) = db ln (sin b) 2. Calcolare la derivata della funzione: F (x) =

Zx

ln tdt

Z

ln tdt

1

Poniamo: G (t) = Quindi:

F (x) = G (x) − G (1) =⇒ F ′ (x) = G′ (x) = ln x 3. Calcolare la derivata della funzione: F (x) =

Zx2

e−t dt

Z

e−t dt

2

x

Poniamo: G (t) = Quindi: La derivata è:

2

 F (x) = G x2 − G (x)  d d G x2 − G (x) dx  dx = 2xG′ x2 − G′ (x)

F ′ (x) =

4

= 2xe−x − e−x

2

244

4.2 Teorema fondamentale del calcolo integrale. 4. Calcolare la derivata della funzione: F (x) =

Z0 √

1 + t4 dt

x

Poniamo: G (t) = Quindi:

Z √

1 + t4 dt

√ F (x) = G (0) − G (x) =⇒ F ′ (x) = − 1 + x4

5. Calcolare la derivata della funzione: √

F (x) =

Zx

1/x

Poniamo: G (t) = Quindi:

Z

 cos t2 dt

 cos t2 dt

√  F (x) = G x − G

La derivata è:

  1 x

  √  d d 1 F (x) = G x − G dx dx x   1 1 1 = √ cos x + 2 cos x x2 2 x ′

6. Determinare i punti estremali della funzione seno integrale: F (x) =

Zx

sin t dt t

Z

sin t dt, t

0

Poniamo: G (t) = donde:

F (x) = G (x) − G (0)

245

4.2 Teorema fondamentale del calcolo integrale. Segue: F ′ (x) = G′ (x) =

sin x x

I punti estremali di F (x) sono le radici dell’equazione: F ′ (x) = 0 ⇐⇒

sin x =0 x

Cioè: xk = kπ, ∀k ∈ Z − {0} In figura 4.1 è riportato il grafico di F (x). *** Calcolare: Z1 dx 1) 1+x

2)

3)

dx x3

−2

0

Zx

Z−1

e dt 4) t

−x

Zx

cos tdt

0

Soluzioni R1 1. I = 0

dx . 1+x

Poniamo: F (x) =

Z

dx = ln |1 + x| + C, 1+x

donde: I = F (1) − F (0) = ln 2 2. I = 3. I =

R −1

dx −2 x3

Rx

−1 = − 2x12 −2 = − 38 . x

et dt = et |−x = ex − e−x Rx 4. I (x) = 0 cos tdt = sin t|x0 = sin x −x

***

Calcolare i seguenti limiti attraverso gli integrali definiti.  1. lim n12 + n22 + ... + n−1 n2 n→+∞

246

4.2 Teorema fondamentale del calcolo integrale.

247

y

3 €€€€ 2 1 1 €€€€ 2 x -10 Π

-8 Π

-6 Π

-4 Π

-2 Π







1 - €€€€ 2 -1 3 - €€€€ 2

Figura 4.1: Grafico della funzione seno integrale e degli asintoti orizzontali: y = ±π/2.



10 Π

4.2 Teorema fondamentale del calcolo integrale. 2.

lim

n→+∞

1 n+1

+

1 n+2

+ ... +

1p +2p +...+np np+1 n→+∞

3. λp = lim

1 n+n

248



(p > 0)

Soluzioni 1. È il limite di una somma integrale relativa a f (x) = x in [0, 1]. Infatti, eseguiamo una equipartizione Dn di [0, 1]: xk =

k , con k ∈ N = {0, 1, 2, ..., n − 1} n

La norma è: δn = max (xk − xk−1 ) = k∈N

1 n

Assumendo ξk = xk : σDn

n−1 X = f (ξk ) (xk − xk−1 )

= =

k=0 n−1 X

1 k n2 k=0

1 2 n−1 + 2 + ... + , 2 n n n2

donde: lim

n→+∞



2 n−1 1 + + ... + n2 n2 n2



= lim σDn = n→+∞

2. Poniamo: σDn = =

1 1 1 + + ... + n+1 n+2 n+n n X 1 k=1

n+k

essendo Dn una equipartizione di [0, 1]. Deve essere: σDn =

n X

f (xk ) (xk − xk−1 )

k=1 n X

1 f = n k=1

  k n

Z1 0

xdx =

1 2

4.2 Teorema fondamentale del calcolo integrale.

249

Confrontando con la precedente:   k n f = n n+k 1 = , 1 + nk cioè: f (x) =

1 , 1+x

per cui σDn è una somma integrale relativa a f (x) = x. Da ciò segue: lim

n→+∞



1 1 1 + + ... + n+1 n+2 n+n



=

Z1 0

3. Poniamo: 1p + 2p + ... + np np+1 n 1 X p = p+1 k n k=1

σDn =

essendo Dn una equipartizione di [0, 1]. Deve essere: σDn =

n X

f (xk ) (xk − xk−1 )

k=1 n X

1 f = n k=1

  k n

Confrontando con la precedente:    p k k f = , n n cioè: f (x) = xp Quindi: λp =

Z1 0

xp dx =

1 p+1

dx = ln 2 1+x

4.2 Teorema fondamentale del calcolo integrale.

250

*** Calcolare:

1)

Z2

(x2 − 2x + 3) dx

2)

5)

9)

Z1

√ dx 25+3x

6)

Z−3

−2

0

dx x2 +4x+5

10)

14)

Z1

5

Z7/2



2

dx −x2 +4x+5

√  2x + 3 x dx

Z4

√ 1+ y dy y2

4)

1

Z1

11)

xdx x2 +3x+2

Z1

8)

Z1

s3 ds s8 +1

12)

0

y 6 +4

16)

Soluzioni  x=2 R2 1. I = 1 (x2 − 2x + 3) dx = 31 x3 − x2 + 3x x=1 = 73 √ R8 R8 R8 √ √  2x + 3 x dx = 2 0 x1/2 dx + 0 x1/3 dx 2. I = 0 √ = 2 · 32 · 83/2 + 43 · 84/3 = 100 3 h i4  4 R 4 1+√y R 4 −2 R 4 −3/2 3. I = 1 y2 dy = 1 y dy + 1 y dy = − y1 + −2y −1/2 1 1  7 1 1 = − 4 + 1 + −2 · 2 + 2 · 1 = 4 R6√ 4. I = 2 x − 2 = F (6) − F (2), essendo Z √ F (x) = x − 2dx Z = (x − 2)1/2 d (x − 2) 2 (x − 2)3/2 + C, 3

donde: I=

16 3

x − 2dx

y 5 dy y+2

Z2/2

√ dx 1−x2

0

0

0



−1√

Zπ/2 sin3 xdx 15)

2 dy

=

Z6 2

0

dx x2 −3x+2

√y

3) 7)

dx x2 −1

Z4

0

13)



0

1

Z−3

Z8

Ze2 e

dx x ln x

4.2 Teorema fondamentale del calcolo integrale. 5. I =

R −3 0

√ dx 25+3x

= F (−3) − F (0), essendo: Z dx √ F (x) = 25 + 3x Z 1 = (25 + 3x)−1/2 d (25 + 3x) 3 2√ 25 + 3x + C, = 3

donde: I= 6. I =

R −3

dx −2 x2 −1

donde:

2√ 2√ 2 16 − 25 = − 3 3 3

= F (−3) − F (−2), essendo: Z dx 1 x − 1 F (x) = + C, = ln x2 − 1 2 x + 1 1 ln 2 + C 2 1 F (−2) = ln 3 + C 2 F (−3) =

Quindi: I= 7. I =

R1

xdx 0 x2 +3x+2

1 2 ln 2 3

= F (1) − F (0), essendo: F (x) =

Z1

xdx x2 + 3x + 2

0 3 x + 1 1 2 +C = ln x + 3x + 2 − ln 2 2 x + 2 = − ln |x + 1| + 2 ln |x + 2| + C,

donde: F (1) = − ln 2 + 2 ln 3 + C F (0) = 2 ln 2 + C Quindi: I = ln

9 8

251

4.2 Teorema fondamentale del calcolo integrale. 8. I =

R1

y 5 dy −1 y+2

= F (1) − F (−1), essendo: Z 5 y dy F (y) = y+2  Z  32 4 3 2 y − 2y + 4y − 8y + 16 − = dy y+2 1 1 1 = y 5 − y 4 + y 3 − 4y 2 + 16y − 32 ln |y + 2| + C, 5 2 5

donde: I= 9. I =

R1

dx 0 x2 +4x+5

526 − 32 ln 3 15

= F (1) − F (0), essendo: Z dx F (x) = 2 x + 4x + 5

Per la (3.45): F (x) = arctan (x + 2) + C, donde: I = arctan 3 − arctan 2, che può eesere semplificata: con α = arctan 3, β = arctan 2

tan I = tan (α − β) , Come è noto:

tan (α − β) = da cui: tan I = 10. I =

R1

dx 0 x2 −3x+2

Per la (3.45):

donde:

tan α − tan β , 1 + tan α tan β

1 1 =⇒ I = arctan 7 7

= F (4) − F (5), essendo: Z dx F (x) = x2 − 3x + 2 x − 2 + C, F (x) = ln x − 1 I = ln

3 8 2 − ln = ln 3 4 9

252

4.2 Teorema fondamentale del calcolo integrale. 11. I =

R1

s3 ds 0 s8 +1

= F (1) − F (0), essendo: F (s) =

Z

s3 ds s8 + 1

Eseguendo il cambio di variabile s → y = s4 : Z 1 dy 1 F (y) = = arctan y 2 4 y +1 4 Ripristinando la variabile s: F (s) = donde:

 1 arctan s4 , 4 I=

12. I = 13. I =

R √2 2

0

R 7/2 2

√ dx 1−x2 √

√ 2 2

= [arcsin x]0 =

dx −x2 +4x+5

=F

7 2



π 4

− F (2), essendo:

F (x) =

Z



Dalla (3.46): F (x) = arcsin donde:

14. I =

R1 0

π 16

dx −x2 + 4x + 5 

x−2 3



+ C,

  π 1 I = arcsin = 2 6 √y

2 dy

y 6 +4

= F (1) − F (0), essendo: F (y) =

Z

y 2 dy p y6 + 4

Eseguendo il cambio di variabile y → x = y 3 : Z √ 1 1 dx 2 √ F (x) = = ln x + x + 4 + C 2 3 3 x +4

253

4.2 Teorema fondamentale del calcolo integrale. Cioè:

254

1 3 p 6 F (y) = ln y + y + 4 + C 3

donde:

1 I = ln 3

15. I (x) = Z

R π/3 0

π/2

0

sin3 xdx =

R π/2 0

√ ! 1+ 5 2

sin2 xd (− cos x). Integriamo per parti:

π/2 sin xd (− cos x) = − cos x sin x 0 + 2 h π  i =2 F − F (0) , 2 2

2

Z

essendo: F (x) = =

Z

Z

sin x cos2 xdx cos2 xd (− cos x)

1 = − cos3 x + C, 3 donde: I= 16. I =

R e2 e

dx x ln x

2 3

= F (e2 ) − F (e), essendo: Z dx F (x) = x ln x Z d (ln x) = ln x = ln (ln x) + C,

donde:  F e2 = ln 2 + C F (e) = C Quindi: I = ln 2 Calcolare:

π/2

sin x cos2 xdx 0

4.2 Teorema fondamentale del calcolo integrale. Zπ/4

1)

5)

tan xdx

−π/4 Z−3

6)

√ dx 25+3x

9)

π/6 Z−3 −2

0

Z1

Zπ/3 2) cot4 xdx

dx x2 +4x+5

10)

Z4

14)

Z1

5

0

Z7/2

13)



2

dx x2 −1

dx −x2 +4x+5

7)

4)

0

Z6

Z1

8)

Z1

ex dx 1+e2x

2 dy

y 6 +4

11)

xdx x2 +3x+2

x − 2dx

y 5 dy y+2

−1√

Z1

s3 ds

12)

s8 +1

Z2/2

√ dx 1−x2

0

0

Zπ/2 sin3 xdx 15)

16)

0

0



2

0

dx x2 −3x+2

√y

3)

Z1

255

Ze2

dx x ln x

e

Soluzioni Rπ 1. I = −4π tan x. Risulta I = 0. Infatti se f (x) è una funzione dispari e per 4 ogni a ∈ (0, +∞): Ia =

Za

f (x) dx

−a

=

Z0

f (x) dx +

−a

Za

f (x) dx

0

Nel secondo integrale eseguiamo il cambio di variabile x → x′ = −x, per cui: 0 ≤ x = −x′ ≤ a =⇒ 0 ≥ x′ ≥ −a cioè: Ia =

Z0

−a

=

Z0

−a

2. I = 3. I =

R π3 π 6

R1

Z−a f (x) dx − f (x′ ) dx′ 0

f (x) dx +

Z0

f (x′ ) dx′ = 0

−a

π  cot4 xdx = − 13 cot3 x + cot x + x π3 =

ex dx 0 1+e2x

6

8 √ 9 3

= F (1) − F (0), essendo: Z ex F (x) = dx 1 + e2x

+ π6 .

4.2 Teorema fondamentale del calcolo integrale. Eseguendo il cambio di variabile x → y = ex : Z dy F (y) = = arctan y + C 1 + y2 Cioè: F (x) = arctan (ex ) + C, donde: I = arctan e −

π 4

256

Capitolo 5 Estensione del concetto di integrale 5.1

Introduzione

Nel capitolo 1 abbiamo introdotto la nozione di integrale definito di una funzione continua in un intervallo chiuso e limitato [a, b]. Ci si può chiedere se tale nozione possa essere estesa al caso di una funzione che abbia punti di discontinuità e/o che sia definita in un intervallo illimitato. Sotto opportune ipotesi, la risposta è affermativa. Per rendere operativa tale estensione della nozione di integrale, ricordiamo la seguente Definizione. Una funzione reale di variabile reale f (x) definita in un intervallo A ⊆ R, si dice generalmente continua, se l’insieme dei suoi punti di discontinuità è al più numerabile. In questa sezione ci occuperemo del caso in cui f (x) è generalmente continua e non negativa nell’intervallo A ⊆ R. È facile rendersi conto che è comunque possibile costruire una successione finita di intervalli: [a1 , b1 ] , [a2 , b2 ] , ..., [an , bn ] tali che: [ak , bk ] ⊂ A, con k = 1, 2, ..., n [ak , bk ] ∩ [ak′ , bk′ ] = ∅, ∀k 6= k ′ ∀k, f (x) è continua in [ak , bk ] Si osservi che tale proprietà continua a valere anche se A è illimitato. Ad esempio se A = [a, +∞), fissiamo A′ = [a, b] ⊂ A e ripetiamo il procedimento per l’intervallo A′ . Poniamo per definizione:

5.1 Introduzione

258

def

D =

n [

(5.1)

[ak , bk ]

k=1

Chiamiamo l’insieme (5.1) dominio limitato e misurabile di continuità per f (x). Osservazione. Assegnato l’intervallo A ⊆ R, esistono infiniti domini (5.1). Esempio 20. Consideriamo la seguente funzione non negativa in A = [0, 3]: 1 + (x − 2)2 se x ∈ [0, 1) x f (x) = −x2 + 4x se x ∈ [1, 3] f (x) =

(5.2)

L’insieme delle discontinuità di f (x) è: S = {ξ0 = 0, ξ1 = 1} , donde f (x) è generalmente continua. Costruiamo gli intervalli: [a1 , b1 ] , [a2 , b2 ] essendo a1 = 0.2, b1 = 0.8, a2 = 1.2, b2 = 3 (figura 5.1). Si conclude che un dominio limitato di continuità per f (x) è D = [a1 , b1 ]∪[a2 , b2 ]. *** Costruiamo quindi la famiglia: F = {D | D è un dominio limitato e mis di continuità per f (x)}

Ciò posto, indichiamo con il simbolo: Z

f (x) dx

X

l’integrale di f (x) esteso ad un intervallo chiuso e limitato X in cui f (x) è continua. Evidentemente: ∀D ∈ F,

Z

D

bk

f (x) dx =

n Z X

k=1a k

f (x) dx ≥ 0,

poiché è f (x) ≥ 0 per ipotesi. Inoltre: Z f (x) dx = λ ∈ Λ ⊆ [0, +∞) , ∀D ∈ F, D

5.1 Introduzione

259

y

x a1

b1 Ξ1 a2

Figura 5.1: Grafico della funzione (5.2)

b2

5.2 Rettangoloide generalizzato

260

Definizione. Si chiama integrale della funzione generalmente continua f (x) ≥ 0 esteso all’intervallo A, l’estremo superiore dell’insieme Λ: Z f (x) dx = sup Λ ≤ +∞ (5.3) A

5.2

Rettangoloide generalizzato

La definizione (5.3) non si presta ad un calcolo diretto dell’integrale di una funzione generalmente continua esteso ad un intervallo limitato o illimitato. Ciò può essere realizzato attraverso un’operazione di passaggio al limite. Iniziamo con l’osservare che: D ∈ F =⇒ ∃D′ ∈ F | D′ ⊇ D,

per cui possiamo costruire una successione di domini:

Inoltre:

{Dn }n∈N | D1 ⊆ D2 ⊆ ...Dn ⊆ ...

(5.4)

∀D ∈ F, D ⊆ A − S

essendo S l’insieme (numerabile) dei punti di discontinuità di f (x). Definizione. Al crescere indefinito di n, la successione (5.4) tende all’insieme A − S se risulta: ∀D ⊆ A − S, ∃ν ∈ N | Dν ⊇ D Quindi:

Esprimiamo ciò scrivendo:

∀n ∈ N, ∃ν ∈ N | Dν ⊇ Dn lim Dn = A − S

n→+∞

Teorema 21. Nelle ipotesi poste: Z Z f (x) dx = sup Λ = f (x) dx lim n→+∞ Dn

A

Dimostrazione. Omessa. Questo teorema ci dice che nelle ipotesi poste: Z Z f (x) dx = lim f (x) dx A

n→+∞ Dn

(5.5)

5.2 Rettangoloide generalizzato

261

Diamo ora un’interpretazione geometrica alla (5.5). A tale scopo consideriamo il sottoinsieme di R2 : def

U =



(x, y) ∈ R2 | x ∈ A − S, 0 ≤ y ≤ f (x)

Tale insieme di punti si chiama rettangoloide generalizzato di base A − S, relativo a f (x). Si osservi che U è illimitato in uno dei seguenti casi: 1) A è limitato e f (x) non limitata (cioè dotata di singolarità); 2) A illimitato e f (x) limitata (dotata al più di punti di discontinuità eliminabili o di prima specie); 3) A e f (x) non limitati. Ciò premesso, sussiste il Teorema 22. Nelle ipotesi poste: misU =

Z

f (x) dx

A

Dimostrazione. Omessa. Osserviamo che la misura di U può essere finita anche se U è illimitato, come nel caso della funzione: 1 con x ∈ A = [−1, 1] 1 − x2 Tale funzione è generalmente continua in A: f (x) = √

Più precisamente:

S = {−1, 1} =⇒ A − S = (−1, 1)

lim f (x) = +∞

x→1−

lim f (x) = +∞

x→−1+

Il rettangoloide generalizzato è:   1 2 U = (x, y) ∈ R | −1 < x < 1, 0 ≤ x ≤ √ 1 − x2 Costruiamo la successione {Dn }n∈N :   1 1 Dn = −1 + , 1 − n n come mostrato in figura 5.2.

262

y

5.2 Rettangoloide generalizzato

x

-1+1/n

Figura 5.2: Grafico della funzione f (x) =

1-1/n

√ 1 . 1−x2

Quindi: Z1

−1

dx √ = lim 1 − x2 n→+∞

1 1− n

Z



1 −1+ n

dx 1 − x2

     1 1 = lim arcsin 1 − − arcsin −1 + n→+∞ n n = arcsin (1) − arcsin (−1) = π,

donde: misU = π Un esempio di insieme illimitato di misura infinita è dato dal rettangoloide generalizzato relativo alla seguente funzione: f (x) = f (x) è non negativa, inoltre:

ex

1 −1

con x ∈ A = [0, 1]

lim f (x) = +∞,

x→0+

5.2 Rettangoloide generalizzato

263

donde: A − S = (0, 1]

Costruiamo la successione di domini:

come mostrato in figura 5.3.

 1 ,1 , Dn = n 

14

12

10

y

8

6

4

2

0 0

1/n

0.2

0.4

0.6

0.8

x

Figura 5.3: Grafico della funzione f (x) = Evidentemente: lim Dn = (0, 1]

n→+∞

Abbiamo: Z1 0

dx = lim ex − 1 n→+∞

Z1

1/n

dx ex − 1

1  = lim ln 1 − e−x 1/n n→+∞ = ln 1 − e−1 − ln 0+ = +∞,

1 . ex −1

1

5.2 Rettangoloide generalizzato

264

donde: misU = +∞ Riportiamo di seguito un esempio di integrale di una funzione continua esteso ad un intervallo illimitato. 1 per x ∈ A = (−∞, +∞) 1 + x2 La funzione (5.6) è manifestamente continua ed infinitesima all’infinito:

(5.6)

f (x) =

1 = 0+ 2 |x|→+∞ 1 + x lim

Costruiamo la successione (5.4) assumendo: Dn = [−n, n] come mostrato in fig. 5.2.

1

0.8

y

0.6

0.4

0.2

0 -3

-2

-1

0 x

1

Figura 5.4: Grafico della funzione f (x) = donde:

2

1 . 1+x2

3

5.2 Rettangoloide generalizzato

Z+∞

−∞

265

Zn dx dx = lim 2 n→+∞ 1+x 1 + x2 −n

= lim [arctan (n) − arctan (−n)] n→+∞ π  π = − − =π 2 2

Quindi anche in questo caso abbiamo un insieme illimitato di misura finita:  U = (x, y) ∈ R2 : −∞ < x < +∞, 0 ≤ y ≤ f (x) misU = π *** Calcoliamo l’integrale: I=

xZ0 +h

f (x) dx,

x0 −h

essendo:

f (x) =

1 |x − x0 |α

(5.7)

Qui è A = [x0 − h, x0 + h], essendo per ipotesi x0 ∈ R, e h, α > 0. L’integrando ha una singolarità in x0 : 1 α = +∞, x→x0 |x − x0 | lim

donde A − S = A − {x0 }. Costruiamo la successione (5.4) assumendo:     1 1 ∪ x0 + , x0 + h Dn = x0 − h, x0 − n n come mostrato in fig. 5.5. Quindi:



 I = lim  n→+∞

1 x0 − n

Z

x0 −h

dx + |x − x0 |α

xZ0 +h

1 x0 + n



dx   |x − x0 |α

(5.8)

5.2 Rettangoloide generalizzato

266

140

120

100

80

60

40

20

0 x -h 0

x0

x0+h

Figura 5.5: Grafico della funzione f (x) =

1 . |x−x0 |α

Poniamo: 1 x0 − n

I1 =

Z

x0 −h

I2 =

dx |x − x0 |α

xZ0 +h

1 x0 + n

dx |x − x0 |α

Eseguendo il cambio di variabile x → ξ = x − x0 : 1

Z− n dξ I1 = − ξα −h

I2 =

Zh

dξ ξα

1 n

In virtù della simmetria di Γ)y = f (ξ) rispetto all’asse delle ordinate: I1 = I2 , per cui:

5.2 Rettangoloide generalizzato

267

Zh

I = 2 lim

n→+∞

dξ ξα

1 n

Se α 6= 1:   2 1 2h1−α I= − 0 = , se α ∈ (0, 1) 1 − α hα−1 1−α   2 1 I=− − (+∞) = +∞, se α ∈ (1, +∞) α − 1 hα−1 Se α = 1 I = 2 lim ln (nh) = +∞ n→+∞

Cioè: I=



2h1−α , 1−α

se α ∈ (0, 1) +∞, se α ∈ [1, +∞)

In fig. 5.6 sono riportati i grafici della funzione (5.7) per x0 = 5, h = 2, nei due casi: a = 0.4, 2. 3

2.5

y

2

1.5

1

0.5

x0

0 3

3.5

4

4.5

5 x

5.5

6

Figura 5.6: Grafico della funzione (5.7).

6.5

7

5.2 Rettangoloide generalizzato

268

Per quanto visto l’integrale (5.8) converge solo per α ∈ (0, 1): lim I (α) = +∞

α→1−

In fig. 5.7 è riportato l’andamento di I in funzione del parametro α. 200 180 160 140

I(α)

120 100 80 60 40 20 0 0

0.2

0.4

0.6

0.8

1

α

Figura 5.7: Andamento dell’integrale I in funzione di α. *** Calcoliamo gli integrali:

I=

Z+∞

f (x) dx

(5.9)

x0 +h

J= essendo

xZ0 −h

g (x) dx,

−∞

1 (x − x0 )α 1 g (x) = (x0 − x)α

f (x) =

(5.10)

5.2 Rettangoloide generalizzato

269

Nella (5.10) x0 è un punto arbitrario dell’asse x, mentre α, h parametri positivi. Il grafico è riportato in fig. (5.8). 0.25

0.2

y

0.15

0.1

0.05

0

x0+h

x

x0+n

Figura 5.8: Grafico della funzione (5.10). Costruiamo i domini Dn : Dn = [x0 + h, x0 + n] Quindi: I = lim In , n→+∞

essendo: In =

xZ0 +n

x0 +h

dx (x − x0 )α

Risulta: n1−α − h1−α , se α 6= 1 1−α n In = ln , se α = 1 h

In =

Quindi:

5.2 Rettangoloide generalizzato

270

n1−α − h1−α I (α 6= 1) = lim = n→+∞ 1−α n I (α = 1) = lim ln = +∞ n→+∞ h



+∞, se 0 < α < 1 h1−α , se α > 1 α−1

Si conclude che l’integrale converge solo per α > 1. 100

80

y

60

40

20

0 x0+h

x +h x0

L’integrale J (secondo degli integrali (5.9)) assume lo stesso valore di I, in virtù della simmetria dell’integrando. Infatti possiamo scrivere:

I=

Z+∞

f (x) dx

x0 +h

J=

xZ0 −h

f (x) dx,

−∞

avendo ridefinito la funzione integranda: f (x) =

1 |x − x0 |α

Tale funzione è manifestamente simmetrica rispetto alla retta x − x0 = 0, per cui:

5.2 Rettangoloide generalizzato

271

Z+∞ dξ I= ξα h

J =−

Z−h

dξ ξα

−∞

*** Calcoliamo l’integrale: Z+∞ e−|x| dx I= −∞

Procediamo per decomposizione:

I=

Z0

−|x|

e

−∞

Z+∞ dx + e−|x| dx 0

def

= I1 + I2 ,

essendo:

I1 =

Z0

−|x|

e

dx =

Z0

−∞

−∞

0

0

ex dx

Z+∞ Z+∞ −|x| e−x dx e dx = I2 = Eseguiamo in I1 il cambio di variabile: x → x′ = −x. Osserviamo che: per cui:

Quindi:

−∞ ≤ x = −x′ ≤ 0, +∞ ≥ x′ ≥ 0

5.2 Rettangoloide generalizzato

272

I1 = −

Z0



e−x dx′

+∞

Z+∞



ex dx′ = I2 ,

=

0

donde: Z+∞ I = 2 e−x dx 0

Assumendo: Dn = [0, n] , si ha:

I = 2 lim

Zn

n→+∞

e−x dx

0

= 2 lim

n→+∞

1 − e−n

=2



che è la misura del rettangoloide generalizzato:  U = (x, y) ∈ R2 | −∞ < x < +∞, 0 ≤ y ≤ e−|x| ,

come riportato in figura (5.9).

*** Passiamo ora al caso in cui la funzione f (x) è generalmente continua e di segno qualunque in A ⊆ R. Definiamo: f1 (x) =

|f (x)| − f (x) |f (x)| + f (x) , f2 (x) = 2 2

(5.11)

Quindi: f (x) = f1 (x) − f2 (x)

(5.12)

5.2 Rettangoloide generalizzato

273

1

0.8

y

0.6

0.4

0.2

0 -4

-3

-2

-1

0 x

1

2

3

4

Figura 5.9: Grafico di f (x) = e−|x| . Evidentemente: f1 (x) = f (x) , per x ∈ A | f (x) > 0 f1 (x) = 0, altrimenti Mentre: f2 (x) = −f (x) , per x ∈ A | f (x) < 0 f2 (x) = 0, altrimenti Sempre dalla (5.11) segue ∀x ∈ A, f1 (x) ≥ 0, f2 (x) ≥ 0

Cioè le (5.11) sono non negative in A, e al pari di f (x) sono generalmente continue in A. Più precisamente, se S è l’insieme dei punti di discontinuità di f (x), indicando con S1 , S2 l’insieme dei punti di discontinuità di f1 (x) , f2 (x) rispettivamente, segue: Sk ⊆ S

con k = 1, 2

Verifichiamo la (5.13) attraverso degli esempi. Sia ξ ∈ S tale che:

(5.13)

5.2 Rettangoloide generalizzato

274

y

x Ξ

lim f (x) = l1 ∈ (−∞, 0)

x→ξ −

lim f (x) = 0

x→ξ +

Un possibile andamento è illustrato in fig. (5.2). Abbiamo: lim f (x) = l1 ∈ (−∞, 0) =⇒ (∃δ > 0 | x ∈ (ξ − δ, ξ) ∩ A =⇒ f (x) < 0 =⇒ f1 (x) = 0)

x→ξ −

Cioè: lim f1 (x) = 0

x→ξ −

lim f1 (x) = 0,

x→ξ +

da cui la continuità di f1 (x) in ξ. Quindi ξ ∈ S, ξ ∈ / S1 . Per la f2 (x) consideriamo quest’altro esempio: lim f (x) = l2 ∈ (0, +∞)

x→ξ +

lim f (x) = 0

x→ξ −

Abbiamo:

5.2 Rettangoloide generalizzato

275

lim f (x) = l2 ∈ (0, +∞) =⇒ (∃δ > 0 | x ∈ (ξ, ξ + δ) ∩ A =⇒ f (x) > 0 =⇒ f2 (x) = 0)

x→ξ +

Cioè: lim f2 (x) = 0

x→ξ −

lim f2 (x) = 0,

x→ξ +

da cui la continuità di f2 (x) in ξ. Quindi ξ ∈ S, ξ ∈ / S2 . L’andamento è illustrato in fig. (5.2).

Ξ

Ricapitolando: assegnata la funzione f (x) generalmente continua in A e di segno qualunque, restano definite le funzioni (5.11) non negative e generalmente continue in A. La funzione f (x) si esprime poi attraverso la (5.12). Per quanto detto, le fk (x) sono non negative, quindi hanno senso gli integrali: Z Z f2 (x) dx ≤ +∞ (5.14) f1 (x) dx ≤ +∞, A

A

Si noti che il rettangoloide generalizzato relativo ad una f (x) che cambia segno in A, sarà costituito da punti di ordinata y < 0; ad esempio consideriamo la funzione il cui grafico è riportato in figura (5.10). Il rettangoloide generalizzato è riportato in fig. (5.2). Passando alle funzioni (5.11) possiamo definire i seguenti rettangoloidi generalizzati:

5.2 Rettangoloide generalizzato

276

y

x Ξ1

Ξ2

Figura 5.10: Grafico di una funzione generalmente continua e di segno qualunque.

y

x Ξ1

Ξ2

5.2 Rettangoloide generalizzato

277

U1 = {(x, y) ∈ R | x ∈ A − S, 0 ≤ y ≤ f1 (x)} U2 = {(x, y) ∈ R | x ∈ A − S, − f2 (x) ≤ y ≤ 0} , riportati in figg. (5.11)-(5.12). y

x Ξ1

Ξ2

Figura 5.11: Rettangoloide U1 y x Ξ1

Ξ2

Figura 5.12: Rettangoloide U2 Evidentemente: misU1 =

Z

f1 (x) dx,

A

mentre: Z Z misU2 = − [−f2 (x)] dx = f2 (x) dx A

In forza delle (5.14):

A

5.2 Rettangoloide generalizzato

278

misU1 ≤ +∞, misU2 ≤ +∞

D’altro canto la (5.12) suggerisce di porre: Z Z Z f (x) dx = f1 (x) dx − f2 (x) dx A

A

A

(5.15)

La (5.15) non è accettabile se i due integrali a secondo membro sono entrambi infiniti; da un punto di vista geometrico significa che i due rettangoloidi U1 e U2 hanno entrambi misura infinita. In tal caso la (5.15) si presenta nella forma indeterminata ∞ − ∞. Ciò implica la seguente Definizione. La funzione f (x) è integrabile in A se almeno uno dei due integrali (5.14) è < +∞. In particolare: Z A

Z

Z

f1 (x) dx < +∞,

A

Z

Z

f1 (x) dx < +∞,

Z

f1 (x) dx = +∞,

A

A

A

A



f2 (x) dx < +∞ =⇒ 

f2 (x) dx = +∞ =⇒ 

f2 (x) dx < +∞ =⇒

Z

f (x) dx = +∞

Z

f (x) dx = −∞

Z

f (x) dx < +∞

A

A

A

Da un punto di vista geometrico: Z f (x) dx = misU1 − misU2 , A

R cioè RA f (x) dx ha senso solo se uno dei due rettangoloidi ha misura finita. In tal caso A f (x) dx è la differenza tra le due aree, e può essere finita o pari a +∞ o −∞. Per quanto detto: Z misU1 = +∞, misU2 = +∞) =⇒ f (x) dx non ha senso A

Si osservi che potremmo comunque scegliere una particolare successione {Dn } tale da rimuovere la forma indeterminata ∞ − ∞, ma in tal caso il valore assunto dall’integrale viene a dipendere dalla scelta della successione, per cui questa non

5.2 Rettangoloide generalizzato

279

è una definizione operativa. Inoltre molte proprietà classiche non sono più verificate. Tuttavia la rimozione della forma indeterminata è spesso utilizzata nelle applicazioni. Si parla in tal caso di “integrali impropri”. Assumiamo per ora la definizione precedente: Z

f (x) dx =

Z

f1 (x) dx −

A

A

A

Z

⇐⇒ ∃k ∈ {1, 2} |

(5.16)

f2 (x) dx ⇐⇒ Z

fk (x) dx < +∞

A

Si osservi che la (5.16) contiene come caso particolare la definizione di integrale per una funzione generalmente continua e non negativa. Infatti: ∀x ∈ A, f (x) ≥ 0 =⇒ f1 (x) ≡ f (x) , f2 (x) ≡ 0 =⇒

Z

f (x) dx =

A

Z

Z

Z f (x) dx = − f2 (x) dx

f1 (x) dx

A

(5.17)

Viceversa:

∀x ∈ A, f (x) ≤ 0 =⇒ f1 (x) ≡ 0, f2 (x) ≡ −f (x) =⇒

A

A

(5.18)

Teorema 23. Sia f (x) una funzione generalmente continua in A ed ivi di segno qualunque. Se f (x) è integrabile in A, indicato con S l’insieme dei punti di discontinuità, risulta: Z Z ∀ {Dn }n∈N | lim Dn = A − S, lim f (x) dx = f (x) dx n→+∞

n→+∞ Dn

A

Dimostrazione. Omessa Si osservi che tale teorema è applicabile solo se la funzione è integrabile. In altri termini, l’integrabilità della funzione va stabilita a priori. Esempio 24. Studiamo l’integrabilità di f (x) = log x in A = [0, 1]. x0 = 0 è una singolarità poichè limx→0+ log x = −∞, quindi A − S = (0, 1]. D’altro canto è f (x) ≤ 0 in A, per cui la funzione è ivi integrabile. Più precisamente (per la (5.18)) Z Z f (x) dx = −

A

f2 (x) dx

A

5.2 Rettangoloide generalizzato Risulta:

Z A

280

f (x) dx = lim

Z

n→+∞ Dn

f (x) dx

Assumiamo come successione di domini limitati e misurabili di continuità per f (x):   1 Dn = ,1 (5.19) n Ciò è riportato in fig. (24). y

1 n

x 1

-1

-2

-3

-4

Al crescere di n, il dominio (5.19) si “avvicina” all’intervallo A, come mostrato in fig. (24). y

1 n -1

-2

-3

-4

x 1

5.2 Rettangoloide generalizzato

281

Quindi: Z1

log xdx = lim

Z1

n→+∞ 1/n

0

log xdx

Risulta: Z1

log xdx =

1/n

= [x log x]x=1 x= 1 − n

Z1

dx

1/n

  1 1 1 −1+ = − ln n n n Quindi: Z1

1 + ln n n→+∞ n

log xdx = −1 + lim

0

Ma:

1 ln n 1 + ln n = lim + lim = 0, n→+∞ n n→+∞ n→+∞ n | {z } | {z n } lim

=0

donde:

Z1

=0

log xdx = −1

0

Esempio 25. Studiamo l’integrabilità f (x) = e−x sin x nell’intervallo A = [0, +∞). Qui abbiamo una funzione continua in un intervallo illimitato. Osserviamo che: |f (x)| = e−x |sin x| ≤ e−x =⇒ −e−x ≤ f (x) ≤ ex Cioè f (x) è un’oscillazione sinuisoidale il cui inviluppo di modulazione è y = ±e−x . Inoltre: lim f (x) = 0 x→+∞

Il grafico è riportato in fig. (5.13). f (x) cambia segno infinite volte in A, per cui determiniamo le funzioni: f1 (x) =

|f (x)| + f (x) |f (x)| − f (x) , f2 (x) = 2 2

5.2 Rettangoloide generalizzato

282

y

Π

fH 2 L

x

Π 2

Π



Figura 5.13: Grafico di f (x) = e−x sin x in [0, 2π]. Risulta:

 [ −x  e sin x, x ∈ [2kπ, (2k + 1) π]   k∈N [ f1 (x) =  0, x ∈ [(2k + 1) π, (2k + 2) π]   k∈N

 [ −x  −e sin x, x ∈ [(2k + 1) π, (2k + 2) π]   k∈N [ f2 (x) =  0, x ∈ [2kπ, (2k + 1) π]   k∈N

Restano così definiti i rettangoloidi generalizzati:  U1 = (x, y) ∈ R2 | x ∈ A, 0 ≤ y ≤ f1 (x)  U2 = (x, y) ∈ R2 | x ∈ A, − f2 (x) ≤ y ≤ 0 Poniamo:

 W = (x, y) ∈ R2 | x ∈ A, 0 ≤ y ≤ e−x ,

donde:

Z+∞ misW = e−x dx = 1 0

Le curve y = e sin x, y = e sono tangenti nei punti di intersezione xk = π + 2kπ come possiamo vedere in fig. (25). 2 Poniamo:  U2′ = (x, y) ∈ R2 | x ∈ A, 0 ≤ y ≤ f2 (x) −x

−x

5.2 Rettangoloide generalizzato

283

y

Π

fH 2 L

Π 2

x Π

Evidentemente: misU2 = misU2′ Inoltre dal grafico di fig. (5.14). y 0.4

0.3

0.2

0.1

Π 2

x Π



Figura 5.14: segue: misU1 + misU2′ < misW = 1 =⇒ misU1 + misU2 < 1 Cioè: misU1 < +∞, misU2 < +∞ Si conclude che f (x) è integrabile in A. Per il calcolo dell’integrale, assumiamo: Dn = [0, n]

5.2 Rettangoloide generalizzato

284

Quindi: Z+∞ Zn −x e sin xdx = lim e−x sin xdx n→+∞

0

L’integrale

Rn 0

−x

e

0

sin xdx si calcola per parti:

Zn

1 1 e−x sin xdx = − e−n [sin (n) + cos (n)] + 2 2

0

Osservando che: lim e−n [sin (n) + cos (n)] = 0,

n→+∞

si conclude:

Z+∞ 1 e−x sin xdx = 2 0

Esempio 26. Studiamo l’integrabilità della funzione:  x e sin x, x ∈ [0, π] ∪ [2π, 3π] ∪ [4π, 5π] ∪ ... f (x) = −x e sin x, x ∈ [π, 2π] ∪ [3π, 4π] ∪ [5π, 6π] ∪ ... nell’intervallo A = [0, +∞]. Osserviamo che il grafico di f (x) è un’oscillazione sinusoidale il cui inviluppo di modulazione è y = ex per le semionde positive e y = e−x per le semionde negative.  x e sin x, x ∈ [0, π] ∪ [2π, 3π] ∪ [4π, 5π] ∪ ... f1 (x) = 0, x ∈ [π, 2π] ∪ [3π, 4π] ∪ [5π, 6π] ∪ ... f2 (x) =



−e−x sin x, x ∈ [π, 2π] ∪ [3π, 4π] ∪ [5π, 6π] ∪ ... 0, x ∈ [0, π] ∪ [2π, 3π] ∪ [4π, 5π] ∪ ...

Quindi i rettangoloidi:

 U1 = (x, y) ∈ R2 | x ∈ A, 0 ≤ y ≤ f1 (x)  U2 = (x, y) ∈ R2 | x ∈ A, − f2 (x) ≤ y ≤ 0

Appendice A Integrali notevoli Z

A.1

sin2 xdx, Z

Z

cos2 xdx

1 1 (x − sin x cos x) + C = (2x − sin 2x) + C 2 4 Z 1 1 cos2 xdx = (x + sin x cos x) + C = (2x + sin 2x) + C 2 4

A.2 1. 2. 3.

Z

Z

Z

4.

Z

5.

Z

Z dx x2 −1

sin2 xdx =

(A.1)

dx n 2 x ( −1) = − arctanh x+const

dx (x2 −1)2 dx (x2 −1)3

+const − 14 ln x−1 x+1

=

x 2(1−x2 )

=

x(3x2 −5) 8(x2 −1)

2

+

3 16

+const ln x−1 x+1

dx 1 1 = − 48(−1+x) 3 + 16(−1+x)2 (x2 −1)4 5 1 5 − 32(1+x) + 32 ln (1 + x) 16(1+x)2



5 32(−1+x)



5 32

ln (−1 + x) −

35 1 dx 5 15 35 = − 128(−1+x) − 256(−1+x) 4+ 2 + 256(−1+x) + 256 192(−1+x)3 (x2 −1)5 35 1 5 15 35 − 256 ln (1 + x) + 192(1+x) + 256(1+x) 3 + 128(1+x)4 256(1+x)2

1 48(1+x)3



ln (−1 + x)+

A.3

Z

6.

Z

dx (x2 +1)n

dx (x2 −1)6

63 512

7.

8.

2. 3. 4. 5. 6.

3 7 7 63 − 256(−1+x) − 512(−1+x) − 3 + 256(−1+x)4 128(−1+x)2 3 7 7 63 1 63 − 256(1+x) − 512(1+x) x)− 320(1+x) + 512 ln (1 + 5− 3− 256(1+x)4 128(1+x)2

1 = − 320(−1+x) 5 +

ln (−1 +

Z

7 7 7 105 dx 1 − 512(−1+x) − 2048(−1+x) = − 768(−1+x) 6 + 4 + 2 1280(−1+x)5 256(−1+x)3 (x2 −1)7 7 7 7 231 231 1 + 512(1+x) + 2048 ln (−1 + x) + 768(1+x) 6 + 4 + 2048(−1+x) 1280(1+x)5 256(1+x)3 105 231 231 − 2048 ln (1 + x) + 2048(1+x) 2048(1+x)2

+

Z

+

dx 1 1 9 15 55 = − 1792(−1+x) − 1280(−1+x) − 2048(−1+x) 7 + 5 + 3 384(−1+x)6 1024(−1+x)4 (x2 −1)8 429 1 429 1 9 99 − 4096 ln (−1 + x) − 1792(1+x) − 4096(−1+x) − 1280(1+x) 7 − 5 2048(−1+x)2 384(1+x)6 55 99 429 15 429 − 2048(1+x)3 − 2048(1+x)2 − 4096(1+x) + 4096 ln (1 + x) 1024(1+x)4

A.3 1.

286

Z

Z

Z

Z

Z

Z

7.

Z

8.

Z

9.

Z

Z dx x2 +1

x)

+



dx

n x2 +1

(

)

= arctan x+const

dx (x2 +1)2

=

1 x 2 x2 +1

dx (x2 +1)3

=

1 x 4 (x2 +1)2

+

3 x 8 x2 +1

dx (x2 +1)4

=

1 x 6 (x2 +1)3

+

5 x 24 (x2 +1)2

+

5 x 16 x2 +1

dx (x2 +1)5

=

x 1 8 (x2 +1)4

+

x 7 48 (x2 +1)3

+

x 35 192 (x2 +1)2

dx (x2 +1)6

=

1 9 21 21 63 x x x x x + 80 + 160 + 128 + 256 + 63 10 (x2 +1)5 x2 +1 256 (x2 +1)4 (x2 +1)3 (x2 +1)2

+ 12 arctan x+const + 38 arctan x

dx 1 11 x x = 12 + 120 (x2 +1)7 (x2 +1)6 (x2 +1)5 x 231 231 + 1024 arctan x 1024 x2 +1

+

+

5 16

arctan x+const

+

x 35 128 x2 +1

33 x 320 (x2 +1)4

dx x x x 1 13 143 = 14 + 168 + 1680 (x2 +1)8 (x2 +1)7 (x2 +1)6 (x2 +1)5 429 x x 429 143 + 2048 + 2048 arctan x 1024 (x2 +1)2 x2 +1

+

+

+

35 128

77 x 640 (x2 +1)3

x 429 4480 (x2 +1)4

1 15 65 143 x x x x dx = 16 + 224 + 896 + 1792 (x2 +1)9 (x2 +1)8 (x2 +1)7 (x2 +1)6 (x2 +1)5 x x x 429 + 326435 arctan x + 162145 + 326435 4096 (x2 +1)3 384 (x2 +1)2 768 x2 +1 768

arctan x+const

77 x 512 (x2 +1)2

+

x 143 1280 (x2 +1)3

+

1287 x 14 336 (x2 +1)4

+

+

+

+

arctan x+const

A.4

Z

10.

Z

11.

Z

dx ; (sin x)n

Z

dx (cos x)n

287

dx 1 17 85 x x x x x = 18 + 288 + 1344 + 161105 + 322431 128 (x2 +1)6 256 (x2 +1)5 (x2 +1)10 (x2 +1)9 (x2 +1)8 (x2 +1)7 155 155 x x x x 155 2431 + 242431 + 12 + 12 + 12 arctan x 28 672 (x2 +1)4 576 (x2 +1)3 98 304 (x2 +1)2 65 536 x2 +1 65 536

dx 1 19 323 323 x x x x x = 20 + 360 + 5760 + 5376 + 644199 + 512 (x2 +1)6 (x2 +1)11 (x2 +1)10 (x2 +1)9 (x2 +1)8 (x2 +1)7 46 189 46 189 46 189 46 189 x x x x x 46 189 46 189 + 573 + 491 + 393 + 262 + 262 645 120 (x2 +1)5 440 (x2 +1)4 520 (x2 +1)3 216 (x2 +1)2 144 x2 +1 144

arctan x

A.4

Z

dx ; (sin x)n

Z

dx (cos x)n

Applicando le (3.108)-(3.109) per valori crescenti di n = 3, 4, 5: Z

Z

Z

Z

Z

Z

A.5

+

Z

dx sin3 x dx cos3 x dx sin4 x dx cos4 x dx sin5 x dx cos5 x

 x  cos x i 1 h ln tan +C − 2 2 sin2 x    x π  1 sin x ln tan +C = + + 2 2 4 cos2 x 1h cos x i = − 2 cot x + +C 3 sin3 x   1 sin x = tan x + +C 3 cos3 x   x  cos x  cos x  1 3  +C = − ln tan − 4 2 2 sin2 x sin4 x      x π  sin x 1 3 sin x ln tan + +C = + + 4 2 2 4 cos2 x cos4 x =

(tan x)n dx;

Z

(cot x)n dx

Applicando le (3.112)-(3.113) per n = 2, 3, ..., 8:

A.5

Z

n

(tan x) dx;

Z

Z

Z

Z

Z

Z

Z

Z

Z

Z

Z

Z

Z

Z

Z

(cot x)n dx

tan2 xdx = tan x − x + C cot2 xdx = − cot x − x + C  1 1 tan2 x − ln 1 + tan2 x + C 2 2  1 1 cot3 xdx = − cot2 x + ln 1 + cot2 x 2 2 1 tan4 xdx = tan3 x − tan x + x + C 3 1 cot4 xdx = − cot3 x + cot x + x + C 3  1 1 1 tan5 xdx = tan4 x − tan2 x + ln 1 + tan2 x + C 4 2 2  1 1 1 cot5 xdx = − cot4 x + cot2 x − ln 1 + cot2 x + C 4 2 2 1 1 tan6 xdx = tan5 x − tan3 x + tan x − x + C 5 3 1 1 cot6 xdx = − cot5 x + cot3 x − cot x − x + C 5 3  1 1 1 1 tan7 xdx = tan6 x − tan4 x + tan2 x − ln 1 + tan2 x + C 6 4 2 2  1 1 1 1 cot7 xdx = − cot6 x + cot4 x − cot2 x + ln 1 + cot2 x + C 6 4 2 2 1 1 1 tan8 xdx = tan7 x − tan5 x + tan3 x − tan x + x + C 7 5 3 1 1 1 cot8 xdx = − cot7 x + cot5 x − cot3 x + cot x + x + C 7 5 3

tan3 xdx =

288

Related Documents

Int
April 2020 38
Int
November 2019 46
Int
August 2019 52
Int
November 2019 57
02283-int
October 2019 8